Sei sulla pagina 1di 139

UNIVERSIDADE FEDERAL DE SANTA MARIA CENTRO DE CIENCIAS NATURAIS E EXATAS DEPARTAMENTO DE MATEMATICA CADERNO DIDATICO

Geometria Plana e Desenho Geomtrico e

Joo Batista Peneireiro a Maur Fronza da Silva cio

Santa Maria, Inverno de 2008 (Reviso) a

Apresentao ca
Prezado leitor, estas anotaes de Geometria Euclidiana Plana so material de notas co a de aula da disciplina de Geometria Plana e Desenho Geomtrico do Curso de Matemtica e a da UFSM. Elas servem de s ntese para os tpicos tratados na disciplina e nem de longe o pretendem ser completas e sem falhas. Como orientao para o uso destas notas, queremos chamar a ateno para a necessica ca dade de dar-se uma introduo sobre o mtodo dedutivo, com nfase ao racioc lgico e ca e e nio o as regras de lgica que so, em geral, utilizadas ao longo de todo curso. Ateno especial o a ca deve ser dada ao mtodo de reduo ao absurdo (reductio ad absurdum). e ca Ao ler este texto, o leitor deve estar municiado de lpis para poder acoma panhar o racioc nio com esquemas (desenhos). Os desenhos so dispensveis a a para as demonstraes apresentadas, mas revelam-se um bom aux para o co lio entendimento das mesmas. Lembre-se, toda demonstrao dever ser indeca a pendente dos desenhos, o uso destes somente um auxiliar util. Argumentaes e co baseadas em desenhos podem levar a resultados absurdos (veja o surpreendente Apndice e A). Estas anotaes constituem uma verso corrigida e ampliada daquelas do inverno de co a 2001. Todas as sugestes que visem a melhoria deste texto so bem vindas, erros que o a eventualmente existam no texto devem ser debitados, exclusivamente, aos autores.

Joo Batista Peneireiro (peneireiro@smail.ufsm.br) a Maur Fronza da Silva (mfronza@smail.ufsm.br) cio

Lista de S mbolos
A, B, C, ... r, s, t, ... AOB, O , , , ... 1, 2, 3, ... AB AB ABC, ABC AB rP x (A) ponto reta a ngulo de vrtice O ou medida de ngulo e a a ngulo ou medida de ngulo a a ngulo ou medida de ngulo a segmento de extremidades A e B ou reta que passa por A, B medida do segmento AB tringulo de vrtices A, B, C a e

semi-reta de origem A contendo B semiplano determinado por r contendo P coordenada do ponto A em relao a um sistema de ca coordenadas da reta C (O, r) circunferncia de centro O e raio r e AB CD segmento AB congruente ao segmento CD AOB C O D ngulo AOB congruente ao ngulo C O D a a ABC XY Z tringulo ABC congruente ao tringulo XY Z, e a cona a gruncia a aplicao tal que A X, B Y, C Z e e ca r s retas r, s so paralelas a rs retas r, s so perpendiculares a A1 A2 ...An pol gono de vrtices A1 , A2 , ..., An e ABCD quadriltero cujos lados opostos so AB, CD; e AC, BD a a ABC XY Z tringulo ABC semelhante ao tringulo XY Z, e a semela a hana a aplicao tal que A X, B Y, C Z c e ca AB m(AB) tAB ln an a (R) AC B arco AB medida do arco AB (em graus) a ngulo em que um dos lados passa por B e o outro a e semi-reta t lado de um pol gono regular de n lados inscrito em uma circunferncia e aptema de um pol o gono regular de n lados inscrito em uma circunferncia e a rea da regio poligonal R a C est entre A e B. a

Sumrio a
1 A Origem da Geometria e o Mtodo Axiomtico e a 9

2 Axiomas de Incidncia e da Ordem e 13 2.1 Axiomas de incidncia . . . . . . . . . . . . . . . . . . . . . . . . . . . . . 13 e 2.2 Axiomas da ordem . . . . . . . . . . . . . . . . . . . . . . . . . . . . . . . 15 2.3 Exerc cios complementares . . . . . . . . . . . . . . . . . . . . . . . . . . . 20 3 Axiomas Sobre Medida de Segmentos e de Angulos 3.1 Axiomas sobre medida de segmentos . . . . . . . . . . . . . . . . . . . . . 3.2 Angulo e Axiomas sobre medida de ngulos . . . . . . . . . . . . . . . . . a 3.3 Exerc cios complementares . . . . . . . . . . . . . . . . . . . . . . . . . . . 23 23 28 32

4 Axiomas de Congruncia e 35 4.1 Axiomas sobre congruncia de segmentos, ngulos e tringulos . . . . . . . 35 e a a 4.2 Exerc cios complementares . . . . . . . . . . . . . . . . . . . . . . . . . . . 39 5 Desigualdades nos Tringulos a 43 5.1 O teorema do ngulo externo e algumas de suas conseqncias . . . . . . . 43 a ue 5.2 Desigualdades nos tringulos . . . . . . . . . . . . . . . . . . . . . . . . . . 45 a 5.3 Exerc cios complementares . . . . . . . . . . . . . . . . . . . . . . . . . . . 47 6 Axioma das Paralelas 51 6.1 O axioma das paralelas e algumas consequncias . . . . . . . . . . . . . . . 51 e 6.2 Construo de um sistema de coordenadas no plano . . . . . . . . . 55 ca 6.3 Exerc cios complementares . . . . . . . . . . . . . . . . . . . . . . . . . . . 55 7 Pol gonos 57 7.1 Denies gerais . . . . . . . . . . . . . . . . . . . . . . . . . . . . . . . . . 57 co 7.2 Quadrilteros convexos . . . . . . . . . . . . . . . . . . . . . . . . . . . . . 60 a 8 Paralelismo e o Teorema do Feixe de Retas Paralelas 63 8.1 Feixe de retas paralelas . . . . . . . . . . . . . . . . . . . . . . . . . . . . . 63 8.2 Exerc cios complementares . . . . . . . . . . . . . . . . . . . . . . . . . . . 67 7

SUMARIO

SUMARIO

9 Tringulos Semelhantes e Semelhana de Pol a c gonos 69 9.1 Denies e consideraes gerais . . . . . . . . . . . . . . . . . . . . . . . . 69 co co 9.2 Casos de semelhana de tringulos e o Teorema de Pitgoras . . . . . . . . 70 c a a 9.3 Exerc cios complementares . . . . . . . . . . . . . . . . . . . . . . . . . . . 74 10 Circunferncia e 10.1 Elementos da circunferncia . . . . . . . e 10.2 Angulos e arcos numa circunferncia . . e 10.3 Trigonometria . . . . . . . . . . . . . . . 10.4 Comprimento de uma circunferncia e de e 10.5 Exerc cios complementares . . . . . . . . 77 77 78 86 88 89 93 93 97 100

. . . . . . . . . . . . . . . . . . . . . arco de uma . . . . . . .

. . . . . . . . . . . . . . . . . . . . . . . . . . . circunferncia . e . . . . . . . . .

. . . . .

. . . . .

. . . . .

11 Resultados Notveis sobre Tringulos a a 11.1 Pontos notveis de um tringulo . . . . . . . . . . . . . . . . . . . . . . . . a a 11.2 Pontos de Brocard . . . . . . . . . . . . . . . . . . . . . . . . . . . . . . . 11.3 Exerc cios complementares . . . . . . . . . . . . . . . . . . . . . . . . . . . 12 Construes com Rgua e Compasso co e 12.1 Regras para construo com rgua e compasso . ca e 12.2 Alguns problemas de construo e suas solues ca co 12.3 Expresses algbricas . . . . . . . . . . . . . . . o e 12.4 Construo de tringulos . . . . . . . . . . . . . ca a 12.5 Problemas de tangncia . . . . . . . . . . . . . e 12.6 Exerc cios complementares . . . . . . . . . . . . 13 Area de Figuras Planas 13.1 Denies e axiomas . . . . . . . . . . . . co 13.2 Area de um quadrado . . . . . . . . . . . . 13.3 Area de um retngulo . . . . . . . . . . . . a 13.4 Area de paralelogramos e tringulos . . . . a 13.5 Area de um c rculo e de um setor circular . 13.6 Equivalncia plana . . . . . . . . . . . . . e 13.7 Exerc cios complementares . . . . . . . . . Apndice A e

. . . . . .

. . . . . .

. . . . . .

. . . . . .

. . . . . .

. . . . . .

. . . . . .

. . . . . .

. . . . . .

. . . . . .

. . . . . .

. . . . . .

. . . . . .

. . . . . .

101 . 101 . 102 . 106 . 108 . 110 . 113 119 119 121 122 123 126 127 128 135

. . . . . . .

. . . . . . .

. . . . . . .

. . . . . . .

. . . . . . .

. . . . . . .

. . . . . . .

. . . . . . .

. . . . . . .

. . . . . . .

. . . . . . .

. . . . . . .

. . . . . . .

. . . . . . .

. . . . . . .

. . . . . . .

. . . . . . .

. . . . . . .

Cap tulo 1 A Origem da Geometria e o Mtodo e Axiomtico a


D-se um curto apanhado histrico da geometria antiga, a o introduz-se comentrios sobre mtodo axiomtico e listaa e a se os termos no denidos a serem considerados neste traa balho. A palavra geometria vem do grego geometrien ( ) onde geo signica terra e metrien medida. Geometria foi, em sua origem, a cincia de medio de terras. e ca O historiador grego Herdoto (500 a.C.) atribuiu aos eg o pcios o in da geometria, cio mas outras civilizaes antigas (babilnios, hindus, chineses) tambm possuiam muitas co o e informaes geomtricas.1 co e A geometria dos povos antigos era uma coleo de regras obtidas a partir de experica mentaes e observaes de analogias, tentativas e ocasionais lampejos de intuiao. Foram co co c as necessidades prticas que impulsionaram a busca de respostas `s questes geomtricas, a a o e mesmo que s de forma aproximada. Assim, para os babilnios, no per o o odo de 2000 a 1600 a.C., a rea do c a rculo era calculada tomando trs vezes o quadrado do raio (isto , eles e e tomavam como 3 o valor ; este tambm era o valor de para os chineses naquela poca). e e 16 2 Os eg pcios de 1800 a.C., de acordo com o papiro de Rhind, usavam 9 3, 1604 como valor aproximado de . Muitas vezes, os eg pcios tinham clculos corretos; por exemplo, a conheciam a frmula correta para o clculo do volume de um tronco de pirmide de base o a a quadrada. Por outro lado, a frmula correta para o clculo da rea do retngulo, era por o a a a eles aplicada tambm a qualquer quadriltero. e a A geometria eg pcia no foi uma cincia no sentido entendido pelos gregos, era somente a e uma coleo de regras para clculo sem qualquer motivao ou justicativa. ca a ca A matemtica babilnica foi mais avanada que a dos eg a o c pcios na aritmtica e lgebra; e a alm disso, eles conheciam o tradicional teorema de Pitgoras, num tringulo retngulo o e a a a quadrado do comprimento da hipotenusa igual a soma dos quadrados dos comprimentos e dos catetos, bem antes mesmo do que Pitgoras. a
1

As idias de cunho histrico aqui expostas so anotaes baseadas no excelente livro [10]. e o a co

A Origem da Geometria e o Mtodo Axiomtico e a Foram os gregos, por volta de 600 a.C. com Tales de Mileto, que iniciaram as investigaes de cunho geomtrico, estabelecendo a necessidade de se empregar o mtodo co e e dedutivo no lugar do mtodo de tentativa e erro. Tales, segundo Proclus2 , visitou o Egito e e a Babilnia tendo trazido desses lugares os conhecimentos geomtricos da poca. Com o e e o objetivo de vericar a correo dos resultados executados, ele desenvolveu a primeira ca geometria lgica. O desenvolvimento organizado dos teoremas atravs de provas (demono e straes) foi caracter co stica da matemtica grega, e uma prtica inteiramente nova at a a e ento. a A sistematizao iniciada por Tales foi continuada, nos dois sculos seguintes, por ca e Pitgoras de Samos e seus disc a pulos. Em Crotona, sul da atual Itlia, ele fundou uma a irmandade que cou conhecida como Escola Pitagrica. Foram os pitagricos, membros o o dessa irmandade, que descobriram os nmeros irracionais, tais como 2, criando um u verdadeiro trauma na escola, uma vez que o lema fundamental dessa irmandade era que tudo nmero, onde por nmero entendia-se os inteiros positivos e razo entre eles. e u u a A fundamentao sistemtica da geometria plana foi realizada pela escola pitagrica, ca a o por volta de 400 a.C., em Elementos escrito pelo matemtico Hipcrates de Chios (no a o a confundir com o mdico). Um sculo depois, Euclides de Alexandria, publicou sua obra e e Elementos [12], onde reuniu, praticamente, quase tudo que se conhecia de matemtica a at ento. No sculo 4 a.C., Plato fundou sua famosa academia onde na entrada ese a e a tava xado o lema Que ningum que ignore a geometria entre aqui. Euclides foi um e disc pulo da escola platnica. Por volta de 300 a.C. ele produziu o tratamento denitivo o da geometria grega e da teoria dos nmeros nos seus treze volumes do Elementos. Neste u tratado, Euclides colocou os trabalhos de Pitgoras nos livros I-IV e IX; no livro VIII, os a de Architas; os de Eudoxo, nos livros V, VI e XII e os de Teeteto nos livros X e XIII. O livro Elementos se tornou, ao longo do tempo, a obra mais publicada e lida. Sua abordagem da geometria dominou o ensino desta matria por mais de 2000 anos. e Fora tudo disso, o mtodo axiomtico usado por Euclides em Elementos o prottipo e a e o de tudo que chamamos hoje de matemtica pura. Ela pura no sentido de puro a e pensar; nenhum experimento f sico preciso para vericar se suas armaes so corretas, e co a somente o racioc nio nas demonstraes precisa ser conferido. co Os Elementos de Euclides puro no sentido de que esse trabalho no inclui aplicaes e a co prticas. Naturalmente, a geometria de Euclides tem um grande nmero de aplicaes a u co a problemas prticos na engenharia, mas eles no so mencionados em Elementos. a a a Muitas vezes, resultados puramente matemticos passam a ter importncia em questes a a o aplicadas, sendo por isso uteis ` sociedade. Alm disso, aquelas partes da Matemtica a e a que no tm sido aplicadas so tambm vlidas ` sociedade, tanto como trabalhos a e a e a a estticos, comparados ` msica e ` arte, como contribuio ` expanso da conscincia e e a u a ca a a e do conhecimento do homem. Matemticos podem fazer uso da tentativa e erro, clculo de casos especiais, coma a putadores, ou outros meios para demonstrar teoremas. Para alguns dos mais importantes resultados em matemtica foram, originalmente, dadas provas incompletas (o ultimo Teoa
Proclus (410-488 d.C.), lsofo neoplatnico, escreveu Comentrio sobre o primeiro livro de Os o o a Elementos de Euclides.
2

10

A Origem da Geometria e o Mtodo Axiomtico e a rema de Fermat, por exemplo). Provas corretas sero dadas mais tarde e assim o trabalho a matemtico estar satisfeito. a a As provas nos do segurana de que os resultados so corretos. Em muitos casos elas a c a nos do resultados mais gerais. Um exemplo o Teorema de Pitgoras, que generaliza a e a resultados que os eg pcios, hindus e outros povos conheciam s casos particulares. Finalo mente, provas, muitas vezes, nos do uma viso de relaes entre coisas diferentes, nos a a co forando a organizar nossas idias de um modo coerente. c e Anal, o que o mtodo axiomtico? Se quisermos nos persuadir, por meio de e e a racioc da necessidade de alguma armao A1 , podemos mostrar como esta armao nio ca ca segue logicamente de outra armao A2 que podemos aceitar como verdadeira. No ca entanto, se no temos estabelecido A2 como verdadeira, podemos mostrar que A2 decorre a logicamente de A3 . Podemos ter que repetir este processo muitas vezes at atingirmos uma e armao que j tenhamos como certa e que no precise de justicativa. Essa armao, ca a a ca assumida como certa, toma o papel de um Axioma (ou Postulado). Se no pudermos a atingir uma armao que ser aceita como base de nossas argumentaes, estaremos ca a co mergulhados numa regresso innita, dando uma demonstrao aps outra, sem m. a ca o Assim, dois quesitos devem ser assumidos para que possamos concordar que uma prova est correta: a 1. Aceitao de algumas armaes chamadas Axiomas ou Posca co tulados sem justicao adicional. ca 2. Aceitao de certas regras de racioc ca nio, isto , aceitao de e ca como, e quando, uma armao segue logicamente de outra. ca A grande realizao de Euclides foi ter estabelecido, a partir de uns poucos postulados, ca 465 proposies (teoremas) mais complicadas e nem sempre intuitivas, onde est conco a tida toda a geometria conhecida de seu tempo, [12]. Uma razo para ser os Elementos a um belo trabalho que muito foi deduzido de to pouco. e a Tendo combinado as duas regras acima, para se poder concordar quando uma demonstrao est correta, antes de tudo, precisamos xar certas terminologias e s ca a mbolos a serem utilizados. Assim, combinamos tambm o seguinte: e 3. Os teoremas e s mbolos usados no sistema axiomtico devero a a ter signicado compreendido igualmente por todos. Temos o direito de dar denies de termos novos baseados em outros que assumico mos como indenidos. Assim, no desenvolvimento da geometria plana que faremos nos prximos cap o tulos, iremos assumir quatro termos que sero bsicos para denir todos os a a outros termos geomtricos, a saber: e - ponto; - reta; - incidente; - estar entre. Esta lista constitui-se dos termos geomtricos indenidos ou termos primitivos. e Todo corpo axiomtico que construiremos estar baseado nesses termos, e nas noes a a co algbricas de conjunto, correspondncia, aplicao, etc. e e ca 11

A Origem da Geometria e o Mtodo Axiomtico e a

12

Cap tulo 2 Axiomas de Incidncia e da Ordem e


O ambiente no qual constri-se a geometria chamado o e plano, e denominam-se ponto e reta os conceitos geomtricos primitivos (objetos primitivos) do plano. e Assume-se que o plano constitu por pontos e que retas e do so subconjuntos distinguidos de pontos do plano. Listaa se um grupo de axiomas, em n mero de sete, que xaro u a relaes e deniro conceitos relativos aos objetos primico a tivos (no se apresenta axiomas relacionando esses entes a no contexto do espao por s estar-se interessado em dec o senvolver geometria de objetos planos).

2.1

Axiomas de incidncia e
... nos dizem sobre a disposio m tua de pontos e retas; ca u caracteriza reta.

Axioma 1 Dados quaisquer dois pontos distintos, A e B, existe uma unica reta que os contm. e
 

2.1 Observao. Reservaremos as letras maisculas do alfabeto latino para nomear ca u ponto e as letras minsculas para nomear retas. Assim, dizemos simplismente ponto u P e reta r, por exemplo. 2.2 Observao. Usando a notao da teoria dos conjuntos podemos, sinteticamente, ca ca escreve o axioma 1 do seguinte modo: A, B, A = B | r : A, B r. 2.3 Observao. As expresses um ponto pertence a uma reta, um ponto incidente ca o e em uma reta, um ponto est sobre uma reta e uma reta passa por um ponto so a a assumidas como equivalentes. 13

Axiomas de incidncia e 2.4 Observao. De acordo com este axioma, somente uma reta passa atravs de dois ca e pontos. Segue-se da que uma reta est completamente determinada pela especicao , a ca de dois pontos. A partir disto, denotaremos ento a reta que passa pelos pontos A e B, a por reta AB. 2.5 Denio. Se um ponto comum a duas retas dizemos que elas se interceptam ca e nesse ponto, ou que esse ponto um ponto de interseo dessas retas. Duas retas que e ca se interceptam num unico ponto so ditas concorrentes ou mutuamente transversais. a 2.6 Denio. A coleo de todos os pontos chamaremos de Plano. Nomearemos planos ca ca utilizando-se das letras do alfabeto grego. Por exemplo: plano .


Axioma 2 Em cada reta existem ao menos dois pontos distintos.


 

Axioma 3 Existem trs pontos distintos que no pertencem a uma mesma reta. e a


2.7 Observao. O axioma 3 diz que reta um subconjunto prprio do plano. ca e o 2.8 Exerc cio. Prove que existem retas concorrentes. 2.9 Teorema. Duas retas distintas ou no se interceptam, ou se interceptam somente a num ponto. Prova: Dadas duas retas distintas, suponhamos que elas se interceptam em dois (ou mais) pontos. Se isso um fato, essas retas sero coincidentes devido ao axioma 1, o que e a uma contradio pois, por hiptese, elas so distintas. Assim a interseo dessas retas e ca o a ca ou vazia ou s contm um ponto. e o e 2.10 Denio. Pontos pertencentes a uma mesma reta so ditos colineares, caso ca a contrrio so no-colineares. a a a 2.11 Observao. E comum darmos uma representao grca aos entes primitivos da ca ca a geometria euclidiana, imaginando um plano como a superf de uma mesa que se estende cie indenidamente em todas as direes. Nela, a marca da ponta de um lpis representa co a um ponto e a parte de uma reta obtida usando-se uma rgua. Cabe esclarecer que, ao e e estudarmos geometria, hbito utilizarmos desenhos. (vide apndice A). e a e

14

Axiomas da ordem Por exemplo, na Figura 2.1, esto rea b presentadas duas retas, a e b, e trs pontos e A A, B e P . O ponto A pertence a reta a e B, a reta b; o ponto P o ponto de interseo e ca das duas retas. P Muitas vezes faremos uso desse arB tif cio, mas chamamos a ateno do leitor ca a a para o fato de que o desenho s um inse o trumento de apoio ` intuio e ` linguagem a ca a Figura 2.1: Retas a, b e pontos A, B e P. mas, em momento algum, deve ser utilizado como dado para demonstraes. (vide apndice A). co e

. . .

2.2

Axiomas da ordem
...nos dizem sobre as propriedades da disposio mtua de ca u pontos numa reta; para essa disposio de pontos usa-se o ca termo estar entre.

Com os prximos axiomas, estaremos o interessados em caracterizar, exatamente, o que signica dados trs pontos em uma e reta dizer quando um deles se localiza entre os outros dois. Gracamente, isso est a ilustrado na Figura 2.2, para pontos A, B e C de uma reta r.


. .
C

Figura 2.2: O ponto C est entre A e B. a




Axioma 4 Para quaisquer trs pontos distintos colineares, um, e somente um deles, est e a entre os outros dois.

2.12 Observao. As expresses, um ponto C est entre A e B, um ponto C separa os ca o a pontos A e B ou os pontos A e B esto em lados opostos do ponto C, so equivalentes. a a 2.13 Observao. Para dizer que C est entre A e B vamos usar a seguinte notao: ca a ca A C B.


Axioma 5 Se A C B ento estes trs pontos so distintos, colineares e B C A. a e a




15

Axiomas da ordem 2.14 Observao. Em notao de teoria dos conjuntos o axioma 5 pode ser escrito do ca ca seguinte modo: A = B, A = C, B = C; r : A, B, C r e AC B B C A. 2.15 Observao. Se C separa A e B, podemos dizer tambm que B e C esto situados ca e a do mesmo lado em relao a A. ca 2.16 Denio. Dados dois pontos A e B, distintos, o conjunto de pontos constitu ca do pelos pontos A, B e todos os pontos que esto entre A e B chamado de segmento AB, a e a e (ou segmento de reta AB). Se A e B so coincidentes dizemos que AB o segmento nulo. Os pontos A e B so denominados extremidades ou extremos do segmento AB. a 2.17 Exerc cio. Use a notao da teoria de conjuntos e escreva a denio de segmento ca ca AB. 2.18 Exerc cio. Mostre que AB = BA (observe que esta uma igualdade de conjuntos). e 2.19 Denio. A todo subconjunto prprio do plano chamaremos de gura geomtrica, ca o e gura plana ou simplesmente gura. 2.20 Exerc cio. Utilizando somente os axiomas 1 - 5, d exemplos de guras planas. e Utilizando-se segmentos podemos construir muitas guras geomtricas no plano. Uma e das mais simples aquela formada por trs pontos no colineares e pelos segmentos e e a denidos por esses trs pontos. (Figura 2.3) e A 2.21 Denio. Sejam A, B, C trs pontos noca e a colineares. Tringulo determinado por A, B, C a e a o conjunto ABACBC. Os pontos A, B, C so os vrtices do tringulo e os segmentos AB, AC, BC e a so os lados do tringulo. a a

2.22 Observao. Indicaremos o tringulo deter- Figura 2.3: Representao grca do ca a ca a minado pelos pontos A, B, C por ABC. tringulo determinado pelos pontos a A, B e C. 2.23 Exerc cio. Prove que existe pelo menos um tringulo (sugesto: use os axiomas e a a denies dados). co


Axioma 6 Dados dois pontos distintos A e B, existe um ponto C entre A e B.




16

Axiomas da ordem 2.24 Exerc cio. Mostre que existem pelo menos sete tringulos. a
 

Axioma 7 Dados dois pontos distintos A e B, existe um ponto D tal que B est entre a A e D e um ponto E tal que A est entre E e B. a
 

A partir da noo de segmento de reta vamos introduzir o conceito de semi-reta atravs ca e da denio seguinte. ca 2.25 Denio. Dados dois pontos distintos, A e B, semi-reta de origem A contendo ca o ponto B o conjunto de pontos do segmento AB unido com todos os pontos C tais e que B est entre A e C. a 2.26 Observao. Usaremos a notao AB para representar a semi-reta de origem A ca ca contendo o ponto B. 2.27 Exerc cio. Mostre que existe um ponto na reta AB que no pertence ` AB. a a 2.28 Exerc cio. Qual o conjunto de pontos denido por AB? 2.29 Exerc cio. Mostre que AB = BA. 2.30 Denio. Dada a semi-reta AB tomamos um ponto C tal que A esteja entre B e ca C. Chamamos a semi-reta AC de semi-reta oposta a AB. 2.31 Observao. O conceito de semi-reta nos diz que um ponto de uma reta divide ca essa reta em duas semi-retas de mesma origem. 2.32 Exerc cio. Mostre que AB = AB BA. 2.33 Denio. Dizemos que dois pontos distintos, P e Q, esto em um mesmo lado ca a em relao a uma reta r se a interseo do segmento P Q com r for vazia. ca ca Caso contrrio, dizemos que os pontos esto a a em lados opostos em relao a r. ca

. .

.Q

2.34 Denio. Sejam P um ponto e r uma ca r reta que no contm P ; semiplano determia e nado por r contendo P o conjunto de todos e os pontos Q tais que P e Q esto em um mesmo a Q' lado em relao ` reta r. ca a A reta r chamada de origem do semie Figura 2.4: Lados em relao a reta r. ca plano. 17

Axiomas da ordem O semiplano caracterizado pela reta r e pelo ponto P ser representado por rP . a
 

Axioma 8 (Separao do Plano) Uma reta determina somente dois semiplanos disca tintos (ou opostos).
 

O teorema seguinte arma a existncia de uma innidade (neste texto, os termos uma e innidade e innito signicam tanto quanto quisermos) de pontos num segmento de reta AB. 2.35 Teorema. Se A e B so pontos no pertencentes ` uma reta r, onde A pertence a a a a um semiplano e B ao outro daqueles determinados por r, ento a interseo do segmento a ca AB com r no vazia. (Sugesto: prove por reduo ao absurdo). e a a ca 2.36 Teorema. (de Pash) Se ABC um tringulo e r uma reta que intercepta o lado e a e AB num ponto entre A e B, ento r tambm intercepta um dos outros dois lados. a e Prova: C pertence ` r ou no. Caso armativo, segue a validade da concluso; isto a a a , C intercepta ambos os lados BC e AC. Consideremos o caso em que C no pertence e a a ` r (vide gura 2.5). Como A e B no pertencem ` r e AB interceptam r, segue-se por a a denio que A e B esto em lados opostos em relao ` r. Se C no pertence ` r, ou C ca a ca a a a est do mesmo lado de A em relao ` r ou no lado oposto a este (Axioma da Separao). a ca a ca Se C e A esto do mesmo lado em relao a ca A r a ` r, ento C e B esto em lados opostos em a a relao ` r. Isto signica que r intercepta ca a a BC e no intercepta AC. Se C e B esto do a mesmo lado em relao ` r, de modo anlogo ca a a a ` concluso anterior, temos que r intercepta a B C AC, mas no intercepta BC. Assim, em cona cluso ` todas as hipteses para r, chegamos a a o que r intercepta um dos outros dois lados do Figura 2.5: r intercepta outro lado do tringulo que no AB. a a tringulo. a 2.37 Lema. Se A B C e B C D, ento B e C pertencem ao segmento AD. a Prova: Seja E um ponto que no pertence a retaAB. Seja F um ponto da reta CE, a tal que C E F . Sendo AEC um tringulo e A B C, temos que BF intercepta o a segmento AE ou a retaCE. Como C E F , F no pode estar entre C e E. Dos dois a fatos anteriores, segue-se que a reta BF tem que interceptar o segmento AE (Teorema de Pash). Considerando-se agora o tringulo BF C e a reta AE, temos novamente do a Teorema de Pash que o ponto de interseo das retas AE e BF est entre os pontos B e ca a F . Chamemos de G esse ponto de interseo. Analogamente, prova-se que CF intercepta ca o segmento GD em algum ponto H. 18

Axiomas da ordem Como H deve estar no segmento GD, e E no pertence ao segmento AG, ento a a a retaEH ter um ponto em comum com o a segmento AD (Teorema de Pash aplicado ao AGH). Assim, C est no segmento AD. Da a mesma forma, pode-se facilmente mostrar que cio!) B tambm pertence ` AD. (Exerc e a

.E G. .H . B C . . A

.D

Figura 2.6: B e C esto contidos em AD. a 2.38 Lema. Se A C D e A B C, ento A B D e B C D. a Prova: Seja G um ponto no pertencente a reta AB e F um ponto tal que B G F . a A reta CF no tem ponto em comum com a F AB, nem com BG. Assim, CF no tem ponto a em comum com AG. Como A C D e AGD um trinngulo, temos do Teorema de Pash e a que CF intercepta GD em algum ponto H (vide gura 2.7). Pelo mesmo modo, sendo BGD um G H tringulo, segue-se que F H intercepta BD. Vea mos assim, que B C D, e da conclu mos a veracidade da segunda armao do lema. Das ca hipteses, e, da armao e do lema anterior, A o ca B C D segue-se que A B D. Figura 2.7: A B D e b C D.

. . . .

2.39 Teorema. Entre dois pontos distintos , existe uma innidade de pontos. Prova: Seja r uma reta e a e B dois pontos distintos desta reta. Entre A e B existe um ponto C, tal que A C B. Do mesmo modo, existe D tal que A D C. Assim, do lema anterior, segue-se que A D B, e consequentemente A, B, C, D so pontos a distintos de r. De maneira anloga, pode-se armar que existe um ponto E em r, tal a que A E C e A E B, de forma que os pontos A, B, C, D, E so distintos e a ` pertencentes A r. Continuando este mesmo racioc nio, podemos obter entre A e B um conjunto innito de pontos C, D, E, ... Isto prova o teorema. 2.40 Observao. O teorema acima nos diz que qualquer segmento de reta um conca e junto innito de pontos. 2.41 Corolrio. (a) Em uma semi-reta existe um conjunto innito de pontos. (que no a a aquele de um segmento que tenha a origem como extremidade). (b) Em uma reta existe um conjunto innito de pontos. (que no aquele de um segmento a particular). 19

Exerc cios complementares 2.42 Observao. A representao usual para reta, semi-reta e segmento, que estamos ca ca habituados a fazer, no se justica pelos axiomas e teoremas at aqui apresentados. A a e idia de cont e nuo de pontos s ser introduzida na geometria a partir dos axiomas sobre o a medidas (prximo cap o tulo). 2.43 Exerc cio. Mostre que: a) Se A C B, ento todos os pontos de AC pertencem a AB. (Sugesto: use o a a lema 2.38) b) Se AC B, ento nenhum ponto interior de AC pode ser ponto de CB. (Sugesto: a a Use reduo ao absurdo, o lema 2.38 e o axioma 6) ca c) Se A C B, ento cada ponto de AB, diferente de C, pertence ou a AC ou a CB. a (Sugesto: Suponha M pertencente ` AB e M = C. Agora, considere M no pertencente a a a ca a ` AC, nem a CB. Procure argumentar usando o lema 2.37 e a segunda armao do lema 2.38). 2.44 Denio. Um subconjunto do plano convexo se o segmento que liga quaisquer ca e dois de seus pontos est totalmente contido nele. Em s a mbolos: F convexo se, e e somente se, A, B F, AB F. 2.45 Exerc cio. Quanto a subconjuntos convexos, verique: a) A interseo de subconjuntos convexos um subconjunto convexo. ca e b) Um semiplano um subconjunto convexo. e c) A interseo de n semiplanos um subconjunto convexo. ca e d) Unio de subconjuntos convexos um subconjunto convexo? a e e) Um tringulo uma gura convexa? a e f) Um tringulo determina dois conjuntos no plano, um dos quais convexo. O a e conjunto convexo assim obtido, menos os pontos do tringulo , por denio, o interior a e ca do tringulo. a

2.3

Exerc cios complementares

2.46 Exerc cio. Considere o conjunto = {A1 , A2 , ..., An }, n 2 e os subconjuntos binrios a


rij = {Ai , Aj }, i, j = 1, 2, ..., n, i = j. Chamemos Ai de pontos, rij de retas e de plano. a) Verique que, nesta geometria, vale o axioma 1. b) Mostre que, nesta geometria, vale o teorema 2.9.

2.47 Exerc cio. Considere um conjunto C de n (n 3) pontos de um plano que tem um


subconjunto C1 formado por p (2 p n) pontos colineares. Sabe-se que toda vez que 3 pontos de C so colineares eles so pontos de C1 . Calcule o nmero de retas determinadas por esses n a a u pontos (Sugesto: considere o conjunto formado por todos os pontos 3 a 3 colineares). a

2.48 Exerc cio. Mostre que imposs construir um modelo de uma geometria com 6, e vel
pontos, em que sejam vlidos os axiomas 1 e 2 e em que todas as retas tenham exatamente 3 a pontos.

20

Exerc cios complementares


(sugesto: considere as vrias possibilidades de C pertencer a AB). a a

2.49 Exerc cio. Mostre que, se C AB e C = A, ento AB = AC, BC AB e A BC a /

2.50 Exerc cio. Considere as armaes: co


i) Existem dois segmentos distintos que tm dois pontos em comum. e ii) Existem dois segmentos distintos que tm somente dois pontos em comum. e Diga quais dessas armaes so verdadeiras ou falsas. Justique sua resposta. co a

21

Exerc cios complementares

22

Cap tulo 3 Axiomas Sobre Medida de Segmentos e de Angulos


Baseados nos axiomas de Incidncia e da Ordem, deniu-se e segmento e semi-reta. O conceito de medida de um segmento, ou a determinao de seu comprimento, que inca e troduzido, usualmente, mediante ` adoo de uma unidade a ca de comprimento, aqui estabelecido atravs de axiomas. e e Introduz-se tambm o conceito de ngulo e sua medida e a

3.1

Axiomas sobre medida de segmentos


...dizem como medir segmentos; do uma maneira de coma parar segmentos.

Com os axiomas de incidncia e de ordem, vistos nas sees anteriores, pudemos denir e co segmento de reta. Em geral, o conceito de medida de um segmento, ou a determinao ca de seu comprimento, introduzido mediante a adoo de uma unidade de comprimento. e ca Neste curso faremos axiomaticamente a introduo de tal medida, relacionando-a com a ca prtica usual de adoo de uma unidade. A partir do momento em que soubermos como a ca medir segmentos estaremos aptos ` poder compar-los em tamanho e, da dizer se um a a , segmento maior, menor ou igual, em comprimento, a um outro. e
'

Axioma 9 Existe uma funo d que a cada par (A, B) de pontos do plano, associa um ca nmero real d (A, B) tal que: u (i) d (A, B) 0, A, B, e d (A, B) = 0 A = B. (ii) d (A, B) = d (B, A) , A, B.
&

3.1 Denio. O nmero d (A, B) chamado distncia entre os pontos A e B, ou ca u e a medida do segmento AB, ou ainda comprimento do segmento AB. 23

Axiomas sobre medida de segmentos 3.2 Observao. Quase sempre indicaremos d (A, B) , escrevendo em seu lugar AB. ca O axioma seguinte estabelece uma correspondncia entre nmeros reais e pontos de e u uma reta.
 

Axioma 10 (Existncia de Sistema de Coordenadas) Para cada reta r do plano e existe um bijeo x : r R que, a cada ponto P r, associa um nmero real x (P ) tal ca u que para quaisquer dois pontos A, B r tem-se |x (A) x (B)| = d (A, B) .
 

A bijeo x do axioma 10 chamada sistema de coordenadas para r e o nmero ca e u real x (P ) , associado ao ponto P, ser chamado de coordenada de P em relao a r. O a ca ponto de r associado ao nmero 0 chamado de origem do sistema de coordenadas. u e A origem de um sistema de coordenadas divide a reta r em duas semi-retas opostas. A semi-reta que contm o ponto associado ao nmero 1 chamaremos de parte positiva e u de r e, a outra, parte negativa. O axioma seguinte estabelece que a medida de segmentos goza da propriedade de aditividade.


Axioma 11 Se A C B ento AC + CB = AB. a




3.3 Observao. Agora que sabemos determinar a medida de um segmento a partir das ca coordenadas de suas extremidades, e lembrando que os nmeros reais so ordenados pela u a relao menor do que (ou pela relao maior do que), vamos poder estabelecer uma ca ca relao de ordem para os pontos de uma reta. ca Se a < c < b (ou b < c < a ), dizemos que o nmero c est entre a e b. u a Como prembulo ao teorema 3.5, demonstraremos o seguinte lema: a 3.4 Lema. Se um ponto C, distinto de dois pontos A e B, pertence a AB, e AC tal e que AC < AB ento C est entre A e B. a a Prova: Desde que A a origem da semi-reta S(AB) e B S(AB) , A no pode estar e a entre B e C. Se B estivesse entre A e C ter amos (axioma 11) AB + BC = AC e, da , AB < AC, contrrio a hiptese AC < AB. Pelo axioma 3 temos ento A C B. a o a A correspondncia entre a relao de ordem de nmeros reais e a relao de ordem de e ca u ca pontos de uma reta ca estabelecida no seguinte teorema: 3.5 Teorema. Sejam x(A), x(B), x(C) as respectivas coordenadas dos pontos A, B, C de uma mesma reta. O ponto C est entre A e B se, e somente se, o nmero x(C) est entre a u a x(A) e x(B). 24

Axiomas sobre medida de segmentos Prova: (Necessidade) Suponhamos que A C B. Decorre do Axioma 11 que AC + CB = AB, isto : e |x(C) x(A)| + |x(B) x(C)| = |x(B) x(A)| . Inicialmente, vamos supor que x(B) > x(A). Neste caso, temos |x(B) x(A)| = x(B) x(A) e, da igualdade anterior, segue que |x(C) x(A)| < x(B) x(A) e |x(B) x(C)| < x(B) x(A). Assim: x(B) + x(A) < x(C) x(A) < x(B) x(A) e x(B) + x(A) < x(B) x(C) < x(B) x(A). Logo x(C) < x(B) e x(A) < x(C). Assim, x(A) < x(C) < x(B), isto , x(C) est entre x(A) e x(B). e a Se x(B) < x(A), um racioc nio anlogo nos leva ` mesma concluso. Deste modo a a a temos provada a condio necessria. ca a (Sucincia) Suponhamos que o nmero x(C) est entre x(A) e x(B), isto , x(A) < e u a e x(C) < x(B). Temos ento: a |x(C) x(A)| + |x(B) x(C)| = |x(B) x(A)| , e, em particular, AC < AB e CB < AB. Consideremos as semi-retas com origem em A; se B e C pertencem a uma mesma dessas semi-retas e sendo AC < AB, do lema 3.4, segue que C est entre A e B. a Resta mostrar que B e C no podem pertencer a a semi-retas distintas, isto , o ponto A no pode separar e a B e C. De fato, se isto acontecer ento B A C a 0 1 2 3 e da BA + AC = BC, resultando BA < BC, o que contradiz a desigualdade CB < AB, j obtida acima. a 1 0 2 3 Logo, A C B. O teorema acima justica que a disposio dos Figura 3.1: Ordem na repreca nmeros ao longo de uma reta, deve ser como no di- sentao dos nmeros como ponu ca u agrama de cima da Figura (3.4), e no como no dia- tos de uma reta. a grama que aparece embaixo.

. . . . . . . .

3.6 Exerc cio. Complete a demonstrao do teorema 3.5 considerando o caso x (B) < ca x (A) . 3.7 Exerc cio. D uma caracterizao para segmento e semi-reta, usando as coordenadas e ca dos seus pontos. 25

Axiomas sobre medida de segmentos 3.8 Teorema. Em qualquer segmento AB existe um unico ponto C tal que AC = CB. Prova: (Existncia) Dado o segmento AB, aos pontos A e B esto associadas suas e a coordenadas x(A) e x(B) (axioma 10). Consideremos o nmero real 1 (x(A) + x(B)) . u 2 De acordo com o axioma 10, existe um ponto C da reta determinada por A e B tal que x(C) = 1 (x(A) + x(B)). Como 2 1 AC = |x(A) x(C)| = x(A) (x(A) + x(B)) 2 x(A) x(B) = 2 2 e CB = |x(C) x(B)| = = x(A) x(B) , 2 2 1 (x(A) + x(B)) x(B) 2

1 conclu mos que AC = CB. Como o nmero 2 (x(A) + x(B)) est entre x(A) e x(B) u a (mostre isto), pelo teorema 3.5 segue que o ponto C est entre A e B. Mostramos, assim, a que existe um ponto que satisfaz as exigncias do teorema. e (Unicidade) Suponhamos que, alm do ponto C denido acima, exista um outro ponto, e C , no segmento AB, tal que AC = C B e seja x (C ) a coordenada desse ponto. Ento a |x(C ) x(A)| = |x(B) x(C )| e da

x(C ) x(A) = x(B) x(C ) ou x(C ) x(A) = x(C ) x(B). A segunda possibilidade no pode ocorrer pois implicaria em A = B, o que um a e absurdo. Logo 1 x(C ) = (x(A) + x(B)) . 2 Assim, x(C) = x(C ) e, pelo axioma 10, conclu mos que C = C . Isto prova a unicidade. 3.9 Denio. O ponto C do segmento AB, tal que AC = CB, chamado ponto ca e mdio desse segmento. e 3.10 Observao. O teorema 3.8 mostra que todo segmento tem um ponto mdio e este ca e unico. e 3.11 Observao. A distncia entre pontos satisfaz as seguintes propriedades: ca a i) AB 0 para quaisquer pontos A e B; ii) AB = 0 A = B; iii) AB = BA para quaisquer pontos A e B. 26

Axiomas sobre medida de segmentos Observe que a noo de distncia satisfaz propriedades que so as intuitivamente esperca a a adas. Uma outra propriedade, que se espera estar satisfeita, a chamada desigualdade e triangular, dada por: iv) AC AB + BC para quaisquer trs pontos A, B e C no plano. A igualdade vale e se C AB. Como veremos mais adiante (teorema 5.19), esta propriedade ser uma conseqncia a ue dos axiomas anteriores e do prximo grupo de axiomas a ser apresentado no cap o tulo 4. Com a noo de distncia entre pontos podemos denir circunferncia. Seja A um ca a e ponto do plano e R um nmero real positivo. u 3.12 Denio. Circunferncia de centro A e raio R o conjunto de pontos B do ca e e plano tais que AB=R. Notao: C(A,R) ca Um ponto C do plano tal que AC < R e dito ponto interior ` C(A,R); neste caso, dizemos a tambm que C est dentro da circunferncia. Se e a e A AC > R, o ponto C dito exterior ` C(A,R); neste e a caso dizemos tambm que C est fora da circune a ferncia. O conjunto de pontos interiores ` circune a ferncia chamado disco aberto ou bola aberta de Figura 3.2: Circunferncia de e e e centro A e raio r; que ser indicado por B(A,R). a centro em A. 3.13 Exerc cio. Use notao da teoria de conjuntos para escrever, de modo sinttico, os ca e conceitos da denio acima. ca 3.14 Denio. Um subconjunto do plano dito limitado se existe um disco aberto ca e que o contm; caso contrrio, dito ilimitado (ou no limitado). e a e a 3.15 Exerc cio. Utilizando as denices anteriores: o i) Prove que qualquer conjunto nito de pontos do plano limitado. e ii) Prove que todo segmento limitado. e iii) Prove que a unio nita de subconjuntos limitados ainda um conjunto limitado a e (observao: admita como verdadeira a desigualdade triangular). ca iv) Prove que todo tringulo limitado. a e v) Prove que dados um subconjunto limitado M , no plano, e um ponto P desse plano, existe um disco aberto com centro em P e que contm M . e vi) Prove que retas so conjuntos ilimitados. a 3.16 Exerc cio. Dizemos que o ponto C de um dado segmento AB a seo urea de AB e ca a
AC se C tal que CB = AB . Neste caso, dizemos tambm que C divide AB em mdia e extrema e e e AC razes. o Mostre que se C seo urea de AB, ento AC = 51 AB e que AB = 5+1 AC (o nmero e ca a a u 2 2 5+1 2

conhecido como n mero ureo). Mostre que todo segmento possui uma seo urea. e u a ca a

27

Angulo e Axiomas sobre medida de ngulos a

3.2

Angulo e Axiomas sobre medida de ngulos a


... dene-se ngulo e diz como medi-los; d-se uma maneira a a de comparar ngulos. a

e a 3.17 Denio. Angulo uma gura formada pela unio de duas semi-retas distintas ca que possuem a mesma origem e no esto contidas na mesma reta. a a Angulo raso a unio de duas semi-retas opostas. Semi-retas coincidentes denem e a um ngulo nulo. a As semi-retas so chamadas lados do ngulo e a origem comum o vrtice do ngulo. a a e e a 3.18 Observao. Muitos so os modos para se representar um ngulo; assim, se O o ca a a e vrtice e se A e B so pontos quaisquer distintos de O, um em cada lado do ngulo, este e a a a ngulo pode ser denotado por AOB ou B OA, (Figura 3.3). Nesta notao, a letra que denomina ca o vrtice deve sempre aparecer entre as e A outras duas. Caso nenhum outro ngulo a tenha o mesmo vrtice, podemos utilizar e s a letra que designa o vrtice para repreo e O sentar o ngulo. Assim, o ngulo represena a B tado na Figura 3.3 poderia ser denominado Figura 3.3: Angulo AOB ou O e ngulo . simplesmente por O. Em vrias ocasies a a o e comum utilizar-se letras do alfabeto grego para representar um ngulo; neste caso, escreve-se, prximo do vrtice e entre as duas a o e semi-retas, a letra que designa o ngulo (Figura 3.3). a

3.19 Observao. Dado um tringulo ABC, os ngulos ABC, B AC e ACB so ditos ca a a a os ngulos internos do tringulo ABC. a a De modo anlogo `quele utilizado para se introduzir medida de segmento, a medida a a de um ngulo feita atravs de alguns axiomas apropriados. a e e


Axioma 12 Existe uma funo que a cada ngulo associa um nmero real positivo. ca a u


3.20 Denio. O nmero associado a um ngulo, a que se refere o axioma 12, ca u a e chamado medida do ngulo. a 3.21 Observao. Via de regras, representaremos um ngulo e sua medida pelo mesmo ca a s mbolo. Assim, os s mbolos AOB e sero usados tanto para denotar um ngulo quanto a a a sua medida. 28

Angulo e Axiomas sobre medida de ngulos a Sabemos que uma reta divide um plano em dois semiplanos (axioma 10). Tomemos um ponto O pertencente a uma reta r e uma semi-reta OP , P contido no semiplano H determinado por r. Nessa situao dizeca mos que a semi-reta divide o semiplano H. Fixemos ento um nmero real a u positivo K.
#

Figura 3.4: Semiplano H determinado por r.

Axioma 13 Existe uma bijeo que a cada semi-reta OP que divide H associa um ca nmero real p ]0, K[. Sendo a e b, respectivamente, os nmeros associados as semi-retas u u OA e OB que dividem H, a medida do ngulo AOB o nmero |a b|. a e u
" 

! 

Axioma 14 A uma das semi-retas de r com origem O est associado o nmero 0 e ` a u a semi-reta oposta a ela est associado o nmero K. A medida do ngulo nulo zero e a a u a e medida do ngulo raso K. a e


3.22 Observao. No caso em que K = 180 temos o modo usual de medir ngulo, ca a que aquele onde usamos um transferidor. A medida de ngulos feita nesse sistema e a de coordenadas d o resultado em graus. Falaremos em ngulo de x graus ou x onde a a 0 x 180. Quando K = 200 a medida em grado e quando K = a medida est e a dada em radiano . Daqui por diante, salvo meno em contrrio, vamos assumir que os ca a a ngulos so medidos em graus. a 3.23 Observao. Exibida uma correspondncia caracterizada pelos Ax. 13 e 14, a cada ca e semi-reta ca associado um nmero real entre 0 e K chamado coordenada da semi-reta. u 3.24 Denio. Consideremos as semi-retas OA, OB e OC. Se o segmento AB interca cepta OC, dizemos que OC divide o ngulo AOB. a 3.25 Exerc cio. Mostre que se OC divide o ngulo AOB independente a escolha dos a e pontos A e B nas semi-retas OA e OB, respectivamente. O prximo axioma vai estabelecer a propriedade da aditividade, coisa que se espera o de uma medida de ngulo. Para fazer isso, vamos considerar trs semi-retas de mesma a e origem OA, OB e OC, tais que as duas ultimas estejam contidas, propriamente, num dos semiplanos determinados pela reta que contm OA. e 29

Angulo e Axiomas sobre medida de ngulos a


 

Axioma 15 Se uma semi-reta OC divide um ngulo AOB, ento AOB = AOC + C OB, a a e reciprocamente.
 

3.26 Exerc cio. Se a, b so as coordenadas dos lados do ngulo AOB, ento a medida a a a de AOB dada por |a b|. (Resulta diretamente do Ax. 13). e 3.27 Teorema. Para qualquer ngulo AOB, existe uma unica semi-reta OC que divide a AOB, tal que AOC = C OB. Prova: Vide exerc 3.53. cio 3.28 Denio. A semi-reta OC do teorema acima, chamada bissetriz do ngulo ca e a AOB. 3.29 Observao. O teorema 3.27 mostra que todo ngulo tem uma bissetriz e ela ca a e unica. Vamos introduzir, baseados nos axiomas at agora xados, algumas denies que e co estabelecero terminologias uteis. a 3.30 Denio. Dois ngulos so consecutivos se eles tm um lado em comum. Se os ca a a e outros lados dos ngulos esto em semiplanos opostos, denidos pelo lado comum, esses a a a ngulos so ditos adjacentes. (A Figura 3.5 ilustra a diferena entre ngulos consecutivos a c a e adjacentes) Prolongar um lado de um ngulo signica a tomar a semi-reta oposta a esse lado. A semiA reta assim obtida dita ser o prolongamento e C do lado do ngulo. Dados dois ngulos, se a a a soma de suas medidas 180 eles so ditos sue a plementares. O B Se a soma das medidas de dois ngulos 90 a e eles so ditos complementares. Dizemos que a Figura 3.5: Os ngulos AOB e B OC a a e a so consecutivos mas no so adja- um ngulo complemento de outro ngulo se a a a eles forem adjacentes e complementares. centes.

3.31 Denio. Um suplemento de um ngulo um ngulo adjacente a esse ngulo, ca a e a a obtido pelo prolongamento de um de seus lados. 3.32 Exerc cio. Mostre que um ngulo e seu suplemento so ngulos suplementares. a a a 3.33 Exerc cio. Mostre que se dois ngulos tm a mesma medida, o mesmo ocorre com a e os seus suplementos. 30

Angulo e Axiomas sobre medida de ngulos a 3.34 Denio. Angulos opostos pelo vrtice so aqueles que os lados de um so as ca e a a respectivas semi-retas opostas aos lados do outro. 3.35 Teorema. Angulos opostos pelo vrtice tm a mesma medida. e e 3.36 Exerc cio. Prove o teorema anterior. 3.37 Denio. Angulo reto um ngulo cuja medida 90 . ca e a e 3.38 Denio. Se um ngulo mede menos que 90 ele dito agudo, e dito obtuso ca a e e se mede mais de 90 . 3.39 Exerc cio. Mostre que o suplemento de um ngulo reto tambm um ngulo reto. a e e a 3.40 Teorema. Se duas retas so concorrentes e denem quatro ngulos tais que um a a deles reto, ento os outros trs tambm so retos. e a e e a 3.41 Exerc cio. Prove o teorema anterior. O teorema 3.40 motiva a seguinte denio: ca 3.42 Denio. Duas retas concorrentes so perpendiculares se um dos quatro ngulos ca a a que elas denem reto. e 3.43 Denio. Duas semi-retas de mesma origem so perpendiculares se cada uma ca a delas est contida em retas perpendiculares. a O teorema seguinte d uma caracterizao de retas perpendiculares. a ca 3.44 Teorema. Duas retas concorrentes so perpendiculares se, e somente se, formam a a ngulos adjacentes suplementares de mesma medida. 3.45 Exerc cio. Prove o teorema anterior. Denimos quando duas retas so perpendiculares, mas, at agora, no exibimos uma a e a situao mostrando que, de fato, nossa denio no vazia de signicado, isto , ainda ca ca a e e no mostramos que existem retas perpendiculares (falar de elefante cor-de-rosa no implica a a que exista elefante com essa cor). s Construo de retas perpendiculares. ca Consideremos uma reta r e tomemos em r um ponto A (...podemos?). Esse ponto determina em r duas semi-retas que denem um ngulo raso. Alm a e disso, a reta r determina dois semiplanos. Consider A remos um desses semiplanos. Dentre todas as semiretas com origem em A e contidas nesse semiplano, a existe uma cuja coordenada o nmero 90. Esta semi- Figura 3.6: As retas r e s so pere u reta forma com as semi-retas determinadas em r, pelo pendiculares.

31

Exerc cios complementares ponto A, dois ngulos com medidas iguais a 90 (por que?). Logo, a semi-reta assim cona stru est contida numa reta s que contm o ponto A e perpendicular ` reta r dada. da a e e a Assim, r e s so perpendiculares. Os argumentos acima provam que retas perpendiculares a existem. Unicidade de retas perpendiculares passando por um ponto. Com um pouco mais de trabalho, podemos mostrar que a reta s passando por A e perpendicular ` r unica. a e De fato, suponhamos que s e s so duas retas perpendiculares a r passando por A. As a intersees de s e s com um dos semiplanos so semi-retas, cujas coordenadas chamaremos co a de c e c , respectivamente. Pelo axioma 13 segue que as semi-retas determinadas por A em r, tm coordenadas e iguais a 0 e 180. Como r, s so perpendiculares e a medida do ngulo entre elas dada por a a e |180 c| ou por |0 c| , segue que |180 c| = 90 ou |0 c| = 90. Analisemos o caso em que |180 c| = 90. Como r, s so perpendiculares e a medida do ngulo entre elas dada a a e por |180 c | temos tambm que |180 c | = 90, logo, |180 c| = |180 c | . Observe que e os nmeros c e c pertencem ao intervalo (0, 180) , o que nos permite reescrever a ultima u igualdade como 180 c = 180 c e, ento, c = c . a Como a correspondncia entre semi-retas que dividem um dado semiplano e nmeros e u do intervalo (0, 180) uma funo injetora, as semi-retas consideradas so coincidentes e, e ca a da s e s coincidem. , Os resultados obtidos acima podem ser reunidos no seguinte teorema: 3.46 Teorema. Dada uma reta, por qualquer um de seus pontos passa uma unica reta perpendicular a ela. 3.47 Denio. Dado um segmento AB, a reta perpendicular a AB passando pelo seu ca ponto mdio chamada mediatriz do segmento AB. e e

3.3

Exerc cios complementares

3.48 Exerc cio. (Generalizao do teorema 3.8) Em qualquer segmento AB, mostre que existe ca
um unico ponto C entre A e B tal que AC = kAB, onde k qualquer nmero real, 0 < k < 1. e u

3.49 Exerc cio. Mostre que se A e B so dois pontos distintos e k > 0 um nmero real, a e u
existe um unico ponto P AB tal que AP = kAB.

3.50 Exerc cio. a) Prove que dois ngulos que tm o mesmo complemento possuem a mesma a e
medida. b) Sabe-se que dois ngulos so complementares. Calcule a medida desses ngulos sabendo a a a que o suplemento do maior igual a 7 vezes o menor. e

3.51 Exerc cio. Prove que se um ngulo e seu suplemento tm a mesma medida ento ele a e a e
um ngulo reto. a

32

Exerc cios complementares 3.52 Exerc cio. Mostre que:


a) O suplemento de um ngulo obtuso sempre agudo. a e b) O complemento de um ngulo agudo sempre agudo. O suplemento de um ngulo agudo a e a sempre obtuso? e

3.53 Exerc cio. Prove o teorema 3.27 (sugesto: faa uma demonstrao anloga ` do teoa c ca a a
rema 3.8).

3.54 Exerc cio. Prove que as bissetrizes de dois ngulos opostos pelo vrtice so semi-retas a e a
opostas.

3.55 Exerc cio. Prove que a bissetriz de um ngulo e a de um de seus suplementos so semia a
retas perpendiculares entre si.

3.56 Exerc cio. Prove que se as bissetrizes de dois ngulos adjacentes so perpendiculares, a a
ento os ngulos so suplementares. a a a

3.57 Exerc cio. Prove que as retas perpendiculares ` bissetriz, com origem no vrtice de um a e
a ngulo, formam ngulos de iguais medidas com os lados do ngulo dado. a a

3.58 Exerc cio. Considere o ngulo AOB (no nulo e no raso). Seja OA,B o semiplano a a a
determinado pela reta OA, que contm o ponto B, e OB,A o semiplano determinado pela reta e OB e que contm o ponto A. Mostre que o conjunto OA,B OB,A convexo. O conjunto e e OA,B OB,A AOB chamado interior do ngulo AOB e ser indicado por intAOB, e e a a cada um de seus pontos chamado ponto interior do ngulo AOB. e a AOB = C OD, OB est contida no interior de AOC e OC est contida no interior de B OD. a a Mostre que AOC = B OD.

3.59 Exerc cio. Considere quatro semi-retas, OA, OB, OC e OD, de mesma origem, tais que

existe uma unica semi-reta OC tal que C OB = k AOB.

3.60 Exerc cio. Dado um ngulo AOB e um nmero real positivo k, 0 < k < 1, mostre que a u

3.61 Exerc cio. Se traarmos n semi-retas de mesma origem, OAi , i = 1, 2, ..., n, n 2, c


quantos ngulos elas determinam? a

3.62 Exerc cio. Pergunta-se: a que horas o ponteiro das horas de um relgio ser a bissetriz o a
do ngulo formado pelo ponteiro dos minutos e a posio dos mesmos, ao meio-dia. a ca

33

Exerc cios complementares

34

Cap tulo 4 Axiomas de Congruncia e


A partir das noes de medida de segmentos e de ngulos co a so introduzidos os conceitos de congruncia de segmena e tos, ngulos e tringulos. So apresentados, tambm, teoa a a e remas que do condies sucientes para a congruncia de a co e tringulos. a

4.1

Axiomas sobre congruncia de segmentos, ngulos e a e tringulos a


... denem o conceito de congruncia ou igualdade de e segmentos de retas, tringulos. e ngulos. a a

4.1 Denio. Dois segmentos so congruentes se eles tm a mesma medida. ca a e 4.2 Denio. Dois ngulos so congruentes se eles tm a mesma medida. ca a a e 4.3 Observao. Usamos o termo congruentes, e no iguais, para distinguir do termo ca a igual, que signica, matematicamente, o mesmo objeto matemtico. a Indicamos congruncia entre segmentos (ngulos) AB e CD (A e B) escrevendo AB e a a CD (A B); assim, AB CD AB = CD (A B A = B). E fcil ver que a relao ca satisfaz as seguintes propriedades: (i) (reexiva) AB AB. (ii) (simtrica) Se AB CD, ento CD AB. e a (iii) (transitiva) Se AB CD e CD EF , ento AB EF . a Assim, a relao de congruncia entre segmentos uma relao de equivalncia. ca e e ca e 4.4 Exerc cio. Mostrar que congruncia entre ngulos tambm uma relao de equivalncia. e a e e ca e 4.5 Denio. Dois tringulos ABC e XY Z so congruentes se existe uma aplicao ca a a ca bijetora : {A, B, C} {X, Y, Z} 35

Axiomas sobre congruncia de segmentos, ngulos e tringulos e a a com a seguinte propriedade: se X = (A), Y = (B) e Z = (C) ento a A X, B Y, AB XY , AC XZ, C Z, BC Y Z.

A aplicao chamada de congruncia. ca e e Os vrtices A e X, B e Y , C e Z so ditos correspondentes. Angulos correspone a dentes so aqueles cujos vrtices so correspondentes, e lados correspondentes so os a e a a lados cujas extremidades so vrtices correspondentes. a e Se os tringulos ABC e XY Z so congruentes, escrevemos ABC XY Z, signicando a a que a congruncia leva A em X, B em Y e C em Z. e 4.6 Exerc cio. Mostre que a relao de congruncia entre dois tringulos uma relao ca e a e ca de equivalncia. e O axioma seguinte estabelece a existncia de tringulos congruentes. e a
 

Axioma 16 Sejam ABC um tringulo e r uma semi-reta. Existe um tringulo ABC, a a congruente ao tringulo ABC, tal que A coincide com a origem de r, o vrtice B pertence a e a r e C um ponto contido num dos semiplanos denidos pela reta r que contm r. e e
 

O lema seguinte nos fala sobre a existncia de segmentos congruentes. e 4.7 Lema. (Transporte de segmentos) Sejam AB um segmento e r uma semi-reta. Existe um unico segmento A B contido em r onde A coincide com a origem de r e tal que A B AB. Prova: (Existncia) Tome um ponto C que no pertence ` reta AB (tal ponto e a a existe?) determinando o ABC. Pelo axioma 16, existe um tringulo A B C , congruente a ao tringulo ABC, tal que A coincide com a origem de r , o vrtice B pertence ` r e a e a C um ponto num dos semiplanos denidos pela reta r que contm r . Pela denio de e e ca congruncia de tringulos, A B AB. e a (Unicidade) Suponhamos que, alm do ponto B obtido acima, tal que A B AB, e possamos marcar outro ponto B pertencente a r , B = B , tal que A B AB. Dos trs pontos A , B , B , um deve estar entre os outros dois; ele no pode ser A , pois B e e a B no so separados pela origem da semi-reta. Se B que est entre A e B , ento a a e a a A B = A B + B B , e da B B = 0 o que imposs e vel, pois B = B . Assim, B no a est entre A e B . Concluso anloga obteremos supondo que B est entre A e B . Isto a a a a uma contradio; logo, B = B e, da temos conclu a prova da unicidade. e ca , da O lema seguinte nos fala sobre a existncia de ngulos congruentes. e a 4.8 Lema. (Transporte de ngulos) Sejam AOB um ngulo e r uma semi-reta. a a Existe um unico ngulo A O B , em que o vrtice O coincide com a origem de r , um lado a e coincide com r e A O B AOB. 36

Axiomas sobre congruncia de segmentos, ngulos e tringulos e a a 4.9 Exerc cio. Demonstre o lema 4.8 (sugesto: considere o tringulo AOB e use o axia a oma 16; no esquea que este tambm um resultado que envolve existncia e unicidade). a c e e e
 

Axioma 17 Se dois tringulos ABC e XY Z so tais que AB XY , A X e AC XZ, a a ento ABC XY Z. a

O axioma acima conhecido como e Primeiro Caso de Congruncia de e Tringulos ou simplesmente, caso LAL. a

B
|

Y
|
||

4.10 Observao. Pela denio 4.5, para ca ca vericarmos a congruncia de dois tringulos e a || || A C X Z temos que vericar seis relaes: conco gruncia dos trs pares de lados corree e Figura 4.1: O Caso LAL de congruncia de e spondentes e congruncia dos trs pares e e tringulos. a de angulos correspondentes. No entanto, o axioma 17 arma que suciente vericar apenas trs delas, isto : e e e AB XY AB XY , AC XZ, BC Y Z e AC XZ = A X, B Y , C Z. AX
|

Os resultados que seguem so conseqncias diretas, ou indiretas, do axioma 17. a ue 4.11 Teorema. (Segundo Caso de Congruncia de Tringulos ou caso ALA) e a Dois tringulos ABC e XY Z so congruentes se AB XY , A X e B Y . a a Prova: Sejam ABC e XY Z tringulos tais que AB XY, A X e B Y . Seja a D um ponto da semi-reta AC tal que AD XZ (existe este ponto?). Consideremos o tringulo ABD e comparemo-lo com o tringulo XY Z. Uma vez que AD XZ, AB a a XY e A X, pelo axioma 17 segue que ABD XY Z. Conclu mos da que ABD Y . Logo ABD ABC. Decorre da que as semi-retas BD e BC C Z coincidem. Logo, pelo teorema 2.9, os pontos C e D coincidem, e ento os tringulos a a ABC e ABD tambm coincidem e, pore tanto, ABC ABD. Como ABD | | B X Y XY Z, pela transitividade da relao A ca segue que ABC XY Z. A seguir, denimos alguns conceitos Figura 4.2: O Caso ALA de congruncia de e que sero uteis para xar linguagem sobre tringulos. a a tringulos. a
|

37

||

Axiomas sobre congruncia de segmentos, ngulos e tringulos e a a 4.12 Denio. (a) Um tringulo que tem dois lados congruentes dito issceles. O ca a e o vrtice comum a esses dois lados chamado vrtice do tringulo isceles, os lados cone e e a o gruentes so chamados laterais e o terceiro lado chamado base. Os ngulos adjacentes a e a a ` base so chamados ngulos da base. a a (b) Um tringulo que tem os trs lados congruentes chamado equiltero. a e e a (c) Um tringulo que tem um ngulo reto chamado retngulo. Os lados que a a e a denem o ngulo reto so chamados catetos e o terceiro lado chamado hipotenusa do a a e tringulo. a Seja ABC um tringulo e D um ponto da reta determinada por B e C. a 4.13 Denio. (a) Se D for o ponto mdio do segmento BC, o segmento AD chamado ca e e e mediana do ABC relativamente ao lado BC (ao vrtice A). (b) Se D tal que S(AD) divide o ngulo C AB em dois ngulos congruentes, isto , e a a e C AD DAB, dizemos que AD bissetriz do ngulo A. e a (c) Se D tal que a reta determinada por A e D perpendicular ` reta determinada e e a e e por B e C, dizemos que AD altura do ABC relativamente ao lado BC (ou ao vrtice A). 4.14 Observao. A existncia da altura de um tringulo ser garantida no prximo ca e a a o cap tulo. 4.15 Exerc cio. Mostre que nas denies 4.12 e 4.13, os objetos ali apresentados esto co a bem denidos. 4.16 Teorema. Em qualquer tringulo issceles, os ngulos da base so congruentes. a o a a Prova: Seja ABC um tringulo issceles em que AB AC. Queremos provar que a o B C. Para isso vamos comparar o tringulo ABC com ele mesmo, denindo a seguinte a lei de correspondncia: e : {A, B, C} {A, B, C} tal que (A) = A, (B) = C e (C) = B. Por hiptese AB AC e AC AB. Como A A, segue, pelo axioma 17, que esta o correspondncia dene uma congruncia, isto , ABC ACB. Logo, B C. e e e 4.17 Teorema. Se um tringulo tem dois ngulos congruentes, ento ele issceles. a a a e o 4.18 Exerc cio. Prove o teorema 4.17. 4.19 Exerc cio. Prove que em qualquer tringulo issceles a mediana relativamente ` a o a base tambm altura e bissetriz. e e 4.20 Teorema. (Terceiro Caso de Congruncia de Tringulos ou caso LLL) e a Dois tringulos so congruentes se eles tm os lados correspondentes congruentes. a a e 38

Exerc cios complementares Prova: Sejam ABC e XY Z dois tringulos tais que AB XY , AC XZ e BC a Y Z. Mostremos que ABC XY Z. Consideremos a semi-reta S(AB) e escolhamos o semiplano denido pela reta determinada por A e B e oposto `quele que contm o vrtice a e e C. Construamos um ngulo com vrtice A, congruente ao ngulo X, tendo por lados S(AB) a e a e uma semi-reta r contida no semiplano escolhido acima. A partir de A, marquemos em r um ponto D tal que AD XZ (certique-se de que tudo isso poss de ser constru e vel do!). o Como AB XY (por hiptese), C Z AD XZ (por construo) e ca DAB X (por construo), temos ca que ADB XZY . Consideremos o segmento CD; como AD XZ AC e DB ZY CB, os tringulos a || || A B X Y ADC e DBC so issceles. Segue, a o da que ADB ACB. Pelo axioma , e 17, temos que ADB ACB. Mas Figura 4.3: O Caso LLL de congruncia de tringulos. a hav amos provado que ADB XZY logo ACB XZY ou ABC XY Z.
|
|

||

4.21 Exerc cio. Enuncie os casos de congruncia de tringulos utilizando-se somente de e a palavras. (Sem uso de notaes simblicas). co o

4.2

Exerc cios complementares

4.22 Exerc cio. Estude a possibilidade de construir um tringulo issceles cujos lados medem a o
a e b sendo a > b.

4.23 Exerc cio. Prove que em qualquer tringulo issceles as bissetrizes dos ngulos da base a o a
so congruentes. a

4.24 Exerc cio. Prove que em qualquer tringulo equiltero as trs bissetrizes so congrua a e a
entes.

4.25 Exerc cio. Prove que em qualquer tringulo issceles as medianas relativamente, aos a o
a ngulos da base, so congruentes. a

4.26 Exerc cio. Prove que em qualquer tringulo equiltero as trs medianas so congruentes. a a e a 4.27 Exerc cio. Prove que em dois tringulos congruentes as bissetrizes dos ngulos respeca a
tivamente congruentes so congruentes. a

4.28 Exerc cio. Prove que em dois tringulos congruentes as medianas relativas a lados rea
spectivamente congruentes so congruentes. a

39

||

Exerc cios complementares 4.29 Exerc cio. Detalhe o procedimento que voc utilizaria para construir um tringulo cone a
gruente a um tringulo dado. a

4.30 Exerc cio. Prove que todo tringulo, no qual uma altura e uma bissetriz so coincidentes, a a
issceles. e o

4.31 Exerc cio. Prove que todo tringulo, no qual uma altura e uma mediana so coincia a
dentes, issceles. e o

4.32 Exerc cio. Prove que todo tringulo equiltero tem os trs ngulos congruentes, e recipa a e a
rocamente. suas respectivas bissetrizes OM , ON e marcam-se sobre as semi-retas OX, OM , OY , ON OZ, respectivamente, os segmentos congruentes OA OB OC OD OE. Encontre a relao ca entre os segmentos AB, BC, CD e DE. Compare os ngulos B AC e DCE. a

4.33 Exerc cio. So dados dois ngulos adjacentes congruentes: X OY e Y OZ. Traam-se a a c

4.34 Exerc cio. Duas estradas retil neas ligam uma cidade A a duas cidades B e C, conforme indicado na gura 4.4. Indique uma estratgia que permita voc determinar a e e distncia entre as cidades B e C, sabendo que entre B e C existe um morro (qualquer a medio que no seja atravs do morro poss ca a e e vel).
B

. .C

.
Figura 4.4: Exerc 4.34 cio

4.35 Exerc cio. Na gura 4.5, os tringulos ABC e ABD so issceles com base comum a a o AB. Prove que os ngulos C AD e C BD so congruentes e que CD bissetriz do ngulo a a e a ACB.
C

Figura 4.5: Exerc 4.35. cio

40

Exerc cios complementares 4.36 Exerc cio. Na gura 4.5, o ngulo C M A reto e M ponto mdio de AB.Prove a e e e que AC BC.

.C

Figura 4.6: Exerc cios 4.36.

41

Exerc cios complementares

42

Cap tulo 5 Desigualdades nos Tringulos a


Estabelece-se, usando o teorema do ngulo externo, a exa istncia de retas paralelas, a existncia e unicidade da pere e pendicular a uma reta dada passando por um ponto no a pertencente a essa reta, e uma condio necessria para ca a que trs n meros positivos possam ser comprimentos de e u lados de um tringulo. a

5.1

O teorema do ngulo externo e algumas de suas a conseqncias ue

5.1 Denio. Os suplementos dos ngulos internos de um tringulo so chamados ca a a a ngulos externos do tringulo. a a 5.2 Teorema. (Teorema do Angulo Externo) Um ngulo externo de um tringulo a a mede mais do que qualquer um dos ngulos internos no adjacentes a ele. a a Prova: Seja ABC um tringulo qualquer. Na semi-reta AB, marquemos um ponto a D tal que B esteja entre A e D. O teorema estar provado se mostrarmos que C BD > C a e C BD > A. Mostremos, inicialmente, que C BD > C. Para isso, consideremos o ponto mdio e C N M do segmento BC. Na semi-reta S(AM ) marquemos um ponto N tal que AM N e AM = M N. Ficam, assim, denidos os tringulos CM A e BM N. a M Como CM = BM, AM = M N e AM C = B M N (por qu?), segue que e A B D os tringulos CM A e BM N so congrua a entes. Logo, C = M BN. Como a semi-reta Figura 5.1: O Teorema do Angulo Externo. BN divide o ngulo C BD ento M BN < a a C BD e, da C < C BD. De modo anlogo, o leitor pode provar que C BD > A. , a

43

O teorema do ngulo externo e algumas de suas conseqncias a ue 5.3 Exerc cio. Complete a demonstrao do teorema anterior. ca 5.4 Corolrio. A soma das medidas de quaisquer dois ngulos internos de um tringulo a a a menor do que 180 . e Prova: Seja ABC um tringulo qualquer; mostremos que A + B < 180 . Chamemos a de a medida do ngulo externo deste tringulo com vrtice em B. Pelo teorema 5.2 a a e < . Como e B so suplementares, temos que + B = 180 . Logo temos que A a + B < + B = 180 . A 5.5 Corolrio. Em todo tringulo existem pelo menos dois ngulos internos agudos. a a a Prova: Suponhamos que um tringulo possu a sse dois ngulos internos no agudos. a a Neste caso a soma deles seria maior ou igual a 180 , o que no pode ocorrer devido ao a teorema 5.4. Logo, vale o corolrio. a 5.6 Denio. Duas retas que no se interceptam so ditas paralelas. ca a a 5.7 Corolrio. Se r e s so retas distintas e perpendiculares a uma terceira, ento r e s a a a so paralelas. a Prova: Se r e s se interceptassem, ter amos denido um tringulo com dois ngulos a a retos, o que um absurdo devido ao corolrio 5.5. e a 5.8 Teorema. (Quarto Caso de Congruncia de Tringulos ou caso LAA ) e a Dois tringulos ABC e XY Z so congruentes se AB XY , B Y e C Z. a a 5.9 Observao. O s ca mbolo A do teorema anterior representa o ngulo oposto ao lado a BC de um tringulo ABC. a 5.10 Exerc cio. Prove o teorema anterior. 5.11 Exerc cio. Mostre que existem retas paralelas (sugesto: use o teorema 3.46). a O teorema seguinte nos fornecer um outro mtodo para constru a e rmos retas perpendiculares. Como consequncia do corolrio 5.7, podemos usar este mtodo para construir e a e retas paralelas. 5.12 Teorema. Por um ponto no pertencente a uma reta passa uma unica reta perpena dicular ` essa reta. a Prova: (Existncia) Seja r uma reta e P um ponto no pertencente ` r. Tomemos e a a em r um ponto A, qualquer. Consideremos o segmento AP ; se AP j perpendicular ` ae a r, acabamos de construir a reta procurada, a saber, aquela determinada por A e P . Caso contrrio, observemos que A divide a reta r em duas semi-retas, r e r. A a semi-reta S(AP ) dene, com uma dessas semi-retas, um ngulo agudo com vrtice em A. a e Suponhamos que r seja essa semi-reta e chamemos esse ngulo de P Ar . a 44

Desigualdades nos tringulos a Consideremos, agora, o semiplano denido pela reta r e que no contm P ; nesse a e semiplano construamos uma semi-reta s com origem A e que dena com r um ngulo a congruente a P Ar . Sobre essa semi-reta tomemos um ponto P tal que AP = AP . Mostremos que o segmento P P perpendice r' P ular ` r. De fato, uma vez que P e P so pona a tos pertencentes a semiplanos distintos determinados por r, temos que P P intercepta r num M unico ponto M . Ficam assim denidos dois tringulos, P AM a e P AM . Desde que AP = AP , P AM = P AM s' A P' e AM = AM, os tringulos P AM e P AM so a a r'' congruentes, e da P M A = P M A. Isso implica , que P P perpendicular ` r (vide teorema 3.44) e a e da a reta s determinada por P e P perpen, e Figura 5.2: Teorema 5.12. dicular ` r. Acabamos de provar a existncia. a e (Unicidade) Alm de s, suponhamos que exista outra reta, t, que passa por P e e e perpendicular ` r. Neste caso, ter a amos um tringulo com dois ngulos retos, o que um a a e absurdo de acordo com o corolrio 5.5. Logo, a reta passando por P e perpendicular ` r a a unica. e 5.13 Denio. (a) O ponto M no teorema 5.12 chamado de p da perpendicular ca e e baixada do ponto P a reta r. (b) Se N qualquer outro ponto de r, distinto de M, dizemos que o segmento P N e e obl quo em relao a r. ca (c) O segmento M N chamado de projeo de P N sobre a reta r. e ca (d) O nmero P M chamado de distncia do ponto P a reta r. u e a 5.14 Observao. Na denio 4.13, deniu-se altura de um tringulo, mas nada garanca ca a tia sua existncia; esta agora garantida pelo teorema 5.12. e e

5.2

Desigualdades nos tringulos a

5.15 Teorema. Se num tringulo, dois de seus lados no so congruentes, ento os a a a a a ngulos que se opem a esses lados no tm mesma medida e ao maior lado ope-se o a e o o maior ngulo. a Prova: Suponhamos que no tringulo considerado os ngulos que se opem aos dois laa a o dos no congruentes tenham mesma medida; pelo teorema 4.17, resulta que esse tringulo a a issceles, o que um absurdo. Logo, vale a primeira acertiva do teorema. e o e Mostremos agora que ao maior lado ope-se o maior ngulo. Para isso, consideremos o a um tringulo ABC tal que AB > BC; mostremos que ACB > B AC. Sobre a semi-reta a S(BA) , a partir de B, marquemos um ponto X tal que BC = BX. Pelo lema 3.4, como 45

Desigualdades nos tringulos a BC < AB, o ponto X pertence ao segmento AB, e da a semi-reta CX divide o ngulo , a ACB. Logo, tem-se ACB > B CX. (5.1) No entanto, observemos que B CX = C XB > X AC = B AC, (5.2)

uma vez que o tringulo BCX issceles, e a desigualdade ocorre, uma vez que C XB a e o e a ngulo externo do tringulo CAX. Logo, de (5.1) e (5.2), temos que ACB > B AC. a 5.16 Corolrio. Se num tringulo, dois de seus ngulos no so congruentes, ento os a a a a a a lados que se opem a esses ngulos no tm a mesma medida e ao maior ngulo ope-se o a a e a o o maior lado. 5.17 Exerc cio. Prove o corolrio anterior (sugesto: a primeira parte deste corolrio a a a segue diretamente do teorema 4.17; a prova da segunda parte uma aplicao esperta e ca do teorema 5.15). 5.18 Teorema. Em qualquer tringulo, o comprimento de um lado menor do que a a e soma dos comprimentos dos outros dois. Prova: Consideremos um tringulo qualquer ABC; mostremos que AC < AB + BC. a Para tanto, na semi-reta S(AB) , tomemos C um ponto X de tal modo que B est ena tre A e X, e que BX = BC. Assim, o tringulo CBX issceles com vrtice B a e o e e AX = AB + BC. Logo, temos B CX = B XC. Como B est entre A e X temos a B X A que B XC = B CX < ACX. Logo, pelo corolrio 5.16, segue-se que AC < AX, e a Figura 5.3: Teorema 5.18. da AC < AB + BC. 5.19 Teorema. (Desigualdade triangular) Para quaisquer trs pontos distintos A, B, C e (colineares ou no) tem-se AB AC + CB. A igualdade vale se, e somente se, A C B. a 5.20 Exerc cio. Prove o teorema anterior. 5.21 Observao. Para podermos construir um tringulo cujos lados tm comprimentos ca a e dados, tais comprimentos devem, necessariamente, satisfazer ` desigualdade triangular. a Assim, imposs construir um tringulo cujos lados medem 4, 5 e 11. e vel a 5.22 Teorema. Em qualquer tringulo, o comprimento de um lado maior do que a a e diferena dos comprimentos dos outros dois. c 5.23 Exerc cio. Prove o teorema anterior. 46

Exerc cios complementares

5.3

Exerc cios complementares

5.24 Exerc cio. A soma dos comprimentos dos lados de um tringulo seu per a e metro, e a
metade do per metro o semiper e metro. Mostre que o comprimento de qualquer lado de um tringulo menor do que seu semiper a e metro.

5.25 Exerc cio. Justique as seguintes armaes relativas a tringulos retngulos: co a a


a) os ngulos opostos aos catetos so agudos; a a b) o comprimento da hipotenusa maior do que o comprimento de qualquer cateto; e c) o comprimento da hipotenusa menor do que a soma dos comprimentos dos catetos. e d) Se dois tringulos retngulos so congruentes, os ngulos retos devem se corresponder; a a a a e) (Congruncia de tringulos retngulos) Sejam ABC e A B C dois tringulos retngulos e a a a a cujos ngulos retos so A e A , respectivamente. Mostre que a a a a a i) se BA B A e C C ento os tringulos so congruentes, ii) se CB C B e BA B A ento os tringulos so congruentes, a a a iii) se CB C B e C C ento os tringulos so congruentes. a a a

5.26 Exerc cio. O exerc 5.25-e-(ii) caracteriza um caso (LLA) de congruncia de tringulos cio e a
retngulos. Mostre que esse no um caso de congruncia para tringulos no-retngulos. a a e e a a a

5.27 Exerc cio. Mostre que o lugar geomtrico dos pontos eqidistantes de dois pontos A, B, e u
a mediatriz do segmento AB (a expresso lugar geomtrico dos pontos signica conjunto e a e dos pontos).

5.28 Exerc cio. Mostre que o lugar geomtrico dos pontos eqidistantes das semi-retas S(OA) e u
e S(OB) a bissetriz do ngulo AOB. e a

5.29 Exerc cio. Prove que em qualquer tringulo issceles as alturas relativas aos lados cona o
gruentes so congruentes. a

5.30 Exerc cio. Prove que um tringulo que possui duas alturas congruentes issceles. a e o 5.31 Exerc cio. Prove que um tringulo equiltero tem as trs alturas congruentes. a a e 5.32 Exerc cio. Prove que em tringulos congruentes as alturas relativas a lados respectivaa
mente congruentes so congruentes. a

5.33 Exerc cio. Mostre que, num tringulo retngulo cujos ngulos agudos medem 30 e 60 , a a a
o menor cateto mede metade do comprimento da hipotenusa, e reciprocamente.

5.34 Exerc cio. Prove que tringulos que tm dois ngulos externos congruentes so issceles. a e a a o 5.35 Exerc cio. Prove que todo tringulo retngulo tem dois ngulos externos obtusos. a a a 5.36 Exerc cio. Com relao ` denio 5.13, mostre que P N > P M e P N > N M. ca a ca 47

Exerc cios complementares 5.37 Exerc cio. Mostre que o ponto P obtido na demonstrao do teorema 5.12, independe ca
do ponto A escolhido sobre a reta r.

5.38 Exerc cio. Na construo feita no teorema 5.12, o ponto P , obtido a partir do ponto ca
P e da reta r, chamado de reexo do ponto P relativamente ` reta r (ou imagem de P e a relativamente ` reta r). Mostre que P o reexo de P relativamente ` reta r se, e somente se, a e a P P perpendicular ` r e r intercepta P P no seu ponto mdio. e a e

5.39 Exerc cio. Se P r, por denio, P = P . Seja r uma reta xada no plano; designemos ca
por r a funo que a cada ponto do plano associa o seu reexo relativamente ` reta r. Esta ca a funo chamada reexo. Prove que a funo reexo goza das seguintes propriedades: ca e a ca a i) para todo ponto P do plano, r (r (P )) = P ; ii) r uma isometria, isto , ela preserva distncia entre pontos do plano. Assim, para e e a quaisquer pontos P e Q do plano, tem-se: r (P )r (Q) = P Q; iii) se P r e Q r e Q = r (Q) ento r a bissetriz do ngulo QP Q . / a e a

5.40 Exerc cio. Mostre que poss e vel, utilizando somente uma rgua no graduada e um e a
compasso, construir um tringulo cujos lados so segmentos (dados) com comprimentos a, b, c a a tais que c < a + b, sendo c o maior lado do tringulo. a

5.41 Exerc cio. Dados dois pontos, A e B, no pertencentes a uma reta r, mostre que existe a um ponto X sobre r tal que AX + XB m e nimo. Considere os dois casos: a) A e B esto em a
semiplanos distintos relativamente a r; b) A e B esto em um mesmo semiplano relativamente a a r.

5.42 Exerc cio. Na gura 5.4 (pg.7), = ; mostre que as retas r e s so paralelas. a a 5.43 Exerc cio. Use o exerc 5.42 para construir duas retas paralelas. cio 5.44 Exerc cio. Na gura 5.5 (pg.7), r e s so retas perpendiculares e P, Q so pontos dados. a a a
Determine o caminho mais curto para se ir do ponto P ao ponto Q tocando-se uma unica vez em cada reta.

5.45 Exerc cio. Na gura 5.6 (pg.7), se ABC equiltero e AD = BE = CF, mostre que o a e a
tringulo EDF equiltero. a e a

5.46 Exerc cio. Prove que em qualquer tringulo, a soma dos comprimentos das medianas a
est compreendida entre o per a metro e o semiper metro.

5.47 Exerc cio. Se ABC um tringulo e P um ponto de seu interior, mostre que vale a e a relao: P A + P BCA + CB. ca 48

Exerc cios complementares 5.48 Exerc cio. Se ABC um tringulo e P um ponto de seu interior, mostre que a soma e a
das distncias de P aos trs vrtices est compreendida entre o per a e e a metro e o semiper metro do referido tringulo. a

5.49 Exerc cio. Mostre que, dados uma reta e um ponto no pertencente a ela, dentre toa
dos os segmentos com uma extremidade neste ponto e outra num ponto da reta, o de menor comprimento aquele que perpendicular ` reta dada. e e a

t r

Figura 5.4: t transversal as retas r e s.

A
s

.P .Q
r

F E B D C

Figura 5.5: rs

Figura 5.6: Exerc 5.45 cio

49

Exerc cios complementares

50

Cap tulo 6 Axioma das Paralelas


O corolrio 5.7 acena para a possibilidade de existirem retas a paralelas; os teoremas 3.46 e 5.12 do meios de construir retas a paralelas e, da permitir armar que elas existem. O axioma , seguinte arma a unicidade de reta passando por um ponto e paralela a uma reta dada.

6.1


O axioma das paralelas e algumas consequncias e


Axioma 18 Por um ponto no pertencente a uma reta r passa uma unica reta paralela ` r. a a


Deste axioma decorre que o paralelismo de retas goza da propriedade transitiva, como mostra o teorema seguinte. 6.1 Teorema. Sejam r, s, w retas duas a duas distintas. Se r paralela ` s e s paralela ` w, e a e a ento r paralela a w. a e Prova: Se r e w no fossem paralelas, elas seriam concorrentes num ponto P (teorema 2.9); a desse modo ter amos duas retas passando por um ponto e paralelas a s, o que contradiz o axioma 18. Assim, r e w so paralelas. a 6.2 Teorema. Se uma reta intercepta uma de duas retas paralelas, ento ela tambm intercepta a e a outra. Prova: Se r e s so retas paralelas e se t interceptasse uma delas, digamos r, num ponto a P , mas no interceptasse a outra, s, ter a amos, passando por P , duas retas, r e t, paralelas ` s. a Isto contradiria o axioma 18. Antes de enunciarmos as proposies que garantem paralelismo, vamos, para efeito de nomenco clatura, dar algumas denies. Dadas duas retas r e s, cortadas por uma reta transversal t, co cam determinados oito ngulos, como os indicados na gura 6.1. a

51

O axioma das paralelas e algumas consequncias e


t ^ 6 ^ 7

^ 5 ^ 8 ^ 2 ^ 1 ^ 3 ^ 4

Os pares 1 e 5, 2 e 6, 3 e 7, 4 e 8 so ngulos correa a spondentes. Os pares 1 e 7, 2 e 8 so ngulos alternos internos. a a Os pares 3 e 5, 4 e 6 so ngulos alternos externos. a a Os pares 1 e 8, 2 e 7 so ngulos colaterais internos. a a Os pares 4 e 5, 3 e 6 so ngulos colaterais extera a nos. 6.3 Exerc cio. Conclua que 1 = 3, 2 = 4, 5 = 7 e 6 = 8.

Figura 6.1: Retas r, s e t.

6.4 Exerc cio. Mostre que se 1 = 5 ento 2 = 6, 3 = 7 e a 4 = 8.

Os teoremas seguintes do meios de se poder armar o paralelismo de duas retas. a 6.5 Teorema. Se r e s so retas cortadas por uma transversal t, de modo que um par de a a ngulos correspondentes (alternos internos) so congruentes, ento as retas r e s so paralelas. a a a Prova: Suponhamos que r e s se interceptem num ponto P ; se A o ponto de interseo e ca de r com t e B o ponto de interseo de s com t, os trs pontos A, B, P denem um tringulo ca e a ABP (gura 6.2). Suponhamos que 3 e 7 o par de ngulos e a t r correspondentes que, por hiptese, so cono a gruentes. No tringulo ABP , 3 um ngulo a e a externo e 7 um ngulo interno no adjacente e a a ^ ao ngulo 3. Pelo teorema do ngulo externo a a 7 (teorema 5.2), ter amos 3 = 7, o que contraria a hiptese de serem congruentes. Logo, r e s o ^ 2 s no se interceptam. a

^ 3

Figura 6.2: Tringulo ABP. a

6.6 Exerc cio. Prove o teorema anterior, no caso em que um par de ngulos alternos intera nos so congruentes. a

6.7 Corolrio. Se r e s so retas cortadas por uma transversal, t, de modo que um par de a a a ngulos alternos externos so congruentes, ento as retas r e s so paralelas. a a a 6.8 Corolrio. Se r e s so retas cortadas por uma transversal t, de modo que um par de a a a ngulos colaterais internos (colaterais externos) so suplementares, ento as retas r e s so a a a paralelas. 6.9 Corolrio. Se r e s so retas cortadas por uma transversal t, de modo que um par de a a a ngulos colaterais internos (colaterais externos) no so suplementares, ento as retas r e s no a a a a so paralelas. a 6.10 Exerc cio. Prove o corolrio acima. a O rec proco do teorema 6.5 tambm verdadeiro, e um resultado importante que ser e e e a registrado no seguinte:

52

O axioma das paralelas e algumas consequncias e


6.11 Teorema. Duas retas paralelas cortadas por uma reta transversal determinam ngulos a correspondentes congruentes. Prova: Sejam r e s duas retas paralelas e t uma reta que corta r e s nos pontos A e B, respectivamente. Consideremos r uma reta passando por A e que t r' forma com t quatro ngulos congruentes aos ngulos a a correspondentes formados por s e t. Pelo teorema A 6.5, r e s so paralelas. Pelo axioma 18, r e r devem a r ser coincidentes. Logo, r forma ngulos com a reta a t congruentes aos correspondentes formados por s e t. 6.12 Exerc cio. Prove o teorema anterior no caso em que um par de ngulos alternos internos so cona a gruentes.

Figura 6.3: Teorema 6.11

6.13 Exerc cio. Prove que duas retas paralelas cortadas por uma reta transversal determinam a ngulos alternos externos congruentes. 6.14 Exerc cio. Prove que duas retas paralelas cortadas por uma reta transversal determinam a ngulos colaterais internos (colaterais externos) suplementares. 6.15 Teorema. a) Se uma reta r paralela ` uma reta s e uma reta m paralela ` uma reta e a e a n, de tal modo que m transversal a r e s, ento o mesmo ocorre com n. e a b) Segmentos de paralelas compreendidos entre paralelas so congruentes. a 6.16 Exerc cio. Prove o teorema 6.15 6.17 Teorema. Prove que o segmento de paralela ` base do ABC pelo ponto mdio M do a e lado AB, passa pelo ponto mdio do outro lado e mede a metade do comprimento da base BC. e 6.18 Exerc cio. Prove o teorema 6.17 6.19 Observao. O axioma 18 (axioma das paralelas), da forma como est enunciado, ca a e atribu ao matemtico escocs John Playfair (1748 1819), embora j fosse conhecido por do a e a Proclus, no sculo V d.C. e usado por vrios autores. Euclides (300 a.C.) apresentou um e a axioma das paralelas (quinto postulado) numa forma um pouco diferente, a saber: Se uma linha reta encontrando-se com outras duas retas zer ngulos internos da mesma parte menores a que dois retos, estas duas retas, prolongadas ao innito, concorrero para a mesma parte dos a ditos ngulos internos. Outros quatro axiomas, xados por Euclides no Elementos so os a a seguintes: 1- Pode-se traar uma reta passando por dois pontos; c 2- Uma reta pode ser continuada at onde seja necessrio; e a 3- Pode-se traar uma circunferncia com qualquer centro e qualquer distncia; c e a 4- Todos os ngulos retos so iguais. a a No Elementos de Euclides, as 28 primeiras proposies do Livro 1 foram demonstradas co somente com base nos quatro axiomas acima. Pode-se provar que nosso axioma 18, junto com

53

O axioma das paralelas e algumas consequncias e


os quatro axiomas acima, equivalente `quele enunciado por Euclides. Outras declaraes que e a co podem ser tomadas como axiomas no lugar do axioma das paralelas, e que do origem `s mesmas a a proposies demonstradas no Elementos, podem ser encontradas em [13]. co 6.20 Teorema. A soma das medidas dos ngulos internos de qualquer tringulo 180 . a a e Prova: Seja ABC um tringulo qualquer. Pelo vrtice C, consideremos a reta paralela ao a e lado AB. C determina sobre essa reta duas semi-retas, S(CX) e S(CY ) , onde X um ponto no e semiplano determinado pela reta BC e que no contm A, enquanto Y um ponto no semiplano a e e determinado pela reta AC e que no contm B. Temos que: a e X CB + B CA + ACY = 1 raso.

.
Y A

.
X B

Como as retas determinadas por X, Y e por A, B, respectivamente, so paralelas e a reta dea terminada por A, C transversal a elas, o teorema e 6.11 implica que ACY = B AC. Analogamente, conclu mos que X CB = C BA. Logo C BA + B CA + = 1 raso ou, A + B + C = 180 . B AC

Figura 6.4: Teorema 6.20


6.21 Corolrio. Em qualquer tringulo, a medida de um ngulo externo igual ` soma das a a a e a medidas dos ngulos internos que no lhe so adjacentes. a a a 6.22 Exerc cio. Prove o corolrio anterior. a 6.23 Exerc cio. Mostre que, num tringulo retngulo a soma das medidas dos ngulos agudos a a a 90 . e 6.24 Exerc cio. Os ngulos internos de um tringulo equiltero medem 60 . Prove isso. a a a 6.25 Teorema. Se r e s so retas paralelas, ento qualquer ponto de r dista igualmente de s. a a 6.26 Exerc cio. Prove o teorema anterior (sugesto: utilize o teorema 6.15). a O teorema 6.25 motiva a seguinte denio. ca 6.27 Denio. A distncia entre duas retas paralelas a distncia de um ponto qualquer ca a e a de uma delas a outra. A distncia entre duas retas concorrentes zero. a e 6.28 Observao. Pode-se provar que o teorema 5.4 (ou teorema 5.12), junto com os quatros ca primeiros postulados de Euclides, equivalente ao axioma das paralelas. e

54

Exerc cios complementares

6.2

Construo de um sistema de coordenadas no ca plano

Sejam r e s retas concorrentes num ponto O. Em cada uma delas tomamos um sistema de coordenadas que tenha o ponto O como origem. Chamamos a reta r de eixo das abscissas e a reta s de eixo das ordenadas. O par ordenado de retas (r, s) chamado um sistema e de coordenadas, e o ponto O, a origem do sistema. Dado um ponto P do plano, a ele e associamos um par ordenado de nmeros reais como segue. u (a) Se P r, ento associamos a P o par (x, 0) , a s sendo x a coordenada de P em relao ` r. ca a y (b) Se P s, ento associamos a P o par (0, y) , a P. sendo y a coordenada de P em relao ` s. ca a (c) Se P r e P s, ento passamos por P uma / / a paralela ` s, que encontra r num ponto cuja coora denada chamaremos de x, e uma paralela ` r, que a x O r encontra s num ponto cuja coordenada chamaremos de y. Ao ponto P associamos o par (x, y) acima constru do. Figura 6.5: Sistema de coordenadas no O par de nmeros reais (x, y) , por denio, plano. u e ca as coordenadas do ponto P no sistema de coordenadas (r, s) . O nmero x chamado de abscissa de P e o nmero y de ordenada de P. u e u Quando as retas r e s so perpendiculares, o sistema de coordenadas (r, s) chamado de a e sistema de coordenadas cartesianas ortogonais. 6.29 Observao. A aplicao que a cada ponto do plano associa um par ordenado de nmeros ca ca u reais, como denido acima, bijetiva. Isto nos permite identicar pontos do plano com pares e ordenados de nmeros reais, abrindo a possibilidade de resolvermos problemas geomtricos utiu e lizando lgebra, como se faz normalmente em Geometria Anal a tica. Por isso muitas vezes escrevemos P = (x, y) . 6.30 Exerc cio. Justique a armao feita na observao 6.29. ca ca

6.3

Exerc cios complementares

6.31 Exerc cio. Prove que a bissetriz de um ngulo externo, relativo ao vrtice de um tringulo a e a issceles, paralela ` base desse tringulo. o e a a 6.32 Exerc cio. Sejam ABC um tringulo issceles e P um ponto qualquer da base BC. Sejam a o P M e P N os segmentos perpendiculares `s laterais desse tringulo. Mostre que P M + P N a a e um valor constante, que a medida da altura relativa a uma das laterais. e 6.33 Exerc cio. Provar que se P um ponto interior a um tringulo equiltero, ento a soma e a a a das distncias de P ao lados do tringulo igual ` altura do mesmo. a a e a

55

Exerc cios complementares


6.34 Exerc cio. Prove que se r uma reta cujos pontos so equidistantes de uma reta s (isto e a , todos os pontos de r esto a mesma distncia de s), ento r e s so retas coincidentes ou e a a a a paralelas (este o rec e proco do teorema 6.25). 6.35 Exerc cio. Descubra qual o erro na demonstrao do Teoremado Apndice A. ca e

56

Cap tulo 7 Pol gonos


Apresenta-se uma srie de denies seguidas de exerc e co cios que so, basicamente, consequncias de resultados dos cap a e tulos anteriores, principalmente daqueles sobre congruncia e e paralelismo. Os resultados de muitos exerc cios so ina formaes importantes (verdadeiros teoremas), outros so cuco a riosidades que valem pelo treinamento que eles impem ao o racioc nio.

7.1

Denies gerais co

7.1 Denio. Uma linha poligonal, ou simplesmente poligonal, uma gura geomtrica ca e e formada por uma sequncia de n (n 3) pontos distintos A1 , A2 , ..., An , e pelos segmentos A1 A2 , e A2 A3 , ... , An1 An . Os pontos so os vrtices e os segmentos os lados da poligonal. Angulo a e de uma poligonal com vrtice Aj o ngulo denido pelos lados que tm Aj como ponto comum. e e a e

7.2 Denio. Pol ca gono uma poligonal que satisfaz as seguintes condies: e co (i) An+1 = A1 ; (ii) os lados da poligonal interceptam-se somente em suas extremidades; (iii) dois lados com mesma extremidade no pertencem a uma mesma reta. a

7.3 Observao. Um pol ca gono com vrtices A1 , A2 , ..., An , ser denotado por A1 A2 ...An . Ele e a tem n lados, n vrtices e n ngulos. e a

7.4 Observao. Uma classicao para pol ca ca gonos pode ser feita, facilmente, segundo o nmero u de lados. Os mais usuais so (para n 3, onde n representa o nmero de seus lados): a u

57

Denies gerais co
N mero de lados u 3 4 5 6 7 8 9 10 11 12 15 20 Nome do pol gono tringulo a quadriltero a pentgono a hexgono a heptgono a octgono a enegono a decgono a undecgono a dodecgono a pentadecgono a icosgono a

7.5 Exerc cio. Quais dos desenhos da gura 7.1, representam pol gonos?

(a)
D B E B A

(b)
C D

.
F

(c)
E C D B B A C

(d)
D E F

Figura 7.1: Exerc 7.5. cio


7.6 Denio. Diagonal de um pol ca gono um segmento que tem por extremidade dois de e seus vrtices que no pertencem a um mesmo lado. e a 7.7 Exerc cio. (a) Quantas diagonais tm um pol e gono de: (i) 6 lados; (ii) 17 lados ? (b) Mostre que um pol gono com n lados tem n(n3) diagonais. 2 7.8 Exerc cio. Qual o pol gono que possui 27 diagonais distintas? 7.9 Denio. Um pol ca gono que possui todos os ngulos congruentes dito equingulo. Um a e a pol gono que possui todos os lados congruentes dito equiltero. e a

58

Denies gerais co
7.10 Exerc cio. Mostre que um pol gono separa o plano em dois subconjuntos dos quais um e limitado e outro no limitado. e a 7.11 Denio. O subconjunto limitado menos os pontos do pol ca gono chamado interior e do pol gono. Se do subconjunto no limitado exclu a rmos os pontos do pol gono, teremos o exterior do pol gono. 7.12 Denio. Um pol ca gono convexo se cada reta que contm dois vrtices do pol e e e gono determina um semiplano que contm todos os outros vrtices. e e 7.13 Exerc cio. Mostre que o interior de um pol gono convexo uma gura convexa. e 7.14 Exerc cio. Sejam P um pol gono convexo, A um ponto no interior de P e B um ponto em seu exterior. Mostre que existe um unico ponto comum a P e ao segmento AB. 7.15 Denio. Angulo de um pol ca gono convexo chamado de ngulo interno do pol e a gono. Angulo externo de um pol gono convexo um suplemento de um ngulo interno do pol e a gono. 7.16 Denio. Um pol ca gono regular um pol e gono convexo, equiltero e equingulo, isto a a , todos os lados so congruentes entre si e todos os ngulos internos so congruentes entre si. e a a a 7.17 Teorema. A soma das medidas dos ngulos internos de um pol a gono convexo igual a e , onde n o n mero de lados do pol [180(n 2)] e u gono. 7.18 Exerc cio. Prove o teorema anterior. 7.19 Teorema. A soma das medidas dos ngulos externos de um pol a gono convexo vale 360 . 7.20 Exerc cio. Prove o teorema anterior. 7.21 Exerc cio. Nos dois exerc cios anteriores poder amos tirar o adjetivo convexo e manter a concluso das proposies? a co 7.22 Exerc cio. Mostre que um pol gono convexo no pode ter mais do que trs ngulos agudos. a e a 7.23 Exerc cio. Calcule a medida do ngulo interno de um pol a gono regular de n lados. 7.24 Exerc cio. Calcule a medida do ngulo externo de um pol a gono regular de n lados. 7.25 Exerc cio. Quer-se revestir um soalho com tacos na forma de pol gonos regulares. Que tipo de pol gonos, de mesmo formato, pode-se utilizar de modo a cobrir todo o soalho? 7.26 Exerc cio. Suponha que para revestir um soalho possam ser utilizadom tacos na forma de pol gonos regulares, mas no necessariamente de mesmo formato. Estude as possibilidades a para tal revestimento. 7.27 Denio. Per ca metro de um pol gono a soma das medidas de seus lados. Semie per metro de um pol gono a metade do per e metro.

59

Quadrilteros convexos a
7.28 Exerc cio. Seja A1 A2 ...An um pol gono convexo e A1 A2 ...An outro pol gono convexo que possui cada um de seus vrtices sobre os lados do pol e gono anterior. Prove que o per metro do pol gono A1 A2 ...An menor do que o per e metro do pol gono A1 A2 ...An . 7.29 Exerc cio. Mostre que, em todo pol gono convexo, a soma dos comprimentos dos segmentos denidos por seus vrtices e um ponto no seu interior maior do que seu semiper e e metro.

7.2

Quadrilteros convexos a

7.30 Denio. Quadrilteros convexos so pol ca a a gonos convexos que possuem quatro lados. 7.31 Denio. Num quadriltero, vrtices no consecutivos so ditos opostos, assim como ca a e a a dois ngulos e dois lados no consecutivos so ditos opostos. a a a 7.32 Exerc cio. A soma das medidas dos ngulos internos de um quadriltero convexo igual a a e a 4 vezes a medida de um ngulo reto. a 7.33 Denio. Paralelogramo um quadriltero no qual os lados opostos so paralelos. ca e a a 7.34 Teorema. Em um paralelogramo temos sempre: (i) ngulos adjacentes a um lado suplementares; a (ii) ngulos opostos congruentes; a (iii) lados opostos congruentes; (iv) as diagonais se interceptam em um ponto que o ponto mdio das duas diagonais. e e 7.35 Exerc cio. Prove o teorema 7.34. 7.36 Exerc cio. Mostre que: (i) Se um paralelogramo tem um ngulo reto ento todos os demais ngulos so retos. a a a a (ii) Se num paralelogramo dois de seus lados consecutivos so congruentes, ento todos os a a seus lados so congruentes. a 7.37 Teorema. Um quadriltero um paralelogramo se: a e (i) os ngulos adjacentes a cada um dos seus lados so suplementares; a a (ii) os ngulos opostos so congruentes; a a (iii) os lados opostos so congruentes; a (iv) as diagonais interceptam-se mutuamente em seus pontos mdios. e 7.38 Exerc cio. Prove o teorema 7.37. 7.39 Teorema. Se um quadriltero possui dois lados opostos congruentes e paralelos, ento ele a a um paralelogramo. e 7.40 Exerc cio. Prove o teorema 7.39. 7.41 Denio. Um quadriltero que possui todos os seus ngulos retos recebe o nome de ca a a retngulo. a

60

Quadrilteros convexos a
7.42 Exerc cio. Mostre que todo retngulo um paralelogramo. a e 7.43 Exerc cio. Mostre que as diagonais de um retngulo so congruentes. a a 7.44 Exerc cio. Mostre que se as diagonais de um paralelogramo so congruentes, ento o a a paralelogramo um retngulo. e a 7.45 Denio. Losango um quadriltero que tem os quatro lados congruentes. ca e a 7.46 Exerc cio. Mostre que todo losango um paralelogramo. e 7.47 Exerc cio. Mostre que, num losango, as diagonais so perpendiculares entre si e cada a uma bissetriz do ngulo correspondente. e a 7.48 Exerc cio. Um paralelogramo um losango se: e (i) suas diagonais so perpendiculares entre si; a (ii) uma das diagonais bissecta os ngulos opostos. a 7.49 Denio. Quadrado um retngulo que tambm um losango. ca e a e e 7.50 Exerc cio. Mostre que num quadrado, (i) as diagonais so congruentes e perpendiculares; a (ii) cada diagonal bissetriz dos seus ngulos. e a 7.51 Exerc cio. Mostre que se as diagonais de um quadriltero so congruentes e se intercepa a tam num ponto que ponto mdio de ambas, e ainda so perpendiculares, ento o quadriltero e e a a a um quadrado. e 7.52 Denio. Trapzio um quadriltero em que somente dois lados so paralelos. ca e e a a 7.53 Denio. Os lados paralelos de um trapzio so chamados bases e os outros dois so ca e a a as suas laterais. Um trapzio dito issceles se suas laterais so congruentes. e e o a 7.54 Denio. Um trapzio que possui um ngulo reto dito trapzio retngulo. ca e a e e a 7.55 Exerc cio. Se ABCD um trapzio issceles e AB uma base, mostre que A = B, C = D, e e o e e reciprocamente (basta A = B ou C = D). 7.56 Exerc cio. Mostre que num trapzio issceles, as diagonais so congruentes, e reciprocae o a mente. 7.57 Exerc cio. Mostre que num trapzio issceles, a mediatriz de uma das suas bases mee o e diatriz da outra base, e reciprocamente. 7.58 Teorema. O segmento determinado pelos pontos mdios de dois lados de um tringulo e a e paralelo ao terceiro lado e mede a metade do comprimento desse lado. 7.59 Exerc cio. Demonstre o teorema anterior (sugesto: se X e Y so os pontos mdios de a a e dois lados, marque em XY um ponto D tal que XD = 2XY ).

61

Quadrilteros convexos a
7.60 Exerc cio. Prove que qualquer tringulo que possui duas medianas congruentes issceles. a e o 7.61 Exerc cio. Mostre que o segmento que liga os pontos mdios das laterais de um trapzio e e paralelo `s bases e que seu comprimento a mdia aritmtica dos comprimentos das bases. e a e e e 7.62 Exerc cio. Prove que ligando-se os pontos mdios dos lados de um tringulo qualquer, e a este car dividido em quatro tringulos congruentes. a a 7.63 Exerc cio. Prove que as paralelas aos lados de um tringulo qualquer, traadas passando a c pelos vrtices opostos aos respectivos lados, formam um novo tringulo cujos pontos mdios dos e a e lados so os vrtices do tringulo inicialmente dado. a e a 7.64 Exerc cio. Mostre que ligando-se os pontos mdios dos lados de um quadriltero qualquer e a obtm-se um paralelogramo. e 7.65 Exerc cio. Mostre que se num tringulo qualquer ABC prolongarmos a mediana AM , a relativa ao lado BC, at um ponto D tal que M D = AM , obtemos o quadriltero ABCD, que e a um paralelogramo. e 7.66 Exerc cio. Prove que qualquer tringulo no qual uma mediana e uma bissetriz so coina a cidentes, issceles. e o 7.67 Exerc cio. Mostre que as bissetrizes de dois ngulos opostos de um paralelogramo so a a coincidentes ou paralelas. 7.68 Exerc cio. Mostre que as bissetrizes dos ngulos internos de um paralelogramo interceptama se formando um retngulo. a 7.69 Exerc cio. Mostre que se o paralelogramo do exerc anterior um retngulo, o retngulo cio e a a formado um quadrado. e 7.70 Exerc cio. Em um tringulo issceles ABC, com vrtice em A, toma-se um ponto P a o e sobre a base e traam-se os segmentos P R e P S, paralelos a AB e AC, respectivamente, onde c R AC e S AB. Prove que o per metro do paralelogramo ASP R independente da posio e ca do ponto P sobre a base BC. 7.71 Exerc cio. A partir de cada vrtice de um quadrado ABCD, cujos lados so percorridos e a em um mesmo sentido, marcam-se pontos U, F, S, M , tais que AU = BF = CS = DM. Mostre que o quadriltero U F SM tambm um quadrado. a e e 7.72 Exerc cio. Qual a gura obtida quando ligamos os pontos mdios dos lados de um e retngulo? a 7.73 Exerc cio. Pelo ponto de encontro das diagonais de um quadrado, traam-se dois segmenc tos perpendiculares entre si e limitados pelos lados do quadrado. Mostre que esses segmentos so congruentes. a 7.74 Exerc cio. Mostre que em um trapzio issceles, o ngulo formado pelas bissetrizes de e o a seus ngulos agudos congruente a um de seus ngulos obtusos. a e a

62

Cap tulo 8 Paralelismo e o Teorema do Feixe de Retas Paralelas


Estabelece-se resultados gerais sobre feixe de retas paralelas cortado por retas transversais e demonstra-se o teorema da bissetriz interna.

8.1

Feixe de retas paralelas

A uma coleo de retas paralelas d-se o nome de feixe de retas paralelas. ca a 8.1 Teorema. Sejam r, s, t retas de um feixe de retas paralelas que cortam as retas transversais u e v nos pontos A, B, C e A , B , C , respectivamente. i) Se A B C, ento A B C . a ii) Se AB BC ento A B B C . a Prova: i) Se A B C ento A e C pertencem a semiplanos distintos relativamente ` reta a a s. Uma vez que r e s so retas paralelas e A, A pera A A' r tencem ` r, segue que A, A pertencem a um mesmo a semiplano denido por s. De modo anlogo, cona B B' s clu mos que C e C pertencem a um mesmo semiplano determinado por s. Assim A e C pertencem a semiplanos distintos, relativamente a reta s. Logo, s intercepta o C C' t segmento A C em um unico ponto (!). Como B o ponto de interseo de v com s, e A , C pere ca v u tencem ` v (... e da determinam v), conclu a mos que A C intercepta s exatamente no ponto B . AsFigura 8.1: Feixe de paralelas r, s, t corsim, B pertence ` A C e da A B C . Isto a tadas pelas transversais u, v. prova i). ii) Passando por B , consideremos a unica reta u paralela ` u. a

63

Feixe de retas paralelas


Esta reta intercepta as retas r e t, respectivamente, nos pontos D e E. Como DB BA e B ECB so paralelogramos (!), ento DB a a AB e B E BC. Como, por hiptese, AB o BC, segue que DB B E (!); por outro lado, DB A E B C (!) e B DA B EC (!). Pelo caso ALA de congruncia de tringulos, e a conclu mos que os tringulos A DB e C EB a so congruentes. Daqui decorre A B B C . a Isto prova (ii).
A B A' D B'

r s

C u u'

C' v

Figura 8.2: Prova de (ii).


8.2 Corolrio. Se ai ,(i = 1, 2, ..., k), um feixe de retas paralelas que interceptam duas retas a e transversais u e v nos pontos Ai e Ai , (i = 1, 2, ..., k), respectivamente, e tais que A1 A2 = A2 A3 = ... = Ak1 Ak , ento A1 A2 = A2 A3 = ... = Ak1 Ak . a 8.3 Exerc cio. Prove o corolrio 8.2. a O prximo teorema um resultado bsico para se estabalecer uma teoria de semelhana o e a c de tringulos (prximo cap a o tulo); sua demonstrao fundamenta-se no axioma 10 e no fato do ca corpo dos nmeros reais ser completo. u 8.4 Teorema. (Tales) Se uma reta, paralela a um dos lados de um tringulo, intercepta os a outros dois lados, ento ela os divide na mesma razo. a a Prova: Sejam ABC um tringulo e r uma reta, paralela ao lado BC, que intercepta os a lados AB e AC nos pontos P e Q, respectivamente. (Ver gura 8.3) O teorema arma que: AP AQ = . AB AC

A
(8.1)

r Para mostrar isso, tomemos na semi-reta S(AB) um segmento AX1 de modo que as AB razes AX1 e AX1 no sejam nmeros inteiros o AP a u B C (o caso em que as razes so nmeros inteiros o a u ca como exerc cio para o leitor). Em S(AB) Figura 8.3: Teorema de Tales consideremos os pontos X2 , X3 , ..., Xk, ... tais que AXk = k AX1 para todo k 2. Existem dois nmeros inteiros m e n, m n, tais que Xm P Xm+1 e Xn B Xn+1 , isto , u e
m AX1 < AP < (m + 1) AX1 e n AX1 < AB < (n + 1) AX1 . Da primeira desigualdade em (8.2) segue m AP < . n+1 (n + 1) AX1 (8.4) (8.3) (8.2)

64

Feixe de retas paralelas


Por outro lado, da segunda desigualdade em (8.3), segue que 1 1 < (n + 1) AX1 AB e como AP > 0 temos AP AP < . (n + 1) AX1 AB m AP < . n+1 AB Da primeira desigualdade em (8.3) tem-se 1 1 . < AB n AX1 Assim, AP AP < . AB n AX1 Por outro lado, da segunda desigualdade em (8.2) temos (m + 1) AX1 m+1 AP < = . n AX1 n AX1 n Assim, de (8.6) segue que AP m+1 < . AB n De (8.5) e (8.7) vem que m AP m+1 < < . n+1 AB n (8.8) (8.7) (8.6) (8.5)

Usando isto em (8.4) resulta

Pelos pontos X1, X2 , ..., Xn+1 , tracemos as retas paralelas a BC (que existem e so unicas); a estas retas cortam a semi-reta S(AC) em pontos Y1 , Y2 , ..., Yn+1 (segundo o corolrio 8.2), de a modo que k AY1 = AYk para todo k, 2 k n + 1. Alm disso Ym Q Ym+1 e Yn C Yn+1 . e Assim m AY1 < AQ < (m + 1) AY1 e n AY1 < AC < (n + 1) AY1 . Racioc nio anlogo ao que zemos anteriormente implica que a m AQ m+1 < < . n+1 AC n De (8.8) e (8.9) segue que m m+1 AP AQ m+1 m < < n+1 n AB AC n n+1 isto , e AP AQ m+1 m < . AB AC n n+1 (8.9)

65

Feixe de retas paralelas


Desde que m n, temos que AP m+n+1 AQ 2n + 1 2n + 2 2 < < = . AB AC n(n + 1) n(n + 1) n(n + 1) n Assim, AQ 2 AP < . AB AC n (8.10)

Agora, como o segmento AX1 arbitrrio, podemos tom-lo to pequeno quanto quisermos e a a a 2 de modo que o nmero n possa ser tomado sucientemente grande, resultando n to pequeno u a quanto quisermos. Como o membro esquerdo de (8.10) independe de n, concluimos que o nmero u AQ AQ AP AP AB AC deve ser zero e da AB = AC , provando que vale (8.1). O teorema seguinte uma conseqncia direta do teorema 8.4. e ue 8.5 Teorema. Um feixe de paralelas cortadas por duas transversais determina sobre elas, segmentos correspondentes proporcionais. 8.6 Exerc cio. Demonstre o teorema anterior. 8.7 Observao. Os teoremas 8.4 e 8.5 so conhecidos como Teoremas de Tales. ca a 8.8 Teorema. (Bissetriz Interna) Uma bissetriz interna de um tringulo divide o lado oposto a em segmentos cujos comprimentos so proporcionais aos comprimentos dos lados adjacentes. a Prova: Seja ABC um tringulo e AD a bissetriz do ngulo A. O ponto D tal que B DC a a e e determina sobre o lado BC os segmentos BD e DC. Mostremos que BD = DC . AB AC Na semi-reta BA tomemos o ponto E tal que E B A E e AE = AC. O tringulo EAC issceles a e o com vrtice A e, da ACE = AEC. Por outro lado, e , B AC ngulo externo, donde B AC = ACE +AEC ea e, portanto, B AC = 2AEC. Como AD bissetriz e de B AC, temos que B AC = 2B AD. Destas duas A ultimas relaes segue que AEC = B AD. co Pelo teorema 6.5 temos que AD paralela a EC e e, no BEC, pelo teorema 8.4, temos que BD = BC AB . Deste modo, BE AB BD + DC AB + AE BD = = BD + DC AB + AE BD AB Logo, como
DC BD

AE AB

e AE = AC, segue-se que terna

Figura 8.4: Teorema da Bissetriz In-

BD DC = . AB AC

66

Exerc cios complementares

8.2

Exerc cios complementares

8.9 Exerc cio. Enuncie e demonstre o rec proco do Teorema da Bissetriz Interna. 8.10 Exerc cio. Usando o resultado do corolrio 8.2, crie um mtodo, usando rgua e compasso, a e e para dividir um segmento de reta em um nmero dado de parte iguais. u 8.11 Exerc cio. Mostre que em qualquer tringulo retngulo, a medida da mediana relativa ` a a a hipotenusa igual ` metade da medida da hipotenusa. e a 8.12 Exerc cio. Num paralelogramo ABCD traa-se uma paralela ` diagonal AC que corta c a AB no ponto E e BC no ponto F . Dos pontos E e F, traam-se as paralelas a BD que cortam c AD no ponto H e CD no ponto G, respectivamente. Mostre que AH CD = AD CG.

67

Exerc cios complementares

68

Cap tulo 9 Tringulos Semelhantes e a Semelhana de Pol c gonos


Dene-se semelhana de tringulos e de pol c a gonos, estabelecendo-se condies sucientes que garantam semelco hana; como consequncia, obtm-se o famoso Teorema de c e e Pitgoras e seu rec a proco. A semelhana de pol c gonos e apresentada na forma de exerc cios no nal do cap tulo.

9.1

Denies e consideraes gerais co co

9.1 Denio. Dois tringulos so semelhantes se existe uma bijeo (chamada semelhanca a a ca c a) entre seus vrtices, de modo que ngulos correspondentes so congruentes e lados correspone a a dentes so proporcionais. a A denio acima tem o seguinte signicado: dados dois tringulos, ABC e A B C , seja ca a : {A, B, C} {A , B , C } tal que A = (A) , B = (B) , C = (C) ; se A = A , B = AB AC BC B , C = C e A B = A C = B C , os tringulos ABC e A B C so semelhantes. a a Quando dois tringulos so semelhantes os ngulos correspondentes so ditos homloa a a a o gos, assim como os lados correspondentes so chamados lados homlogos. a o A razo entre as medidas de dois lados homlogos chamado razo de semelhana entre a o e a c os dois tringulos. a Se os tringulos ABC e A B C so semelhantes, indicaremos esse fato usando a notao a a ca ABC A B C . Assim A = A ,B = B ,C = C ABC A B C AB AC BC AB = AC = BC . 9.2 Exerc cio. Mostre que a relao satisfaz as seguintes propriedades: ca i) ABC ABC (reexiva), ii) ABC A B C A B C ABC (simtrica), e iii) ABC A B C e A B C A B C ABC A B C (transitiva). Assim, uma relao de equivalncia. e ca e

69

Casos de semelhana de tringulos e o Teorema de Pitgoras c a a


9.3 Exerc cio. Mostre que: a) Congruncia de tringulos um caso particular de semelhana. Neste caso, qual a razo e a e c e a de semelhana? c b) Se dois tringulos so semelhantes, com razo de semelhana igual a um, ento eles so a a a c a a congruentes. 9.4 Teorema. Toda reta paralela a um dos lados de um tringulo intercepta os outros dois lados a (ou o prolongamento deles), determinando um novo tringulo semelhante ao primeiro (considere a todas as situaes poss co veis). 9.5 Exerc cio. Demonstre o teorema anterior.

9.2

Casos de semelhana de tringulos e o Teorema c a de Pitgoras a

Neste tem, veremos que para vericar se dois tringulos so semelhantes basta que algumas das a a relaes entre ngulos e/ou entre lados estejam satisfeitas. co a 9.6 Teorema. (Primeiro Caso de Semelhana) Dois tringulos ABC e A B C so semelc a a hantes se A = A e B = B . Prova: Consideremos a correspondncia e : {A, B, C} {A , B , C }, denida por A = (A), B = (B), C = (C). Por hiptese, temos que A = A e B = B ; o como em qualquer tringulo a soma das medidas dos ngulos internos 180 , como, C = a a e A + B e C = 180 A + B , segue que C = C . Mostremos agora que os lados 180 correspondentes so proporcionais. a

C C'

A'

B'

Figura 9.1: Primeiro caso de semelhana de tringulos. c a

Para isso, consideremos a semi-reta S(A B ) e sobre ela um ponto X tal que A X = AB (transporte de segmento). Pelo ponto X, consideremos a reta paralela ao lado B C . Esta paralela corta a semi-reta S(A C ) num ponto Y (!). Fica assim determinado o tringulo A XY a congruente ao tringulo ABC, uma vez que A = A , AB = A X e A XY = B = B (!). Pelo a

70

Casos de semelhana de tringulos e o Teorema de Pitgoras c a a


AX AY teorema 8.4 segue que A B = A C . Como A X = AB e A Y = AC, temos AC BC modo anlogo, prova-se que A C = B C . a AB AB

AC AC

. De

9.7 Exerc cio. Conclua a prova do teorema anterior mostrando que

AC AC

BC . B C

9.8 Exerc cio. Mostre que: a) Dois tringulos issceles que tm ngulos do vrtice congruentes so semelhantes. a o e a e a b) Dois tringulos issceles que tm ngulos da base congruentes so semelhantes. a o e a a 9.9 Exerc cio. Mostre que se dois tringulos tm os seus lados dois a dois paralelos ou perpena e diculares, ento eles so semelhantes. a a 9.10 Exerc cio. Mostre que dado um tringulo, sempre poss a e vel, usando rgua (no gradue a ada) e compasso, construir outro tringulo semelhante a ele (conclua que poss a e vel construir uma innidade de tringulos semelhantes ao tringulo dado). a a 9.11 Teorema. (Segundo Caso de Semelhana) Dois tringulos ABC e A B C so semelc a a AB AC hantes se A = A e A B = A C . Prova: Consideremos a mesma correspondncia entre vrtices usada no teorema 9.6. e e

C C'
||
||

A'

B'

Figura 9.2: Segundo caso de semelhana de tringulos. c a

Construamos um tringulo XY Z tal que XY = A B , X = A e Y = B (conforme o exerc a cio 9.10). Pelo teorema 9.6, os tringulos XY Z e ABC so semelhantes; logo, a a AB AC = . XY XZ Como XY = A B (por construo) e ca
AB AB

(9.1) (por hiptese), temos que o (9.2)

AC AC

AB AC = . XY AC

Assim, de (9.1) e (9.2), vem que XZ = A C . Como XY = A B e X = A = A (!), segue, pelo Primeiro Caso de Congruncia de Tringulos, que A B C e XY Z so congruentes. Assim, e a a XY Z A B C . Como sabemos que ABC XY Z, segue que ABC A B C . 9.12 Exerc cio. Enuncie e demonstre os rec procos dos teoremas 8.4 e 9.4.

71

Casos de semelhana de tringulos e o Teorema de Pitgoras c a a


C C'
||
||

A'

Figura 9.3: Terceiro caso de semelhana de tringulos. c a

9.13 Teorema. (Terceiro Caso de Semelhana) Dois tringulos ABC e A B C so semelc a a AB AC BC hantes se A B = A C = B C . Prova: Consideremos a mesma correspondncia entre os vrtices usada no teorema anterior. e e Construamos um tringulo XY Z tal que XY = A B , X = A e XZ = A C (mostre que esta a AB AC AB AC construo poss ca e vel). Como, por hiptese, A B = A C , segue que XY = XZ . Logo, pelo o AB teorema 9.11, os tringulos ABC e XY Z so semelhantes. Da decorre que XY = BC . Como a a YZ AB BC BC por hiptese, A B = B C (e XY = A B ) temos que B C = BC , logo = B C . o YZ Assim, XY = A B , XZ = A C (por construo) e Y Z = B C implicam (pelo Terceiro Caso ca de Congruncia de Tringulos) que XY Z A B C e da XY Z A B C . Como ABC XY Z e a (por construo, lembra?), segue que (transitividade) ABC A B C . ca O exerc cio seguinte nos d trs condies sucientes para que dois tringulos retngulos a e co a a sejam semelhantes. 9.14 Exerc cio. Verique que: a) Dois tringulos retngulos so semelhantes quando um ngulo agudo de um congruente a a a a e ao ngulo agudo, correspondente, do outro. a b) Dois tringulos retngulos so semelhantes quando tm os catetos correspondentes proa a a e porcionais. c) Dois tringulos retngulos so semelhantes, quando um deles tm a hipotenusa e um dos a a a e catetos proporcionais a hipotenusa e ao cateto correspondentes, do outro. 9.15 Exerc cio. Escreva, usando palavras (sem notao simblica), os enunciados dos teoremas ca o sobre semelhana de tringulos. c a 9.16 Teorema. Em qualquer tringulo retngulo, a medida da altura relativa ` hipotenusa a a a e a mdia geomtrica entre as medidas das projees dos catetos sobre a hipotenusa. e e co Prova: Consideremos um tringulo ABC, retngulo em A. Tracemos a altura AD relativamente a a ao lado BC (hipotenusa). O ponto D est entre B e C (!) e dene os segmentos BD e DC. a Para efeito de simplicao, vamos xar a seguinte notao: BC = a, AB = c, AC = b, AD = ca ca h, BD = m, DC = n, (vide gura ??). Os tringulos ADB e ADC so retngulos em D. Como a a a e B + C = 90 , segue que C AD = B. Analogamente, temos que B AD = C. C AD + C = 90 Assim, os tringulos ADB e CDA so semelhantes entre si e semelhantes ao tringulo CAB. a a a

72

||

|||

B'

Casos de semelhana de tringulos e o Teorema de Pitgoras c a a


Usando a denio de semelhana, podemos escrever vrias relaes entre as medidas a, b, c, m, n ca c a co e h. Assim c h m ADB CDA = = . b n h

A c m B
|

h n a

C
|

Figura 9.4: Tringulos retngulo ABC. a a

9.17 Teorema. (Pitgoras) Em todo tringulo retngulo, o quadrado do comprimento da a a a hipotenusa igual ` soma dos quadrados dos comprimentos dos catetos. e a Prova: Usando a notao estabelecida, temos que mostrar que a2 = b2 + c2 . De fato, j ca a sabemos que os tringulos ADB, CDA, CAB so semelhantes. De ADB CAB, segue que a a c m a = c ; logo am = c2 . (9.3) De CDA CAB, segue que
b a

m b;

logo, an = b2 . (9.4)

a2

Assim, de (9.3) e (9.4), temos: a(m + n) = c2 + b2 . Desde que m + n = a, temos, nalmente, = b2 + c2 .

9.18 Teorema. (Rec proco do Teorema de Pitgoras) Se um tringulo possui lados medindo a a a, b, c e se a2 = b2 + c2 , ento esse tringulo retngulo e a hipotenusa o lado com medida a. a a e a e Prova: Seja ABC um tringulo com lados que medem a, b, c e so tais que a2 = b2 + a a Construamos um tringulo retngulo com catetos a a medindo b e c. Nesse tringulo, pelo a teorema de Pitgoras, temos que sua hipotenusa mede b2 + c2 , que igual a a (por hiptese). a e o Assim, este novo tringulo (que retngulo) tem lados medindo a, b, c. Pelo Terceiro Caso a e a de Congruncia de Tringulos, segue que ele congruente ao tringulo ABC. Logo, ABC e a e a e tringulo retngulo e sua hipotenusa mede a. a a c2 .

73

Exerc cios complementares

9.3

Exerc cios complementares

9.19 Exerc cio. Demonstre os trs casos de semelhana de tringulos, utilizando o teorema e c a 9.4. 9.20 Exerc cio. Dois pol gonos convexos, A1 A2 ...An e B1 B2 ...Bn so semelhantes se existe uma a correspondncia bijetiva (chamada semelhana) entre seus vrtices, Ai Bi , i = 1, 2, ..., n tal e c e que Ai = Bi , i = 1, 2, ..., n A1 A2 A2 A3 An1 An An A1 = = ... = = = k. B1 B2 B2 B3 Bn1 Bn Bn B1 O nmero k a razo de semelhana entre os dois pol u e a c gonos. Dois pol gonos semelhantes, cuja razo de semelhana igual a 1, so ditos congruentes (note que, pela denio, dois a c e a ca pol gonos so congruentes quando possuem todos os lados e todos os ngulos, respectivamente, a a congruentes). a) Mostre que dois pol gonos semelhantes podem ser decompostos no mesmo nmero de u tringulos ordenadamente semelhantes. a b) Dois pol gonos compostos por um mesmo nmero de tringulos ordenadamente semelu a hantes so semelhantes. a 9.21 Exerc cio. Mostre que a razo entre os per a metros de dois pol gonos semelhantes igual e a ` razo de semelhana. a c 9.22 Exerc cio. Chamam-se pontos homlogos de dois pol o gonos semelhantes os pares de pontos P e P tais que, ligando-se P a dois vrtices quaisquer A, B do primeiro pol e gono e, P aos correspondentes A , B do segundo pol gono, os tringulos P AB e P A B so semela a hantes.Segmentos homlogos so aqueles que tm como extremos pares de pontos homlogos. o a e o Mostre que em dois pol gonos semelhantes a relao de comprimento de segmentos homlogos ca o igual ` razo de semelhana. e a a c 9.23 Exerc cio. Demonstre que em dois tringulos semelhantes, a a) a razo entre os comprimentos das bissetrizes de ngulos correspondentes igual ` razo a a e a a de semelhana; c b) a razo entre os comprimentos das medianas relativas a lados homlogos igual ` razo a o e a a de semelhana; c c) a razo entre os comprimentos das alturas relativas a lados homlogos igual ` razo de a o e a a semelhana. c 9.24 Exerc cio. Determine o comprimento do lado do quadrado inscrito num tringulo ABC, a cuja base BC mede l e a altura AD, relativa ` base BC, mede h.(suponha um lado do quadrado a paralelo ` base BC) a 9.25 Exerc cio. Dado um tringulo ABC, calcule a que distncia x que o vrtice B deve estar a a e de um ponto P quando os segmentos P Q (P Q = r) e P R (P R = s) paralelos aos lados BC (BC = a) e AC (AC = b), respectivamente, satisfazem a condio r + s = p + q.(gura 9.5). ca

74

Exerc cios complementares


A q P x B s R C r p Q

Figura 9.5: Tringulo ABC. a

9.26 Exerc cio. Sejam p, q inteiros positivos tais que p > q. Mostre que todo tringulo cujos a lados medem p2 q 2 , 2pq e p2 + q 2 , um tringulo retngulo. e a a 9.27 Exerc cio. Demonstre que, num paralelogramo, as distncias de um ponto da diagonal a aos dois lados adjacentes a ela so inversamente proporcionais aos comprimentos desses lados. a 9.28 Exerc cio. Os lados de um tringulo ABC medem AB = c, AC = b e BC = a. Por um a ponto D , sobre o lado AB , traa-se a paralela ao lado BC , formando um trapzio BDEC , c e onde E um ponto de AC . Se o per e metro do trapzio 2p , ache o per e e metro do tringulo a ADE . 9.29 Exerc cio. Mostre que em todo tringulo ABC, a medida ha , da altura relativa ao vrtice a e 2 A dada por ha = e p (p a) (p b) (p c), sendo a = BC, b = AC, c = AB e p o semi a per metro do tringulo (sugesto: considere a altura AD e aplique o teorema de Pitgoras aos a a a dois tringulos retngulos formados). a a 9.30 Exerc cio. Um tringulo issceles cuja razo entre a base e uma lateral o nmero ureo, a o a e u a e chamado de tringulo ureo (analogamente, dene-se retngulo ureo, elipse urea, etc.). Calcule a a a a a os ngulos de um tringulo ureo. Mostre que todos os tringulos ureos so semelhantes entre a a a a a a si.

75

Exerc cios complementares

76

Cap tulo 10 Circunferncia e


Relacionam-se os fatos bsicos relativos ` geometria na cira a cunferncia, procurando-se enfatizar as relaes entre ngulos e co a e arcos. Resultados relacionados a guras inscritas e circunscritas so tambm abordados. a e

10.1

Elementos da circunferncia e

Recapitulamos, aqui, a denio 3.12, que introduz o conceito de circunferncia. ca e Sejam O um ponto do plano e r um nmero real positivo. Circunferncia de centro O u e e raio r o conjunto de pontos P tais que OP = r. e 10.1 Observao. Para efeito de notao, indicaremos uma circunferncia de centro O e raio ca ca e r por C(O, r). Muitas vezes, tambm chamaremos de raio um segmento determinado por O e e um ponto qualquer da circunferncia. e 10.2 Denio. Corda um segmento cujas extremidades so dois pontos de uma circunca e a ferncia. Dimetro uma corda que passa pelo centro. e a e 10.3 Observao. A medida de um dimetro de uma circunferncia de raio r 2r (muitas ca a e e vezes, chamaremos 2r de dimetro da circunferncia!). a e 10.4 Denio. Uma secante a uma circunferncia uma reta que intercepta a circunferncia ca e e e em dois pontos distintos. 10.5 Teorema. Seja s secante a uma circunferncia C(O, r) nos pontos A e B (onde AB e no um dimetro). Ento um raio intercepta AB em seu ponto mdio se, e somente se, a e a a e e perpendicular ` s (ou ` corda AB). a a Prova. Suponha que um raio intercepte AB em seu ponto mdio, M. Os pontos A, M, O e e os pontos B, M, O determinam os tringulos AM O e BM O, que so congruentes, pois AO = a a BO, AM = M B e M O lado comum. Assim, AM O = B M O. Como AM O e B M O so suplee a ; logo, a reta determinada por O e M perpendicular mentares segue que AM O = B M O = 90 e a ` reta determinada por A e B, que a secante considerada. e

77

Angulos e arcos numa circunferncia e


10.6 Exerc cio. Complete a demonstrao do teorema 10.5. ca 10.7 Denio. Uma tangente a uma circunferncia uma reta que tem um unico ponto em ca e e comum com a circunferncia. O ponto comum a uma tangente e a uma circunferncia chamado e e e ponto de tangncia. e 10.8 Denio. Toda semi-reta que tem apenas um ponto em comum com uma c ca rcunferncia e dita uma semi-reta tangente ` circunferncia. e a e 10.9 Exerc cio. Prove que os pontos de uma tangente a uma circunferncia, distintos do ponto e de tangncia, so pontos exteriores ` circunferncia. (Vide Def. 3.13) e a a e 10.10 Teorema. Uma reta tangente a uma circunferncia se, e somente se, ela perpendicular e e e a um raio em sua extremidade que no o centro. a Prova: Seja t a tangente a uma circunferncia C(O,r) e T o ponto de tangncia. e e Suponhamos que t no seja perpendicular ao raio OT . a Chamemos de P o p da perpendicular baixada do ponto e t O ` t. Temos assim denido o OP T , retngulo em P . a a T Escolha um ponto T = T de t tal que P T = P T . Temos constru o OP T , congruente a OP T (!); logo, do P OT = OT . Assim, T pertence ` circunferncia e outro a e e T' ponto de t distinto de T . Isto signica que t no tana e gente ` C(O, r), o que contradiz a hiptese. Logo, t a o e O perpendicular ao raio OT . Seja OT um raio de uma circunferncia e t uma reta e perpendicular a OT em T . Mostremos que t tangente e a ` circunferncia em T , isto , que t no tem outro ponto e e a distinto de T , em comum com C(O, r). Seja P qualquer outro ponto de t, P = T ; ca determinado o tringulo a Figura 10.1: Teorema 10.10. OT P, retngulo em T . Logo, OP > OT (!). Portanto, P a est no exterior da circunferncia. Assim, T o unico ponto comum ` t e ` C(O, r), e da t a e e a a e tangente a C(O, r) em T .

10.11 Exerc cio. Mostre que se P um ponto exterior a uma circunferncia, os segmentos das e e tangentes traadas por P so congruentes. c a 10.12 Exerc cio. Mostre que uma circunferncia est contida em um dos semiplanos determie a nados por cada uma de suas tangentes.

10.2

Angulos e arcos numa circunferncia e

10.13 Denio. Numa circunferncia C(O, r) tomemos dois pontos A, B e consideremos o ca e a ngulo AOB. Esse ngulo chamado ngulo central. O subconjunto de C(O, r) obtido pela a e a interseo de C(O, r) com o interior de AOB unido com {A, B}, um arco determinado pelos ca e pontos A e B. Esse arco indicaremos por AB. Se AOB um ngulo raso ao arco AB chamaremos e a de semicircunferncia. Ao subconjunto (C(O, r) AB) {A, B}, chamaremos arco maior. e

78

Angulos e arcos numa circunferncia e


10.14 Observao. A reta determinada por A e B separa o plano em dois semiplanos. Quando ca AB no um dimetro, o centro da circunferncia est contido no mesmo semiplano que contm a e a e a e o arco maior. Os raios denidos pelo centro O da circunferncia e pontos do arco AB interceptam e a a corda AB; aqueles que ligam O aos pontos do arco maior no interceptam a corda AB. A cada ngulo central AOB ca associado um unico arco AB na circunferncia, e reciproa e camente. Isto nos permite dar a seguinte denio para medida de arco. ca 10.15 Denio. A medida do arco AB a medida em graus do ngulo central AOB. A meca e a dida do arco maior 360 menos a medida em graus do arco AB. Caso AB seja um dimetro, e a os dois arcos determinados por AOB medem 180 . A medida de um arco AB indicaremos por m(AB). Uma vez que existe uma correspondncia biun e voca entre ngulos centrais e arcos numa a circunferncia, podemos falar em adio de arcos e comparao de arcos. Assim, numa mesma e ca ca circunferncia, temos as seguintes denies: e co 10.16 Denio. 1) AB A B AOB A OB (congruncia de arcos) ca e 2) AB > A B AOB > A OB (comparao de arcos) ca 3) AB = AC + CB AOB = AOC + C OB (adio de arcos) ca 10.17 Exerc cio. Mostre que em circunferncias de mesmo raio, ngulos centrais congruentes e a determinam cordas congruentes, e reciprocamente. 10.18 Exerc cio. Mostre que em circunferncias de mesmo raio, cordas congruentes determie nam arcos congruentes e arcos maiores de mesma medida. 10.19 Denio. Um ngulo dito inscrito numa circunferncia se seu vrtice um ponto ca a e e e e da circunferncia e seus lados interceptam a circunferncia em dois pontos distintos do vrtice. e e e O arco determinado pelos dois pontos distintos e que no contm o vrtice do ngulo inscrito a e e a dito arco subentendido pelo ngulo ou que o ngulo subentende o arco. e a a 10.20 Teorema. A medida de um ngulo inscrito numa circunferncia igual a metade da a e e medida do arco subentendido por este ngulo. a Prova. A prova ser feita considerando trs casos particulares, a saber: a e (i) um dos lados do ngulo inscrito um dimetro, a e a (ii) o ngulo inscrito dividido pelo dimetro com extremidade em seu vrtice, a e a e (iii) o ngulo inscrito no dividido pelo dimetro com extremidade em seu vrtice. a a e a e Caso (i). Seja B AC o ngulo inscrito com vrtice em A. Suponhamos que AB o dimetro, a e e a isto , O AB. Assim, m(BC) = B OC. Como CO =AO, o tringulo AOC issceles com e a e o vrtice em O e da OAC = OCA. Como B OC ngulo externo ao tringulo AOC, temos que e ea a 1 B OC = OAC + OCA = 2 OAC = 2 B AC. Logo: B AC = 2 m(BC). Provamos assim o teorema no caso (i).

79

Angulos e arcos numa circunferncia e


C C B A C

B D

. O

D A B

Figura 10.2: Teorema 10.20.

Suponhamos agora que nenhum dos lados do ngulo inscrito contenha um dimetro. Seja a a AD o dimetro com extremidade no vrtice A do ngulo inscrito. Pelo caso (i) temos que a e a 1 1 B OD = B AD e DOC = DAC. 2 2 (10.1)

Temos nesta situao que considerar os casos (ii) e (iii). ca Caso (ii): AD divide o ngulo inscrito. Neste caso, B AC = B AD + DAC. Daqui e de a (10.1), segue que 1 1 1 1 B AC = B OD + DOC = (B OD + DOC) = B OC 2 2 2 2 e, da B AC = 1 m(BC), provando o caso (ii). , 2 Caso (iii): AD no divide o ngulo inscrito. No caso em que AB divide o ngulo DAC a a a 1 1 temos que DAC = DAB + B AC. Daqui e de (10.1), segue que 2 DOC = 2 B OD + B AC. Como DOC = DOB + B OC, obtemos 1 1 DOB + B OC = B OD + B AC, 2 2 logo, 1 1 B AC = B OC = m(BC). 2 2 O caso em que AC que divide o ngulo B AD pode ser tratado de forma anloga e deixada e a a e para o leitor como exerc cio. 10.21 Corolrio. Angulos inscritos que subentendem um mesmo arco so congruentes. a a Prova. Basta observar que a cada ngulo inscrito nessa situao est associado o mesmo a ca a a ngulo central e aplicar o teorema anterior. 10.22 Corolrio. Todos os ngulos inscritos que subentendem uma semicircunferncia so rea a e a tos. Prova. Aplicao direta do corolrio acima. ca a

80

Angulos e arcos numa circunferncia e

C A' A

C' C''

A B

F B

Figura 10.3: Corolrios 10.21 e 10.22. a

10.23 Exerc cio. Para qualquer tringulo retngulo existe uma unica circunferncia que passa a a e pelos trs vrtices do tringulo (dizemos que todo tringulo retngulo inscrit e e a a a e vel numa circunferncia). e 10.24 Observao. Veremos mais adiante (teorema 10.38) que todo tringulo inscrit ca a e vel numa circunferncia. e 10.25 Denio. Angulo semi-inscrito relativo a uma circunferncia um ngulo que tem ca e e a vrtice na circunferncia, um lado secante e o outro lado tangente ` circunferncia. e e a e Sejam A, B pontos distintos de C(O, r) e t uma reta tangente ` C(O, r) em A. Seja C um a ponto de t distinto de A; o ngulo com vrtice A e lados denidos pelas semi-retas AB e AC a e e um ngulo semi-inscrito e o denotaremos por C AB. O arco AB que tem um ponto no interior a de C AB chamado arco correspondente ao ngulo C AB. e a 10.26 Teorema. A medida de um ngulo semi-inscrito igual ` metade da medida do seu arco a e a correspondente. Prova. Vamos dividir a demonstrao em trs casos: ca e (i) C AB agudo; e (ii) C AB reto; e (iii) C AB obtuso. e Caso (i) : C AB agudo. O tringulo AOB issceles e B AO + AOB + OBA = 180 . Como e a e o AOB e, ento, B AO = OBA, segue que 2 B AO = 180 a 1 B AO = 90 AOB. 2 Sendo t tangente ` circunferncia em A, temos que C AB + B AO = 90 ou a e B AO = 90 C AB. (10.3) (10.2)

81

Angulos e arcos numa circunferncia e


De (10.2) e (10.3), vem que

C .

1 1 C AB = AOB = m AB 2 2 e o caso (i) est provado. a Caso (ii): C AB reto. Neste caso, AB dimetro e e e a

F
B

m(AB) = 180 . Isto prova (ii). Caso (iii): C AB obtuso. Como o suplemento de e tAB agudo, aplicando a este o caso (i), conclu e mos a prova.

Figura 10.4: Teorema 10.26.

10.27 Denio. Consideremos um arco AB contido numa circunferncia C(O, r). O arco ca e complementar de AB relativamente ` C(O, r) o conjunto de pontos {C(O, r) AB}. a e 10.28 Teorema. Os vrtices dos ngulos inscritos (ou semi-inscritos) a uma circunferncia e a e C(O, r) que tem lados passando por dois pontos A, B C(O, r) e medem , = esto no arco complementar de relativamente ` C(O, r). a a
1 2

m(AB),

10.29 Denio. O arco constru acima chamado arco capaz de sobre o segmento ca do e AB. 10.30 Observao. Todos os pontos deste arco enxergam o segmento AB segundo um ngulo ca a de medida . 10.31 Exerc cio. Mostre que dado um segmento AB e um ngulo de medida , existe um arco a capaz de sobre o segmento AB. Esse arco capaz unico? e 10.32 Teorema. Se AB e CD so cordas distintas de uma mesma circunferncia que interceptama e se num ponto P , ento AP P B = CP P D. a Prova. Os tringulos AP D e CP B so semelhantes, pois DAP = B CP, P DA = C BP e a a AP D = C P B (qual a semelhana?). e c PB D Logo CP = P D e, da AP P B = CP P D. , AP 10.33 Exerc cio. Mostre que se A1 A2 , A3 A4 , ..., An1 An uma sequncia de cordas de uma mesma circunferncia e e e e elas interceptam-se num unico ponto P , ento A1 P P A2 = a A3 P P A4 = ... = An1 P P An . O produto An1 P P An e chamado potncia de P em relao ` circunferncia. e ca a e

.
A C

10.34 Exerc cio. Se por um ponto P , externo a uma circunferncia, passamos duas retas que interceptam a cire cunferncia nos pontos A, B, C, D, respectivamente, ento e a P A P B = P C P D.

Figura 10.5: Teorema 10.32.

82

Angulos e arcos numa circunferncia e


10.35 Denio. Um pol ca gono inscrit e vel se existe uma circunferncia que contenha todos e os seus vrtices. Neste caso, a circunferncia dita circunscrita ao pol e e e gono e o pol gono dito e estar inscrito na circunferncia. e 10.36 Lema. Se ABC um tringulo qualquer e m, n so retas perpendiculares a AB e AC, e a a respectivamenteento m e n se interceptam num ponto. a 10.37 Exerc cio. Prove o lema acima. 10.38 Teorema. Todo tringulo inscrit numa unica circunferncia. a e vel e Prova. Existncia da circunferncia circunscrita. Seja ABC um tringulo qualquer. e e a Para provar que o ABC inscrit e vel numa circunferncia, basta exibir um ponto (que ser o centro e a A da circunferncia) eqidistante de A, B e C. Para isso, e u t tomemos os pontos mdios M, N de AB, AC, respectie M F vamente, e sua respectivas mediatrizes t, s. Do exerc cio P 5.27, segue o ponto P , interseo de t e s, eqidistante ca e u C B de A, B e C, logo, a circunferncia C (P, r) com r = P A e N circunscreve o tringulo. Isto prova a existncia. a e Unicidade. Seja C (O, r ) outra circunferncia cire s cunscrita ao ABC. Seu raio que passa por M pere pendicular a AB, conforme o teorema 10.5, logo, este raio mediatriz de AB e, ento, O pertence ` mediae a a Figura 10.6: Teorema 10.38. triz de AB. Mas, pelo mesmo motivo, o raio de C (O, r ) que passsa por N a mediatriz de AC, de modo que O pertence ` mediatriz de AC. Acabamos e a de mostrar que O o ponto de interseo das mediatrizes de AB e AC. Desde que P , por e ca e denio, o ponto comum das mediatrizes de AB e AC, segue que O = P. Note que ca

r = OA = P A = r, sendo a primeira e ultima igualdades vlidas por C (O, r ) e C (P, r) serem circunferncias cir a e cunscritas ao ABC (a primeira por hiptese e a segunda por construo). Segue que as o ca circunferncias C (O, r ) e C (P, r) possuem mesmo centro e mesmo raio e, portanto, so iguais. e a

10.39 Exerc cio. Diga onde, na prova do teorema acima, foi usado o lema 10.36. Como consequncia imediata do teorema anterior temos o seguinte corolrio: e a 10.40 Corolrio. Em qualquer tringulo, as mediatrizes dos seus lados interceptam-se num a a unico ponto. 10.41 Corolrio. Trs pontos no colineares determinam uma unica circunferncia. a e a e 10.42 Denio. O ponto comum `s trs mediatrizes de um tringulo chamado circuncenca a e a e tro do tringulo (centro da circunferncia circunscrita). a e

83

Angulos e arcos numa circunferncia e

A B
F

Pelo que vimos no teorema 10.38, os tringulos tm a proa e priedade de estarem inscritos numa circunferncia. Isto no e a e verdadeiro para um pol gono qualquer. S em situaes bem o co particulares pol gonos estaro inscritos numa circunferncia. As a e proposies que seguem esclarecem esse fato. co 10.43 Teorema. Todo quadriltero inscrit a vel possui um par de ngulos opostos suplementares, e reciprocamente. a

Prova. a) Suponhamos que ABCD seja um quadriltero a Figura 10.7: Quadriltero a que est inscrito numa circunferncia. Da segue que cada um a e inscrito. de seus ngulos est inscrito na circunferncia, logo, os ngulos a a e a A e C subentendem arcos determinados pelos pontos B e D (arcos complementares). Como a medida desses dois arcos soma 360 , de acordo com o teorema 10.20, A + C = 180 . Assim, A e C so suplementares. a b) Seja ABCD um quadriltero que tem um par de ngulos opostos suplementares. Como a a a soma dos ngulos internos de um quadriltero 360 , segue que o outro par tambm constituia a e e se de ngulos opostos suplementares. Consideremos a circunferncia determinada pelos pontos a e A, B, C (vide observao 10.41). Para o outro vrtice, D, do quadriltero s temos trs alternaca e a o e tivas: A (i) D pertence ao interior da circunferncia; e (ii) D pertence ao exterior da circunferncia; e B (iii) D pertence ` circunferncia. a e F Suponhamos que vale (i). Neste caso, tracemos o segD mento BD, e seja E o ponto de interseo da semi-reta BD ca com a circunferncia. Obtemos, assim, um quadriltero e a C ABCE que est inscrito na circunferncia. Da parte a) a e E do teorema, segue que seus ngulos opostos so suplea a mentares. Em particular, temos: Figura 10.8: Teorema 10.43. ABC + AEC = 180 . Por hiptese, temos que o ABC + ADC = 180 . Assim, de (10.4) e (10.5), segue que ADC = AEC. Observemos, no entanto, que aplicando o teorema do ngulo externo aos a temos, respectivamente ADB > AEB e C DB > C EB. Assim, AEC = AEB + B EC < ADB + B DC = ADC. Isto entra em contradio com (10.6). Assim, D no pertence ao interior da circunferncia. ca a e Se supusermos (ii), isto , D pertence ao exterior da circunferncia, racioc e e nio anlogo a tambm nos levar a uma contradio. Assim, s resta a situao (iii), isto , D pertence ` e a ca o ca e a circunferncia e, da o quadriltero ABCD est inscrito na circunferncia. e , a a e AED e (10.6) CDE (10.5) (10.4)

84

Angulos e arcos numa circunferncia e


10.44 Exerc cio. Complete a prova do teorema 10.43 mostrando que (ii) tambm leva a uma e contradio. ca 10.45 Teorema. Todo pol gono regular inscrit e vel. Prova. Seja A1 A2 ...An um pol gono regular. Consideremos a circunferncia determinada e pelos pontos A1 A2 A3 e seja O o seu centro. O A2 OA3 issceles com vrtice O pois OA2 = e o e OA3 , da OA2 A3 = OA3 A2 . , Como o pol gono regular todos os seus ngulos internos so congruentes, logo A1 A2 A3 e a a = A2 A3 A4 . Assim, A1 A2 O = OA3 A4 (!) e, como A1 A2 = A3 A4 e OA2 = OA3 , pelo caso LAL os tringulos OA1 A2 e OA3 A4 so congruentes. Segue da que OA4 = OA1 . Assim, A4 pertence a a a ` circunferncia considerada. De modo anlogo pode-se provar que A5 tambm pertence a essa e a e circunferncia. Como o nmero de vrtices do pol e u e gono nito, atravs de uma sequncia de e e e racioc nios anlogos ao anterior, mostra-se que todos os vrtices pertencem ` circunferncia. a e a e 10.46 Exerc cio. A circunferncia que circunscreve um pol e gono, quando existe, unica. e 10.47 Denio. Uma circunferncia em que um pol ca e gono tem todos os seus lados tangentes dita estar inscrita no pol e gono. Neste caso, dizemos que o pol gono circunscrit e vel e que o pol gono circunscreve a circunferncia. e 10.48 Teorema. Todo tringulo circunscrit numa unica circunferncia. a e vel e Prova. Mostremos, inicialmente, a existncia da circunferncia inscrita. e e Seja ABC um tringulo; consideremos as bisa A setrizes bA e bB dos ngulos A e B. Essas bisa setrizes interceptam-se num ponto P . Por esse ponto, tomemos as retas perpendiculares aos lados F AB, AC, BC. Sejam, respectivamente, E, F e G esses pontos. Se mostrarmos que P E = P F = P G, E o ponto P ser centro de uma circunferncia que a e passa pelos pontos E, F, G. Os lados AB, AC e BC sero tangentes a essa circunferncia nos pona e tos E, F e G, pois sero perpendiculares, respectia vamente, aos raios P E, P F e P G. Logo, a circunB G C ferncia constru estar inscrita no tringulo. e da a a Prova de que P E = P F = P G. ConsidereFigura 10.9: Teorema 10.48. mos os tringulos P F A e P EA. Eles so tringulos a a a retngulos, com P A lado comum e P AF = P AE. Assim, P F A P EA e, da P E = P F. De a , modo anlogo nos tringulos P EB e P GB, que so tringulos retngulos, temos P BE = P BG a a a a a e PB lado comum; logo P EB P GB, e, da P E = P G. Isto conclui a prova da existncia. , e Provemos agora a unicidade. Seja C (O, r) circunferncia inscrita em ABC, e E , F , G e os ps das perpendiculares a AB, AC, BC baixadas por O, respectivamente. Os tringulos e a retngulos OAE e OAF so congruentes, pois possuem hipotenusa comum e um par de catetos a a congruentes. Da segue que OA bissetriz de A. Analogamente mostra-se que OAE OAG , , e de modo que OB a bissetriz de B. O ponto O a interseo entre as bissetrizes de A e e e ca B, como P , por denio, o ponto comum a tais bissetrizes, segue que O = P. Desde que a e ca

85

Trigonometria
perpendicular a uma reta passando por um ponto unica, E = E, F = F e G = G; logo, e C (O, r) possui o mesmo centro e o mesmo raio da circunferncia constru na primeira parte e da da demonstrao deste teorema, portanto, as duas circunferncias so iguais. ca e a 10.49 Exerc cio. Mostre que o segmento P C, da prova do teorema anterior, tambm bissetriz e e do tringulo ABC. Conclua da que as bissetrizes de um tringulo interceptam-se num unico a a ponto. 10.50 Denio. O ponto de interseo das bissetrizes de um tringulo chama-se incentro ca ca a (centro da circunferncia inscrita). e 10.51 Teorema. Todo quadriltero circunscrit a vel possui a soma dos comprimentos de dois lados opostos igual a soma dos comprimentos dos outros dois lados, e reciprocamente. 10.52 Exerc cio. Prove o teorema anterior. 10.53 Exerc cio. Prove que todo pol gono regular circunscrit e vel. 10.54 Denio. Aptema de um pol ca o gono regular a distncia do centro da circunferncia e a e inscrita a um dos lados do pol gono. centro de um pol gono regular o centro desta circune ferncia (que coincide com o centro da circunferncia circunscrita). e e 10.55 Exerc cio. Dados uma circunferncia e um natural n, mostre que existe um pol e gono regular de n lados inscrito (circunscrito) na circunferncia. e 10.56 Exerc cio. A circunferncia inscrita em um pol e gono, quando existe, unica. e

10.3

Trigonometria
A

Seja um nmero qualquer do intervalo (0, 180). Construu amos um ngulo O de medida e, sobre um de seus lados, a tomemos um ponto arbitrrio A, distinto do vrtice. Dea e notamos por B o p da perpendicular de A ao outro lado e do ngulo (ou ao seu prolongamento). a

10.57 Denio. (i) Chamamos de seno do nmero , ca u e denotamos por sen , o quociente AB . OA Figura 10.10: Denio 10.57. ca (ii) Se 0 < 90, chamamos de cosseno do nmero u OB o quociente OA . Se 90 < < 180, chamamos de cosseno do nmero o quociente OB . u OA Denotamos o cosseno do nmero por cos . u Denimos ainda, sen 0 = 0, sen 180 = 0, cos 0 = 1, cos 180 = 1. 10.58 Teorema. Os valores sen e cos independem do ponto A escolhido. 10.59 Exerc cio. Prove o teorema anterior (sugesto: use semelhana de tringulos). a c a

86

Trigonometria
Como os valores de sen e cos dependem somente do nmero escolhido, denimos as u funes seno e cosseno, respectivamente, por co sen : [0, 180] R sen e cos : [0, 180] R cos .

10.60 Observao. Note que, as funes seno e cosseno esto bem denidas devido ao teorema ca co a 10.58. Se escolhermos o ponto A tal que OA = 1, ento as denies de seno e cosseno do nmero a co u cam simplicadas. 10.61 Teorema. Para todo nmero real [0, 180] , tem-se: u sen 90 = 1 0 sen 1 sen (180 ) = sen 10.62 Exerc cio. Prove o teorema anterior. 10.63 Teorema. Para todo nmero real [0, 180] , tem-se u cos2 + sen2 = 1. 10.64 Exerc cio. Prove o teorema anterior. Nos prximos teoremas desta seo, usaremos a notao a = BC, b = AC, c = AB para as o ca ca medidas dos lados de um tringulo ABC. a 10.65 Teorema. (Lei dos senos) Em todo tringulo ABC, tem-se a sen A sen B sen C 1 = = = , a b c 2R sendo R o raio da circunferncia circunscrita ao tringulo ABC. e a Prova. Seja C (O, R) a circunferncia circunscrita ao tringulo ABC. e a Faamos inicialmente o caso 0 < A 90. Conc sidere o dimetro CD e observe que, do fato de C BD a a C ser um tringulo retngulo, segue que sen D = 2R . Como a a os ngulos A e D so congruentes, pois subentendem o a a b a 1 F mesmo arco, resulta que sen A = 2R ; logo, sen A = 2R . a Se 90 < A < 180, ento tome E no arco maior determia A B nado por B e C. Como o quadriltero ABCE est inscrito a a na circunferncia C (O, R) , os ngulos A e E so suplee a a mentares; logo, 0 < E 90. Pelo que acabamos de provar, D a sen E = 2R e, sendo os ngulos A e E suplementares, a cos 90 = 0 1 cos 1 cos (180 ) = cos .

sen A = sen E. Portanto,

Figura 10.11: O caso 0 < 90.

b sen B
b

1 2R

b sen C
c

b sen A a

1 2R .

A demonstrao de ca

1 2R

se faz de modo anlogo. a

87

Comprimento de uma circunferncia e de arco de uma circunferncia e e


10.66 Teorema. (Lei dos cossenos) Em todo tringulo ABC, tem-se a a2 = b2 + c2 2bc cos A. c a Prova. Seja D o p da perpendicular a AC baixada por B. Faamos o caso em que o ngulo e A agudo. Dos tringulos retngulos ABD e BCD resulta que e a a c2 = BD2 + AD2 e a2 = BD2 + CD2 . Caso D esteja entre A e C, temos que AD + CD = b; logo, a2 = BD2 + (b AD)2 = BD2 + b2 2bAD + AD2 = b2 + c2 2bAD = b2 + c2 2bc cos A.

Caso C esteja entre A e D, temos que b + CD = AD; logo, a2 = BD2 + (AD b)2 = BD2 + b2 2bAD + AD2 = b2 + c2 2bAD = b2 + c2 2bc cos A.

Isto conclui a prova para o caso em que o ngulo A agudo. a e 10.67 Observao. Observe que quando A um ngulo reto, a igualdade do teorema anterior ca e a reduz-se ao Teorema de Pitgoras. a 10.68 Exerc cio. Complete a demonstrao do teorema anterior para o caso em que A um ca e a ngulo obtuso.

10.4

Comprimento de uma circunferncia e de arco e de uma circunferncia e

Seja C (O, R) uma circunferncia dada. Pelo exerc e cio 10.55, para cada natural n, existe um pol gono regular de n lados inscrito na circunferncia. Denotaremos o lado e o per e metro deste pol gono por ln e pn , respectivamente. A cada lado do pol gono est associado um tringulo issceles com um vrtice em O, laterais a a o e de medida R e base de medida ln . O ngulo do vrtice O de cada um destes tringulos mede a e a 2 n = n ; logo, pn = n ln = n2R sen n e, ento a sen pn = 2R n .
n

Como
n

lim

sen
n

= 1,

88

Exerc cios complementares


e ` medida que n cresce, o per a metro dos pol gonos inscritos aproximam-se cada vez mais do comprimento da circunferncia, tomamos a seguinte denio: e ca 10.69 Denio. O comprimento de uma circunferncia de raio R, 2R. ca e e Observe que ao multiplicarmos o ngulo central por qualquer natural n, o comprimento do a arco correspondente dever car multiplicado por n. Alm disso, o comprimento do arco aumenta a e a ` medida que o ngulo central cresce, isto , o comprimento de arco uma funo crescente do a e e ca a ngulo central. Pelo teorema fundamental da proporcionalidade (vide [14]), temos que o comprimento l de um arco diretamente proporcional ` medida do ngulo central, isto , e a a e l = k, sendo k a constante de proporcionalidade. Ela pode ser determinada observando que, para = , temos l = R; logo, k = R. Por isso, tomamos a seguinte denio: ca 10.70 Denio. O comprimento de um arco determinado por um ngulo central de meca a dida radianos R. e 10.71 Observao. Note que as denies de medida de arco e de comprimento de arco so ca co a distintas.

10.5

Exerc cios complementares

Indicaremos por R, ln , an as medidas do raio da circunferncia circunscrita, do lado e do aptema e o de um pol gono regular de n lados, respectivamente. 10.72 Exerc cio. Dado o raio da circunferncia circunscrita, calcule as medidas do lado e doo e aptema do quadrado. o 10.73 Exerc cio. Dado o raio da circunferncia circunscrita, calcule as medidas do lado e do e aptema do hexgono regular. o a 10.74 Exerc cio. Dado o raio da circunferncia circunscrita, calcule as medidas do lado e do e aptema do tringulo regular. o a 10.75 Exerc cio. Dado o raio da circunferncia circunscrita, calcule as medidas do lado e o do e aptema do decgono regular. o a 10.76 Exerc cio. Dado o raio da circunferncia circunscrita, calcule as medidas do lado e do e aptema do pentgono regular. o a 10.77 Exerc cio. Dados o raio da circunferncia circunscrita e o lado ln , calcule a medida do e aptema an . o 10.78 Exerc cio. Dados o raio da circunferncia circunscrita e o lado ln , calcule a medida e del2n .

89

Exerc cios complementares


10.79 Exerc cio. Calcule a medida do ngulo central determinado pelos lados de um pol a gono regular de n lados. 10.80 Exerc cio. Se A1 A2 ...A2n um pol e gono regular de 2n lados (nmero par de lados!), u ento suas diagonais A1 An+1 , A2 An+2 , ... , An A2n passam por um unico ponto, e este ponto a e o centro do pol gono. Prove isso. 10.81 Exerc cio. Se A1 A2 ...A2n um pol e gono regular de 2n lados (nmero par de lados!), u ento seus lados A1 A2 e An+1 An+2 ; A2 A3 e An+2 An+3 ; ... ; An1 An e A2n A1 ; so dois a dois a a paralelos. Prove isso. 10.82 Exerc cio. Prove que numa circunferncia ou em circunferncias de mesmo raio, cordas e e so congruentes se, e somente se, so equidistantes do centro. a a 10.83 Exerc cio. Prove que a mediatriz de uma corda passa pelo centro da circunferncia. e 10.84 Exerc cio. Prove que numa mesma circunferncia ou em circunferncias congruentes, se e e duas cordas tm comprimentos diferentes, a mais curta aquela mais afastada do centro. e e 10.85 Exerc cio. Prove que todo paralelogramo circunscrito a uma circunferncia um losango. e e 10.86 Exerc cio. Prove que a soma dos dimetros das circunferncias inscritas e circunscritas a e a um tringulo retngulo igual ` soma dos catetos desse tringulo. a a e a a 10.87 Exerc cio. Prove que todo trapzio inscrito em uma circunferncia issceles. e e e o 10.88 Exerc cio. Prove que todo paralelogramo inscrito em uma circunferncia retngulo. e e a 10.89 Exerc cio. Prove que o segmento determinado por um vrtice de um pol e gono regular e o centro da circunferncia em que ele est inscrito bissetriz do ngulo daquele vrtice. e a e a e 10.90 Exerc cio. Alm do tringulo, existem pol e a gonos inscritos em uma circunferncia que e so equingulos, mas que no so regulares? Alm do tringulo, existem pol a a a a e a gonos circunscritos em uma circunferncia que so equilteros, mas no so regulares? e a a a a 10.91 Exerc cio. Alm dos pol e gonos regulares, existem outros pol gonos equingulos inscrit a veis. conveniente subdividirmos os pol E gonos equingulos em dois grupos: os que possuem um a nmero u mpar de lados, e os que possuem um nmero par de lados. Mostre que todo pol u gono equingulo, inscrit a vel, com um nmero u mpar de lados, regular. Mostre que um pol e gono A1 A2 A3 A4 ...An1 An equingulo, com um nmero par de lados, tal que A1 A2 = A3 A4 = ... = a u An2 An1 e A2 A3 = A4 A5 = ... = An1 An , inscrit e vel. 10.92 Exerc cio. Alm dos pol e gonos regulares, existem outros pol gonos equilteros circunsa conveniente subdividirmos os pol crit veis. E gonos equilteros em dois grupos: os que possuem a um nmero u mpar de lados, e os que possuem um nmero par de lados. Mostre que todo pol u gono equiltero, circunscrit a vel, com um nmero u mpar de lados, regular. Mostre que um pol e gono A1 A2 A3 A4 ...An1 An equiltero, com um nmero par de lados, tal que A1 = A3 = ... = An1 e a u A2 = A4 = ... = An , circunscrit e vel.

90

Exerc cios complementares


10.93 Exerc cio. Mostre que o dimetro a maior corda de uma circunferncia. a e e 10.94 Exerc cio. Mostre que toda reta cuja distncia ao centro de uma circunferncia seja a e menor do que o raio, secante ` circunferncia. e a e 10.95 Exerc cio. Mostre que toda reta cuja distncia ao centro de uma circunferncia maior a e e do que o raio, exterior ` mesma. e a 10.96 Exerc cio. Mostre que, toda reta cuja distncia ao centro de uma circunferncia igual a e e ao raio, tangente ` mesma. e a 10.97 Exerc cio. Mostre que uma reta e uma circunferncia no podem ter mais do que dois e a pontos em comum. 10.98 Exerc cio. Mostre que trs pontos quaisquer de uma circunferncia no esto alinhados. e e a a 10.99 Exerc cio. Enuncie e prove o rec proco do exerc 10.93. cio 10.100 Exerc cio. Mostre que em cada ponto de uma circunferncia a reta tangente unica. e e 10.101 Exerc cio. No caso do exerc 10.11, mostre que se O o centro da circunferncia, T cio e e e T os pontos de tangncia, ento a semi-reta P O bissetriz do ngulo T P T , bissetriz do e a e a e a ngulo T OT e os ngulos T OT e T P T so suplementares. a a 10.102 Exerc cio. Estude todas as poss veis posies relativas de duas circunferncias. co e 10.103 Exerc cio. A reta que contm os centros de duas circunferncias dita reta dos cene e e tros. Se duas circunferncias so tangentes a uma mesma reta em um mesmo ponto, elas so e a a chamadas circunferncias tangentes e o ponto chamado ponto de contato. e e Mostre que se duas circunferncias so tangentes, a reta dos centros passa pelo ponto de e a contato. 10.104 Exerc cio. Na gura 10.12, P a interseo de duas tangentes comuns a duas circune ca ferncias. Mostre que a reta dos centros passa por P . e 10.105 Exerc cio. Mostre que se duas circunferncias tm dois pontos em comum, a reta dos e e centros mediatriz do segmento determinado por esses dois pontos. e 10.106 Exerc cio. Mostre que duas circunferncias distintas no podem ter mais do que dois e a pontos em comum. 10.107 Exerc cio. Descreva um mtodo para traar uma circunferncia de raio conhecido e e c e que seja tangente aos lados de um ngulo dado. a 10.108 Exerc cio. Duas circunferncias interceptam-se nos pontos A e B . Por B traa-se e c uma reta que intercepta uma das circunferncias num ponto X e a outra num ponto Y . Mostre e que a medida do ngulo X AY independe da reta traada. a c

91

Exerc cios complementares

Figura 10.12: Exerc 10.104. cio


10.109 Exerc cio. Sejam dadas duas circunferncias tangentes exteriores (isto , uma no est e e a a contida no interior da outra) e, pelo ponto de tangncia, tracemos duas secantes comuns. Mostre e que as cordas que ligam as extremidades das secantes em cada circunferncia so paralelas. e a 10.110 Exerc cio. Mostre que as tangentes traadas ` uma circunferncia pelos extremos de c a e um mesmo dimetro so paralelas. a a 10.111 Exerc cio. Utilizando a lei dos cossenos, expresse cada uma das medianas de um tringulo em funo de seus lados. a ca 10.112 Exerc cio. Mostre que todo segmento denido por um ponto fora e por outro ponto dentro de uma circunferncia tem um ponto em comum com a circunferncia. e e 10.113 Exerc cio. Mostre que a razo entre os raios das circunferncias inscritas de dois a e tringulos semelhantes igual ` razo de semelhana. a e a a c 10.114 Exerc cio. Mostre que a razo entre os raios das circunferncias circunscritas de dois a e tringulos semelhantes igual ` razo de semelhana. a e a a c 10.115 Exerc cio. Mostre que o raio da circunferncia inscrita em um tringulo de lados a, b, c e a e semiper metro p dado por r = e
(pa)(pb)(pc) . p

10.116 Exerc cio. O lugar geomtrico dos pontos que enxergam um segmento AB segundo e um ngulo o arco capaz de sobre AB. Qual o lugar geomtrico dos pontos que enxergam a e e e uma circunferncia segundo um ngulo ? e a

92

Cap tulo 11 Resultados Notveis sobre a Tringulos a


Apresenta-se alguns resultados que so caracter a sticos de tringulos; em particular, mostra-se a existncia de alguns a e pontos notveis, alm de teoremas que armam certas proa e priedades curiosas.

11.1

Pontos notveis de um tringulo a a

No corolrio 10.40, vimos que, em todo tringulo, as trs mediatrizes interceptam-se num mesmo a a e ponto. Esse ponto chamado circuncentro do tringulo; ele o centro da circunferncia e a e e circunscrita ao tringulo. Do mesmo modo, no teorema 10.48, vimos que a todo tringulo existe a a uma circunferncia inscrita e que o centro dela o ponto comum `s trs bissetrizes do tringulo. e e a e a Esse ponto chamado incentro do tringulo. e a O resultado seguinte nos diz que as trs alturas de um tringulo interceptam-se num unico e a ponto 11.1 Teorema. As retas que contm as trs alturas de um tringulo interceptam-se num ponto. e e a (esse ponto chamado ortocentro do tringulo). e a Prova: Seja ABC um tringulo qualquer e, atravs de cada um de seus vrtices, tracemos a e e retas paralelas aos lados opostos, obtendo um tringulo A B C , onde A est na interseo das a a ca paralelas passadas por B e C, B est na interseo das paralelas que passam por A e C, e C a ca na interseo das paralelas que passam por A e B (gura 11.1 a esquerda ). ca Assim, os quadrilteros ABCB e ACBC so paralelogramos e, da AB = overlineBC = a a , C A. Desde que B C paralelo a BC, a altura traada a partir de A ao lado BC a perpendicular e c e que bissecta o segmento B C ; o mesmo valendo para as alturas traadas por B e C. Em outras c palavras, as trs alturas do tringulo ABC so perpendiculares, que bissectam os trs lados do e a a e tringulo A B C (isto , so as mediatrizes do tringulo A B C ). Da pelo corolrio 10.40, a e a a , a segue que essas trs alturas interceptam-se num ponto. e O prximo teorema arma que as medianas de um tringulo interceptam-se num unico ponto. o a

93

Pontos notveis de um tringulo a a


C' A B'

C
B

G M

A'

Figura 11.1: Teoremas 11.1 e 11.2.


11.2 Teorema. As retas que contm as trs medianas de um tringulo interceptam-se num e e a ponto. (esse ponto chamado centride ou baricentro do tringulo). e o a Prova: Seja ABC um tringulo, com BD e CE medianas relativas aos lados AC e AB, a respectivamente (gura 11.1 a direita). Seja G o ponto de interseo de BD e CE. Prolonguemos AG at um ponto F tal que ca e GF = AG. No tringulo ABF , os pontos E e G so pontos mdios dos lados AB e AF , a a e respectivamente. Do teorema 7.58, segue que BF e EG so paralelas e, da BF e GC so a , a paralelas. Analogamente, CF e GB so paralelas. O quadriltero BF CG um paralelogramo; suas a a e e diagonais BC e F G interceptam-se no ponto mdio, M . Assim, o prolongamento de AG passa por M , ponto mdio de BC, isto , AM mediana relativa ao lado BC. Portanto, as trs e e e e medianas de um tringulo interceptam-se num mesmo ponto. a 11.3 Corolrio. O baricentro divide cada mediana em dois segmentos na razo de 2 para 1. a a Prova: Da prova do teorema 11.2, temos que: AG = GF = 2 GM CG = F B = 2 CE Analogamente, BG = 2 GD. 11.4 Exerc cio. Na prova do teorema 11.2, justique a existncia do ponto G e do ponto F . e No corolrio 11.3, prove que BG = 2 GD. a 11.5 Teorema. Em qualquer tringulo, as bissetrizes de dois ngulos externos e aquela do a a a ngulo interno restante interceptam-se num ponto. (esse ponto chamado excentro do tringulo). e a Prova: A prova deste teorema anloga `quela do teorema 10.48. e a a 11.6 Observao. O ponto mencionado acima tambm centro de uma circunferncia que ca e e e e tangente ao prolongamento de dois lados e ao lado restante do tringulo. Um tringulo tem trs a a e excentros.

94

Pontos notveis de um tringulo a a


11.7 Teorema. (Giovanni Ceva, 1648-1734) Sejam P, Q, R pontos sobre os lados BC, AC, AB (ou prolongamento deles) do tringulo ABC. As semi-retas AP , BQ, CR interceptam-se num a ponto se, e somente se, BP CQ AR = 1. P C QA RB Prova: A gura 11.2 ilustra as duas situaes: co

A A R B T P Q C B T C P

Figura 11.2: Teorema de Ceva.


Suponhamos que AP, BQ e CR interceptam-se num ponto, digamos T . Tracemos a reta que passa por A e paralela a BC; ela intercepta BQ e CR nos pontos B e C , respectivamente e (ou seus prolongamentos). Desde que os tringulos BP T e B AT so semelhantes, bem como a a os tringulos CP T e C AT so semelhantes, temos que a a BP AB PT AT = e = ; PT AT PC AC logo, BP AB = . PC AC Analogamente, CQ BC AR AC = e = . QA AB RB BC Multiplicando membro a membro as trs ultimas igualdades, temos que: e BP CQ AR = 1. P C QA RB Provemos agora a rec proca. Seja T a interseo de BQ e CR (ou de seus prolongamentos) ca a e P a interseo de AT e BC (ou seus prolongamento). Do que j provamos, segue que ca BP CQ AR =1 P C QA RB Por outro lado, por hiptese, temos que: o BP CQ AR = 1. P C QA RB (11.2) (11.1)

95

Pontos notveis de um tringulo a a


De (11.1) e (11.2), segue que BP BP = (11.3) PC PC Adicionando 1 a ambos os membros de (11.3) caso B P C, e 1 caso contrrio, temos a BP + P C BP + P C = ; PC PC
BC logo P C = BC e da P C = P C,o que implica P = P. PC Como corolrios do Teorema de Ceva podemos obter os resultados dos teoremas 10.48, a (11.1) e (11.2). De fato, se no Teorema de Ceva, tomarmos P, Q, R como pontos mdios de e BC, AC, AB, respectivamente, temos:

BP CQ AR = = = 1, PC QA RB e, da a condio suciente do teorema estar satisfeita, levando-nos a concluir que as medianas , ca a de um tringulo interceptam-se num ponto (vide teorema 11.2). a Se tomarmos as retas passando por A, B, C e perpendiculares a BC, AC, AB, respectivamente, temos P, Q, R os ps dessas perpendiculares; com a = BC, b = CA e c = AB, temos: e BP = c cos B, P C = b cos C, CQ = a cos C, QA = c cos A, AR = b cos A, RB = a cos B. Da , BP CQ AR c cos B a cos C b cos A = = 1, P C QA RB b cos C c cos A a cos B e, pelo Teorema de Ceva, conclu mos que as alturas de um tringulo interceptam-se num ponto a (vide teorema 11.1). Vimos, no Teorema da Bissetriz Interna que, as bissetrizes de um ngulo de um tringulo, a a dividem o lado oposto em dois segmentos cujos comprimentos esto na mesma razo que o a a comprimento dos dois lados restantes. Sejam, ento, P, Q e R as intersees das bissetrizes em A, B e C com os respectivos lados a co opostos. Pelo resultado acima, segue que c CQ a AR b BP = , = , = , PC b QA c RB a e, da , c a b BP CQ AR = = 1. P C QA RB b c a Pelo Teorema de Ceva, conclu mos que as bissetrizes de um tringulo interceptam-se num ponto a (vide teorema 10.48). Em resumo: os resultados dos teoremas 10.48, 11.1 e 11.2 so casos especiais do Teorema de a Ceva. 11.8 Teorema. (Euler) Em qualquer tringulo, seu baricentro, ortocentro e circuncentro so a a pontos colineares.

96

Pontos de Brocard
Prova: S eja ABC um tringulo qualquer. Consideremos as medianas relativas aos lados a BC, AC, AB e chamemos de MA , MB , MC os pontos mdios de BC, AC, AB, respectivamente. e Seja G o baricentro do tringulo ABC, isto , G o ponto comum a MA , MB , MC (vide teorema a e e 11.2). Fica, assim, determinado o tringulo MA MB MC . a Como MC MB = 1 BC, MC MA = 1 AC e MA MB = 2 2 1 A a a 2 AB, segue que os tringulos MA MB MC e ABC so semelhantes com razo de semelhana igual a 1/2. Sabea c mos, do exerc 9.22, que, nesses tringulos, segmentos cio a homlogos esto entre s na mesma razo, 1/2. o a a O Agora, pelo ponto A tracemos o segmento AHA , altura relativa ao lado BC. Por MB , tracemos a perpendicular a G AC que vai interceptar a perpendicular a BC pelo ponto I MA , no ponto I. Essas perpendiculares so mediatrizes dos a C B lados AC e BC, respectivamente; logo, I o circuncentro e do tringulo ABC (vide teorema.10.38). Por outro lado, a a altura relativa ao lado AC intercepta AHA num ponto Figura 11.3: A reta de Euler. O, o ortocentro do tringulo ABC (vide teorema 11.1). a Queremos provar que G, O, I, esto sobre uma mesma reta. Para isso, vamos considerar o a tringulo AOG e MA IG. Temos que I o ortocentro do tringulo MA MB MC (!), da MA I = a e a 1 1 a 2 AO, e ainda, MA G = 2 AG. Por outro lado, AO e MA I so paralelas cortadas pela transversal AMA ; logo, os ngulos GAO e GMA I so congruentes. Pelo segundo caso de semelhana de a a c tringulos, segue que os tringulos AOG e MA IG so semelhantes, logo, os ngulos AGO e a a a a a a MA GI so congruentes. Assim GO e GI so semi-retas opostas (!) e, portanto, os pontos G, O, I so colineares. a

..

11.9 Observao. A reta que contm o baricentro, ortocentro e o circuncentro de um tringulo ca e a chamada reta de Euler (em homenagem a Leonhard Euler (1707-1783), matemtico suco e a que descobriu e demonstrou o teorema 11.8). 11.10 Exerc cio. Mostre que o ponto I no teorema 11.8 o ortocentro do tringulo MA MB MC . e a

11.2

Pontos de Brocard

Seja ABC um tringulo qualquer. a 11.11 Denio. O ponto P , no interior de ABC, chamado primeiro ponto de Brocard ca e se P AC = P CB = P BA. O ponto Q, no interior de ABC, chamado segundo ponto de e Brocard se QAB = QCA = QBC (gura 11.4). 11.12 Teorema. Em qualquer tringulo, existe um unico primeiro ponto de Brocard. a Prova: Seja ABC um tringulo e sobre seus lados construamos os tringulos A1 BC, AB1 C a a e ABC1 , semelhantes ao tringulo ABC, como mostrado na gura 11.5. Seja P um ponto no a interior do tringulo ABC. a Como P CB = C P CA, as relaes P AC = P CB e P AC = C P CA so equivalentes. co a + P CA = 180 AP C. Da para um ponto P no interior do tringulo ABC, Assim, C = P AC , a

97

Pontos de Brocard

.P
A C A
Figura 11.4: Pontos de Brocard.

Q C

C1

A1

B1
Figura 11.5: Tringulos A1 BC, AB1 C, ABC1 . a

temos: C = 180 AP C se, e somente se, P um ponto sobre a circunferncia circunscrita ao e e tringulo AB1 C (teorema 10.43). a Um argumento anlogo para os outros ngulos mostra que vale o seguinte resultado (exa a istncia do primeiro ponto de Brocard): P o primeiro ponto de Brocard se, e somente se, ele e e pertence `s trs circunferncias circunscritas aos trs tringulos A1 BC, AB1 C e ABC1 . a e e e a A unicidade do primeiro ponto de Brocard segue da unicidade das circunferncias circune scritas aos tringulos A1 BC, AB1 C e ABC1 . a 11.13 Corolrio. Os segmentos AA1 , BB1 e CC1 passam pelo ponto P. a Prova: Liguemos o ponto P a todos os vrtices dos tringulos considerados. Da congruncia e a e de ngulos inscritos que subentendem o mesmo arco, os ngulos so precisamente como mostrado a a a na gura 11.5. Desde que + + = 180 , temos que os segmentos AA1 , BB1 e CC1 passam pelo ponto P.

98

Pontos de Brocard
11.14 Corolrio. P ponto de interseo das trs circunferncias tangentes `s retas AB, BC, AC a e ca e e a nos pontos A, B e C, e que passam pelos pontos C, A e B, respectivamente. Prova: Desde que ABC B A1 C, B CA C B1 A, C AB AC1 B, segue que as circunferncias circunscritas aos tringulos A1 BC, AB1 C, ABC1 so tangentes `s retas AB, BC, AC e a a a nos pontos A, B e C, respectivamente. 11.15 Exerc cio. Sobre os lados do tringulo ABC construa tringulos A1 BC, AB1 C e ABC1 , a a semelhantes ao tringulo ABC, de modo que AA1 , BB1 e CC1 interceptam-se no segundo ponto a de Brocard (justique sua construo). Prove ainda que o segundo ponto de Brocard unico. ca e Vejamos agora algumas propriedades interessantes dos pontos de Brocard. 11.16 Teorema. Se P o primeiro ponto de Brocard e se as retas AP , BP e CP interceptam e a circunferncia circunscrita ao tringulo ABC nos pontos A1 , B1 e C1 , respectivamente, ento e a a os tringulos ABC e B1 C1 A1 so congruentes (para o segundo ponto de Brocard temos ABC e a a C1 A1 B1 congruentes). Prova: Os arcos BC e A1 C1 so congruentes pois a BA1 C = BA1 + A1 C, A1 BC1 = A1 B + BC1 e BC1 = A1 C, da subentendem cordas congruentes, isto , BC = A1 C1 . Analogamente, , e podemos mostrar que AB = C1 B1 e AC = A1 B1 . Assim, pelo teorema 4.20, segue que ABC B1 C1 A1 . 11.17 Exerc cio. Prove que para o segundo ponto de Brocard ABC C1 A1 B1 . 11.18 Teorema. Se P A , P B e P C so segmentos com extremidades no primeiro ponto de a a a Brocard, P , e perpendiculares aos lados BC, CA e AB, respectivamente, ento os tringulos ABC e B C A so semelhantes e a razo de semelhana sen, onde = P AC = P CB = a a c e P BA. chamado ngulo de Brocard. e a Prova: Os pontos B e C pertencem ` circunferncia com dimetro AP, da a e a , B C P = B AP = e C B P = C AP = . Analogamente, C A P = e A C P = ; A B P = e B A P = . Assim, os ngulos do tringulo B C A so congruentes aos ngulos do tringulo ABC e, da a a a a a , esses tringulos so semelhantes. Alm disso, os ngulos dos tringulos ABP e B P C so a a e a a a congruentes, logo B C = B P = sen. AB AP

99

Exerc cios complementares

11.3

Exerc cios complementares

11.19 Exerc cio. Mostre que traando-se pelo baricentro de um tringulo uma reta qualquer c a que corta dois lados do mesmo, a soma das distncias dessa reta a dois vrtices, situados no a e mesmo semi plano relativo ` reta traada, igual ` distncia dessa reta ao terceiro vrtice. a c e a a e 11.20 Exerc cio. Prove que a soma das distncias dos vrtices de um tringulo a uma reta a e a qualquer igual a trs vezes a distncia dessa reta ao baricentro do tringulo considerado. e e a a 11.21 Exerc cio. Prove que a soma das distncias dos vrtices de um tringulo a uma reta a e a arbitrria igual ` soma das distncias desta reta ao ponto mdio dos trs lados do tringulo. a e a a e e a 11.22 Exerc cio. Seja ABC um tringulo qualquer. a

Mostre que entre o ngulo de Brocard e os ngulos , , do ABC subsistem as relaes: a a co (a) cotg =cotg+cotg+cotg (sugesto: observe a gura 11.6). a (b) sen3 =sen( )sen( )sen( )(sugesto: use a lei dos senos nos tringulos a a AP B, AP Ce BP C).

A1

.P
. A C
Figura 11.6: Exerc 11.22. cio

K1

100

Cap tulo 12 Construes com Rgua e Compasso co e


Os gregos antigos, pais da geometria axiomtica, sempre a tiveram verdadeiro fasc nio por resolverem problemas de geometria, constru ndo suas solues mediante uso de uma rgua co e no gra- duada e de um compasso. Para isso, eles possu a am regras para construo que deniam quais as operaes perca co mitidas para poder-se traar guras num plano. Apresenta-se c nesse cap tulo essas regras e descreve-se a resoluo de alguns ca problemas, apresentando uma construo para cada um deles. ca

12.1

Regras para construo com rgua e compasso ca e

Regra 1: S permitido traar segmentos e circunferncias utilizando-se uma rgua no graduada oe c e e a e um compasso. Regra 2: Supe-se, previamente dado, um segmento com comprimento unitrio. o a Regra 3: Se A e B so pontos previamente dados ou constru a dos, pode-se ligar A e B constru ndo o segmento AB; se este segmento intercepta qualquer segmento ou circunferncia previae mente constru dos, tem-se constru dos os pontos de interseo. ca Regra 4: Se AB um segmento previamente constru e O um ponto dado, ou previamente cone do stru do, pode-se traar uma circunferncia com centro em O e raio AB; se esta circunc e ferncia intercepta qualquer segmento ou circunferncia previamente constru e e das, tem-se constru dos os pontos de interseo. ca Regra 5: Se AB um segmento previamente constru e do, pode-se prolong-lo, em ambos os sentidos, a at interceptar um segmento ou circunferncia previamente constru e e dos (assumindo que o segmento ou a circunferncia intercepta o prolongamento do segmento dado), construindoe se, assim, um ponto. Regra 6: O unico meio de se construir uma gura aplicando as regras anteriores um nmero nito e u de vezes.

101

Alguns problemas de construo e suas solues ca co

12.2

Alguns problemas de construo e suas solues ca co

Para cada construo proposta nos problemas, execute os traados indicados. ca c 12.1 Problema. Dados dois segmentos, AB e CD, construir um segmento soma de AB e CD (entende-se por soma de AB e CD um segmento que tenha medida igual ` soma das medidas a de AB e CD). Construo: ca 1. Com centro em B e raio CD traamos uma c circunferncia. e 2. Prolongamos AB (no sentido de A para B) at interceptar a circunferncia constru no ponto e e da E. O segmento AE a soma desejada. e Desenho: Execute os traados indicados na c construo acima usando os dados da gura 12.1. ca

A C D

Figura 12.1: Problema 12.1.

12.2 Problema. Dado um segmento AB, obter seu ponto mdio M e sua mediatriz. e Construo: ca 1. Com centro em A e raio AB traamos uma circunferncia C1 ; c e 2. Com centro em B e raio AB traamos uma c circunferncia C2 . Obtemos os pontos P1 e P2 na e interseo de C1 e C2 . ca A B 3. Traamos o segmento P1 P2 obtendo o ponto c procurado, M , na interseo dele com AB. A reta ca P1 P2 a mediatriz procurada. e Desenho: Execute os traados indicados na c Figura 12.2: Problema 12.2. construo acima usando os dados da gura 12.2. ca 12.3 Problema. Construir um segmento que passa por um ponto dado e paralelo a um dado e segmento de reta que no contm esse ponto. a e Construo: ca Seja P o ponto e AB o segmento dado. 1. Centro em B e raio AP traamos uma circunferncia C1 . c e c e 2. Com centro em P e raio AB traamos uma segunda circunferncia, C2 . Seja D o ponto de interseo de C1 e C2 , onde D e A esto em lados opostos relativamente a P B. Assim, P D ca a a paralela desejada. e Desenho: Execute os traados indicados na construo acima usando os dados da gura c ca 12.3.

102

Alguns problemas de construo e suas solues ca co

.
A

B
Figura 12.3: Problema 12.3.

12.4 Problema. Construir um segmento que passa por um ponto dado, e perpendicular a e uma reta, tambm dada. e Construo: ca Sejam P e r o ponto e a reta dados, respectivamente. 1. Marquemos um ponto A sobre r e traamos uma circunferncia de centro P, cortando a c e reta r em outro ponto B. 2. Tracemos circunferncias de raio AB com centros em A e B obtendo Q, um dos pontos e de interseo. A reta P Q perpendicular a AB. ca e Desenho: Execute os traados indicados na construo acima usando os dados da gura c ca 12.4.

.
r
Figura 12.4: Problema 12.4.

12.5 Observao. A partir de agora, permitido utilizar esquadros para o traado de perca e c pendiculares e paralelas. Note que, de acordo com os problemas acima, estes instrumentos so dispensveis para tais traados, visto que, somente com rgua no graduada e compasso, a a c e a podemos traar perpendiculares e paralelas. O uso dos esquadros tem a vantagem de agilizar a c resoluo dos problemas seguintes. ca 12.6 Problema. Construir uma semi-reta que divide um ngulo dado em dois outros ngulos a a congruentes (bisseco de um ngulo). ca a Construo: Seja ABC um ngulo dado. ca a e 1. Com centro em B e raio BA, tracemos uma circunferncia que intercepta BC no ponto E. Pode ocorrer que a circunferncia no intercepte BC. Neste caso, prolonguemos BC no sentido e a de B para C at interceptar a circunferncia num ponto que ser chamado de E. e e a 2. Com centro em A e raio AE, tracemos a circunferncia C1 e com centro em E raio AE, e tracemos a circunferncia C2 . Essas duas circunferncias interceptam-se em dois pontos. Seja e e

103

Alguns problemas de construo e suas solues ca co


F o ponto de interseo que est no semi plano oposto `quele determinado por AE e que no ca a a a contm B. Observemos que AEF um tringulo equiltero. e e a a 3. Liguemos B a F para obter o segmento BF que bissecta o ngulo ABC. a Desenho: Execute os traados indicados na construo acima usando os dados da gura c ca 12.5, a esquerda.

A r

fim do papel

B s C
Figura 12.5: Problemas 12.6 e 12.7.
12.7 Problema. Construir a bissetriz de um ngulo cujo vrtice inacess a e e vel. Construo: ca Sejam r, s os lados do ngulo. a 1. Passemos uma paralela ` r que intercepta s no ponto A. a 2. Centro em A e raio qualquer, tracemos uma circunferncia que intercepta s e a paralela e a ` r nos pontos B, C, respectivamente. 3. Unimos B a C e prolonguemos at encontrar r num ponto D. e 4. A mediatriz de BD a bissetriz procurada. e Desenho: Execute os traados indicados na construo acima. c ca 12.8 Problema. Dado um ngulo de vrtice O e uma semi-reta O B, construir um ngulo a e a AO B = . Construo: ca 1. Tracemos uma circunferncia qualquer de centro O, determinando os pontos P e Q nos e lados do ngulo , e uma circunferncia de mesmo raio, e centro em O , determinando P em a e O B. 2. Com raio P Q, tracemos uma circunferncia de centro P para determinar Q sobre a e primeira circunferncia. e Desenho: Execute os traados indicados na construo acima usando os dados da gura c ca 12.6. Relembremos a denio de arco capaz. Consideremos dois pontos, A e B, sobre uma ca circunferncia. Para todo ponto M sobre um dos arcos determinados por A, B, o ngulo AM B e a tem sempre medida . Este arco chama-se arco capaz do ngulo sobre o segmento AB. Assim, a um observador, portanto, que mova-se sobre este arco, consegue ver o segmento AB sempre sob mesmo ngulo. Para qualquer ponto N pertencente ao outro arco, o ngulo AN B tambm a a e e constante e igual a 180 .

104

Alguns problemas de construo e suas solues ca co

O'
Figura 12.6: Problema 12.8.

. B

E ainda interessante lembrar que se M qualquer ponto da circunferncia de dimetro AB, e e a o ngulo AM B reto e, portanto, cada semi circunferncia tambm o arco capaz de 90 sobre a e e e e AB. 12.9 Problema. Dados o segmento AB e o ngulo = B AX, construir o arco capaz do ngulo a a sobre AB. Construo: ca 1. Tracemos a mediatriz do segmento AB e a perpendicular ` AX no ponto A. a 2. A perpendicular ` AX, traada por A, encontra a c a mediatriz de AB em O, centro do arco capaz. 3. O arco de centro O e extremidades A e B situado em semi plano oposto a X (semiplanos relativos a AB) o arco capaz do ngulo sobre AB. e a Desenho: Execute os traados indicados na conc struo acima usando os dados da gura 12.7. ca

.X
Figura 12.7: Problema 12.9.

12.10 Problema. Obter o centro de uma circunferncia dada. e Construo: ca 1. Marquemos trs pontos arbitrrios A, B, C sobre a e a circunferncia. e 2. Tracemos as mediatrizes de AB e AC. 3. O ponto O de encontro das mediatrizes o centro e da circunferncia. e Desenho: Execute os traados indicados na conc struo acima usando os dados da gura 12.8. ca

Figura 12.8: Problema 12.10.

12.11 Problema. Traar duas semi-retas que dividam um ngulo reto em trs ngulos conc a e a gruentes. Construo: ca Seja O o ngulo dado. a

105

Expresses algbricas o e
1. Centro em O e raio qualquer, tracemos um arco, que corta os lados do ngulo em dois pontos, os quais a chamaremos de A, B. 2. Com o mesmo raio e centro em A e B, construamos duas circunferncias, que cortam o arco constru no item e do 1, nos pontos C, D. 3. As semi-retas OC e OD constituem a soluo do ca problema. Desenho: Execute os traados indicados na conc struo acima usando os dados da gura 12.9. ca

O
Figura 12.9: Exerc 12.11. cio

12.12 Observao. E poss mostrar que o problema da trisseo de um ngulo arbitrrio, ca vel ca a a no pode no tem soluo com rgua no graduada e compasso. Por exemplo, o ngulo de 60 a a ca e a a ser trissectado somente com rgua e compasso. e

12.3

Expresses algbricas o e

12.13 Problema. Dado um segmento AB, construir um segmento que tenha medida igual ` a raiz quadrada da medida de AB. Construo: ca 1. Ao segmento AB, adicionemos o segmento de medida unitria OX, para obter um segmento AC a tal que AC = AB + 1. 2. Usemos a construo do problema 12.6 para ca traar a perpendicular ao segmento AC passando c A B O por B. 3. Construamos o ponto mdio, M , de AC. e Figura 12.10: Problema 12.13. 4. Com centro em M e raio M C, tracemos uma circunferncia que cortar a perpendicular do item 2 num ponto E. O segmento BE o segmento e a e requerido, isto , BE = AB. e Desenho: Execute os traados indicados na construo acima usando os dados da gura c ca 12.10.

12.14 Problema. Construir um segmento cuja medida seja o produto das medidas de dois segmentos dados AB e CD. Construo: ca Sejam AB e CD segmentos dados (ou previamente constru dos). 1. Com centros em C e D e raio CD, construamos duas circunferncias que interceptam-se nos e pontos E e E . 2. Com centro em C e raio AB, cortemos CE (ou o prolongamento de CE no sentido de C para E) em F .

B X C A O D

Figura 12.11: Problema 12.14.

106

Expresses algbricas o e
3. Se O e X so pontos que denem um sega mento unitrio, OX, ento, com centro em C e raio OX, cortemos CD (ou CD prolongado no a a sentido de C para D) em G. 4. Unamos F e G, obtendo o segmento F G. 5. Usando o problema 12.3, trace, atravs de D, com segmento paralelo a F G, que intercepta e CE (ou seu prolongamento) em H. O segmento CH o produto requerido. e c ca Desenho: Execute os traados indicados na construo acima usando os dados da gura 12.11. 12.15 Problema. Construir um segmento cuja medida o inverso da medida de um segmento e dado. Construo: ca Seja AB um segmento previamente constru do. X 1. Com centro em A e B e raio AB construamos circunferncias que interceptam-se nos pontos C e e C. 2. Com centro em A e raio OX (segmento A B O unitrio), cortemos AC (ou prolongamento de AC a no sentido de A para C) em D. Figura 12.12: Problema 12.15. 3. Com centro em A e raio OX, corte AB (ou prolongamento de AB no sentido de A para B) em E. 4. Tracemos a reta, passando por E, que paralela ` BD interceptando AC em F . O e a segmento AF a resposta do problema. e Desenho: Execute os traados indicados na construo acima usando os dados da gura c ca 12.12. 12.16 Problema. Dado um segmento AB, obter sua seo urea. ca a Construo: ca 1. Passemos por B uma perpendicular a AB. 2. Sobre esta perpendicular, marquemos um ponto D tal que BD = 1 AB. 2 3. Unamos os pontos A, D. 4. Tracemos um arco de centro D e raio BD, que corta AD em um ponto E. 5. Tracemos um arco de centro A e raio AE, que corta AB em um ponto C. 6. O ponto C a seo urea de AB. e ca a Desenho: Execute os traados indicados na construo acima usando os dados da gura c ca 12.13. 12.17 Problema. Dividir um segmento AB, dado, em 5 partes iguais. Construo: ca 1. Tracemos uma semi-reta qualquer AX. 2. Sobre ela, construamos, com o compasso, os Figura 12.13: Problemas 12.16 e 12.17. segmentos congruentes AA1 , A1 A2 , A2 A3 , A3 A4 e A4 A5 .

107

Construo de tringulos ca a
3. Tracemos as paralelas a A5 B pelos pontos A1 , A2 , A3 e A4 , que determinam, assim, no segmento AB, os pontos P1 , P2 , P3 e P4 , que o dividiro em 5 partes iguais. a Desenho: Execute os traados indicados na construo acima. c ca 12.18 Observao. Tendo como referncia os problemas acima, temos que, partindo de um ca e segmento unitrio OX, podem-se construir segmentos com qualquer medida racional posia tiva. Baseados nesta observao, costuma-se dizer que os nmeros racionais positivos so conca u a strut veis. Observa-se, tambm, que raiz quadrada de qualquer segmento construt tambm e vel e e um exerc construt vel. E cio, no dif mostrar que, se a, b e k so medidas de segmentos cona cil, a strut veis, tambm construt o segmento de medida a + b k. Tambm no tarefa dif e e vel e a e cil 5 mostrar que o nmero 5 + 7 3 + 10+2 pode ser medida de um segmento (tomando OX como u segmento unitrio). Pode-se mostrar que os nmeros reais que podem ser medidas de segmentos a u construt veis, so todos da forma a + b k (vide [11]). a

12.4

Construo de tringulos ca a
A

12.19 Observao. A partir de agora, adotaremos a nomenclatura sugerida pela gura 12.14. ca

ma mb a

b mc C

Figura 12.14: Tringulo ABC e suas medianas. a


12.20 Problema. Construir o tringulo ABC, sendo dados os seus lados a, b, c. a Construo: ca 1. Marquemos o lado AB tal que AB = c. 2. Construamos as circunferncias C1 , C2 e de raios a, b e centros B, A, respectivamente. 3. O ponto de encontro entre C1 e C2 o e vrtice C. e c Desenho: Execute os traados indicados na construo acima usando os dados da gura ca 12.15.

A B A C C

Figura 12.15: Problema 12.20.

108

Construo de tringulos ca a
12.21 Problema. Construir o tringulo ABC, sendo dados os lados a, b e o ngulo C = . a a Construo: ca 1. Marquemos o lado BC tal que BC = a. b a 2. Transportemos o ngulo C, a de modo que seu vrtice coincida e com a extremidade C do segmento BC e um de seus lados seja o segmento BC. 3. Marquemos, sobre o outro lado do ngulo transportado, um a Figura 12.16: Problema 12.21. ponto A tal que AC = b. Desenho: Execute os traados c indicados na construo acima usando os dados da gura 12.16. ca 12.22 Problema. Construir o tringulo ABC, sendo dados os ngulos B, C e o lado a. a a Construo: ca 1. Marquemos o lado BC tal que BC = a. 2. Transportemos o ngulo C, de modo que seu a vrtice coincida com a extremidade C do segmento e BC e um de seus lados seja o segmento BC. 3. Transportemos o ngulo B, de modo que seu a vrtice coincida com a extremidade B do segmento e BC e um de seus lados seja o segmento BC. 4. O ponto de interseo dos lados dos ngulos ca a transportados, que no contm BC, o ponto A. a e e Desenho: Execute os traados indicados na c construo acima usando os dados da gura 12.17. ca

Figura 12.17: Problema 12.22.

12.23 Problema. Contruir o tringulo ABC, sendo dados os lados c, a e o ngulo A = . a a Construo: ca 1. Marquemos o lado AB no papel tal que AB = c. 2. Construamos a semi-reta AX tal que B AX = . O vrtice C ser, ento, um dos e a a pontos de interseo da semi-reta AX com a ca circunferncia de centro B e raio a. e Desenho: Execute os traados indicados c na construo acima usando os dados da gura ca 12.18.

Figura 12.18: Problema 12.23.

12.24 Observao. Com os dados apresentados, o problema teve duas solues. Podemos ca co observar que esse problema nem sempre possui soluo. Se o lado BC, de medida a, for pequeno ca (a < c sen ), a circunferncia, de centro B e raio a, no cortar a semi-reta S(AX) . A contruo e a a ca

109

Problemas de tangncia e
mostra porque uma correspondncia entre dois tringulos do tipo LLA no necessariamente e a a e uma congruncia. e 12.25 Problema. Construir o tringulo ABC, sendo dados o lado a, a altura ha e o ngulo A. a a Construo: ca a ha 1. Tracemos um segmento BC de medida a. 2. Construamos o arco capaz de A relativo ao segmento BC. 3. Construamos uma reta r, paralela ^ a BC contida no mesmo semi plano deA terminado por BC que o arco capaz do tem anterior, e cuja distncia a BC seja a Figura 12.19: Problema 12.25. igual a ha . 4. A interseo de r com o arco capaz d o ponto A. ca a Desenho: Execute os traados indicados na construo acima usando os dados da gura c ca 12.19. 12.26 Observao. Nem sempre existe a soluo. Se ha for grande, a paralela ` r no cortar ca ca a a a o arco capaz. 12.27 Problema. Construir o tringulo ABC, sendo dadas as medianas ma e mb e a altura a ha . Construo: ca 1. Tracemos duas retas paralelas, r e s, ma mb ha distando ha uma da outra. Sobre r, marque um ponto A. 2. Centro em A e raio ma , construamos Figura 12.20: Problema 12.27. uma circunferncia, que corta s num ponto M . e 2 3. Seja G o ponto de AM tal que AM = ma . 3 2 4. Centro em G e raio mb , construamos uma circunferncia, que corta s num ponto B. e 3 Marque, com raio BM , uma circunferncia de centro M , obtendo assim o ponto C sobre s. e Desenho: Execute os traados indicados na construo acima usando os dados da gura c ca 12.20. 12.28 Observao. Os dados apresentados no determinam um unico tringulo. Note ainda ca a a que, dependendo das relaes entre os dados, o problema pode no ter soluo. co a ca

12.5

Problemas de tangncia e

12.29 Problema. Traar uma tangente a uma circunferncia dada, que seja paralela a uma c e reta tambm dada. e Construo: ca

110

Problemas de tangncia e
Sejam C (O, r) e s a circunferncia e a reta dadas, e respectivamente. 1. Passemos por O, uma perpendicular a s, que corta a circunferncia C (O, r) em dois pontos, A, B. e 2. Passemos por A (ou por B), uma paralela a s. Desenho: Execute os traados indicados na conc struo acima usando os dados da gura 12.21. ca

Figura 12.21: Problema 12.29.

12.30 Problema. Dada uma circunferncia, traar as tangentes a esta circunferncia por um e c e ponto P exterior a ela. Construo: Devemos, inicialmente, obter os pontos de tangncia. ca e 1. Se O o centro da circunferncia e A um dos e e e pontos de tangncia, ento o ngulo P AO reto. Logo, e a a e A petence a um arco capaz de 90 sobre P O. 2. Determinemos, ento o ponto mdio de P O e a e P traamos a circunferncia de dimetro P O, que determina c e a sobre a circunferncia dada, os pontos de tangncia procue e rados, A e A . Desenho: Execute os traados indicados na conc Figura 12.22: Problema 12.30. struo acima usando os dados da gura 12.22. ca

12.31 Problema. Traar as tangentes comuns a duas circunferncias dadas. c e Construo: ca Sejam C (O1 , r1 ) e C (O2 , r2 ) as duas circunferncias tais que O1 O2 > r1 + r2 . Se r1 = r2 , a e construo das quatro tangentes comuns ca como exerc ca cio. Faamos o caso r1 < r2 . c 1. Tracemos dois raios paralelos O1 A e O2 B, de C (O1 , r1 ) e C (O2 , r2 ) , respectivamente. 2. Liguemos A a B at cortar a reta dos e centros, no ponto P. 3. Passemos por P uma tangente a C (O1 , r1 ) . Analogamente constru mos a outra tangente comum que passa por P. Passamos agora ` construo das demais tana ca gentes. 5. Tracemos uma circunferncia de centro e O2 e raio r1 + r2 . 6. Determinamos o ponto X de tangncia e de umas das tangentes a C (O2 , r1 + r2 ) que Figura 12.23: Problemas 12.31. passa por O1 .

111

Problemas de tangncia e
7. Construamos O2 X e chamamos de A, a interseo de O2 X com C (O2 , r2 ). Este um ca e dos pontos de tangncia procurados. e 8. Tracemos uma paralela a O2 A passando por O1 , que corta C (O1 , r1 ) no ponto B. e a e e 9. A reta AB uma tangente comum `s circunferncias procuradas. A outra tangente constru de modo anlogo. da a Desenho: Execute os traados indicados na construo acima usando os dados da gura c ca 12.23. 12.32 Problema. Inscrever uma circunferncia, de raio r dado, em um ngulo, tambm dado. e a e Construo: 1. Construamos a bissetriz s do ngulo dado. ca a 2. Construamos um paralela a um dos lados do ngulo, cuja distncia at a a e este lado seja igual ao raio da circunr ferncia dado. e 3. O ponto de encontro da bissetriz com a paralela o centro da circune ferncia. e Figura 12.24: Problema 12.32. Desenho: Execute os traados indic cados na construo acima usando os dados da gura 12.24. ca 12.33 Problema. Traar uma circunferncia tangente a trs retas dadas. c e e Construo: Sejam AB, BC, CD as retas dadas. ca 1. Construamos as bissetrizes dos ngulos a D A B e C, e chamamos de O o ponto de interseo ca entre elas. 2. Passemos por O uma perpendicular a AB, e chamamos de P o p desta perpendice ular. 3. A circunferncia de centro O e raio OP, e B a circunferncia procurada. e e Desenho: Execute os traados indicados c C na construo acima usando os dados da gura ca Figura 12.25: Problema 12.33. 12.25. 12.34 Problema. Traar trs circunferncias de mesmo raio, tangentes exteriormente entre si, c e e e tangentes interiormente a uma outra dada. Construo: ca Seja C (O, r) a circunferncia dada. e 1. Marquemos em C (O, r) , trs pontos A, B, C tais que AB = BC = AC, e constru e mos os raios OA, OB, OC. 2. Prolonguemos os raio AO na direo de A para O. ca 3. Tracemos por C uma tangente a C (O, r) , que corta o prolongamento de AO num ponto, o qual chamaremos de P.

112

Exerc cios complementares


4. Centro em P e raio P C, tracemos um arco que corta OP num ponto, que chamaremos de D. 5. Passemos por D uma perpendicular a AP. O ponto e de encontro desta perpendicular com OB o centro O1 da primeira circunferncia. Seu raio O1 B. e e 6. Construamos, com centros nos pontos A, C, e raio O1 B, dois arcos que cortam OA e OC, respectivamente, nos pontos O2 e O3 . Estes so os centros das demais cira Figura 12.26: Problema 12.34. cunferncias. e Desenho: Execute os traados indicados na construo acima usando os dados da gura c ca 12.26.

12.35 Problema. Descrever com raio dado, uma circunferncia tangente a duas outras dadas. e Construo: ca Sejam C (O1 , r1 ) e C (O2 , r2 ) as duas circunferncias e AB o raio, dados. e 1. Centro em O1 , raios iguais a AB + r1 e AB r1 , tracemos duas circunferncias. e 2. Centro em O2 , raios iguais a AB + r2 e AB r2 , tracemos duas circunferncias. e 3. Estas quatro circunferncias se e cortam em oito pontos. Com centros nestes pontos e raio AB, constru mos oito circunferncias, sendo que cada e Figura 12.27: Problema 12.35. uma delas resolve o problema. Desenho: Execute os traados indic cados na construo acima usando os dados da gura 12.27. ca

12.6

Exerc cios complementares

12.36 Exerc cio. Baseado na axiomtica apresentada nos cap a tulos anteriores, justique as construes propostas nos problemas das sees 12.2-12.5. co co 12.37 Exerc cio. Um aluno apresenta o seguinte problema de desenho geomtrico ao seu proe fessor: construir um tringulo retngulo, dados o segmento soma dos catetos e a hipotenusa. a a O professor deu ao aluno os seguintes passos para a construo: ca 1. Trace o segmento AB, que representa a soma dos catetos. 2. Pela extremidade A,construa um ngulo de 45 . a 3. Com centro em B e raio igual ` hipotenusa, trace uma circunferncia que corta o lado do a e a ngulo constru em 2 no ponto C. do 4. Por C, trace a reta perpendicular a AB obtendo o ponto D em AB . 5. O tringulo CDB o tringulo procurado. a e a Justicar, geometricamente, a construo apresentada pelo professor. ca

113

Exerc cios complementares


Sendo a, a medida da hipotenusa, b e c as medidas dos catetos, diga que relao deve existir ca entre esses valores, para que os dados do problema sejam compat veis? 12.38 Exerc cio. Construir o simtrico de um ponto P em relao a uma reta dada. e ca 12.39 Exerc cio. Construir uma circunferncia que passa por trs pontos no colineares dados. e e a 12.40 Exerc cio. Dados os segmentos de medida a, b,construa um segmento de medida a . b 12.41 Exerc cio. Dados os segmentos de medida a, b,construa um segmento de medida (construo da mdia aritmtica). ca e e 12.42 Exerc cio. Dados os segmentos de medida a, b construa um segmento de medida (construo da mdia geomtrica). ca e e
a+b 2

ab

12.43 Exerc cio. Dados os segmentos de medida a, b, c, construa um segmento de medida x c tal que a = x . O nmero x dito a quarta proporcional dos nmeros a, b, c. u e u b 12.44 Exerc cio. Inscrever, em um tringulo dado ABC, um quadrado (isso signica que, um a lado contido do quadrado deve estar contido num dos lados do tringulo, e todos os vrtices do a e quadrado devem pertencer ao tringulo). a 12.45 Exerc cio. Dados os segmentos de medidas a, b, construa um segmento de medida a b. 12.46 Exerc cio. Dados os segmentos de medidas a, b, construa um segmento de medida a2 + b2 . 12.47 Exerc cio. Dados os segmentos de medidas a, b, construa um segmento de medida a2 b2 . 12.48 Exerc cio. Dados os segmentos de medidas a, b, construa um segmento de medida a + b. 12.49 Exerc cio. Dados os segmentos de medidas a, b, c, construa um segmento de medida a2 + b2 + c2 . 12.50 Exerc cio. Dado o segmento de medida a, construa os segmentos de medidas a 2 , a 3, a 4, a 5, a 6... 12.51 Exerc cio. Dado o segmento de medida a n , construa um segmento de medida a. 12.52 Exerc cio. Construir um segmento de medida 4, 7. 12.53 Exerc cio. Resolva o sistema, geometricamente, dados a, b x+y =a . xy = b2 12.54 Exerc cio. Resolva o sistema, geometricamente, dados a, b, sendo b um nmero positivo u x2 y 2 = a2 . x+y =b

114

Exerc cios complementares


12.55 Exerc cio. Resolva o sistema, geometricamente, dados a, b x2 + y 2 = a2 . x + y = b2 12.56 Exerc cio. Resolva o sistema, geometricamente, dados a, b, sendo a positivo xy =a . xy = b2 12.57 Exerc cio. Resolver, geometricamente, a equao x2 ax b2 = 0. ca 12.58 Exerc cio. Construir x tal que 12.59 Exerc cio. Construir x tal que
1 x2 1 x

= =

1 a2 1 a

1 , b2

dados a, b .

+ 1 , dados a, b positivos. b

12.60 Exerc cio. Dados a hipotenusa, a , e a diferena entre os catetos b c , construa o c tringulo retngulo que tem esses dados como lados (gura 12.28). D uma receita para a a a e construo e justique-a geometricamente. Que relao a, b e c devem satisfazer, para o problema ca ca estar bem posto?

a b-c
Figura 12.28: Exerc cios 12.60.

12.61 Exerc cio. Construa um tringulo com lado a, um ngulo adjacente a esse lado, B e a a a bissetriz b do outro ngulo. Os dados esto na gura 12.29. a a

Figura 12.29: Exerc cios 12.61.

Para os exerc cios 12.62 a 12.66, utilize a gura 12.30. 12.62 Exerc cio. Construa um hexgono regular que est inscrito numa circunferncia dada. a a e Justique, geometricamente, sua construo. ca

115

Exerc cios complementares

Figura 12.30: Exerc cios 12.62 a 12.66.


12.63 Exerc cio. Construa um tringulo regular que est inscrito em uma circunferncia dada. a a e Justique, geometricamente, sua construo. ca 12.64 Exerc cio. Construa um quadrado que est inscrito numa circunferncia dada. Justia e que, geometricamente, sua construo. ca 12.65 Exerc cio. Construa um decgono regular que est inscrito numa circunferncia dada. a a e Justique geometricamente sua construo. ca 12.66 Exerc cio. Construa um pentgono regular que est inscrito numa circunferncia dada. a a e Justique geometricamente sua construo. ca 12.67 Exerc cio. Construir um quadrado conhecendo sua diagonal. 12.68 Exerc cio. Construir um quadrado, dados em posio os pontos mdios de dois lados ca e adjacentes. 12.69 Exerc cio. Construir a circunferncia circunscrita a um tringulo. e a 12.70 Exerc cio. Construir a circunferncia inscrita em um tringulo. e a 12.71 Exerc cio. Construir um trapzio, conhecendo as bases a e b e os outros dois lados c e e d. 12.72 Exerc cio. Construir um hexgono regular, dado, em posio, um lado. a ca 12.73 Exerc cio. Construir uma perpendicular ao segmento AB, pelo ponto A , estando este ponto prximo ao bordo do papel. o 12.74 Exerc cio. Construir um tringulo ABC, conhecendo os lados a e b e a altura ha . a 12.75 Exerc cio. Construir um tringulo ABC, conhecendo os lados b e c e a altura ha . a 12.76 Exerc cio. Construir um tringulo ABC, conhecendo os lados b e c e a mediana ma . a 12.77 Exerc cio. Construir um tringulo ABC, conhecendo o lado a , o ngulo A e a mediana a a ma .

116

Exerc cios complementares


12.78 Exerc cio. Construir um tringulo ABC, conhecendo os lados a e b e o ngulo A. a a 12.79 Exerc cio. Construir um tringulo ABC, conhecendo o lado a , o ngulo A e a mediana a a mb . 12.80 Exerc cio. Construir um tringulo ABC, conhecendo o lado a e as medianas mb e mc . a 12.81 Exerc cio. Construir um tringulo ABC, conhecendo o lado a e as alturas hb e hc . a 12.82 Exerc cio. Construir um tringulo ABC, conhecendo a mediana ma e as alturas ha e a hb . 12.83 Exerc cio. Construir um tringulo ABC, conhecendo a mediana ma e as alturas hb e a hc . 12.84 Exerc cio. Construir um tringulo ABC, conhecendo o lado a, o ngulo A e a soma, s = a a b + c, dos outros dois lados. 12.85 Exerc cio. Construir um tringulo ABC, conhecendo o lado a, o ngulo A e a diferena, a a c d = b c, dos outros dois lados. 12.86 Exerc cio. Construir um tringulo ABC, conhecendo o per a metro 2p e os ngulos B e a C . 12.87 Exerc cio. Construir um tringulo ABC, conhecendo o per a metro 2p, o ngulo A e a a altura ha . 12.88 Exerc cio. Construir um tringulo ABC, conhecendo o lado a, a altura ha e o raio R a da circunferncia circunscrita. e 12.89 Exerc cio. Construir um tringulo ABC, conhecendo a altura ha , a mediana ma e o a raio R da circunferncia circunscrita. e 12.90 Exerc cio. Construir um tringulo ABC, conhecendo o ngulo A, o lado b e o raio r da a a circunferncia inscrita. e 12.91 Exerc cio. Construir um tringulo conhecendo os comprimentos da altura, mediana e a bissetriz relativas a um mesmo vrtice. e 12.92 Exerc cio. Determinar o ponto de contato de uma circunferncia dada, com uma tane gente tambm dada. e 12.93 Exerc cio. Traar uma tangente a uma circunferncia dada, por um ponto dado sobre c e a circunferncia. e 12.94 Exerc cio. Traar uma tangente a uma circunferncia cujo centro seja inacess c e vel. 12.95 Exerc cio. Traar uma circunferncia de raio dado e que seja tangente exterior a uma c e circunferncia dada. e

117

Exerc cios complementares


12.96 Exerc cio. Traar uma circunferncia de raio dado e que seja tangente interior a uma c e circunferncia dada. e 12.97 Exerc cio. Dadas, em posio, uma circunferncia C e uma reta r, construir uma circa e cunferncia de raio dado, tangente ` r e tangente exteriormente ` C . e a a 12.98 Exerc cio. So dadas, em posio, as retas r e s e a circunferncia C . Determinar os a ca e pontos de C que so eqidistantes de r e s . Qual o nmero mximo de solues? a u e u a co 12.99 Exerc cio. So dadas, em posio, uma circunferncia C e uma reta r . Determinar um a ca e ponto P sobre r, de forma que as tangentes traadas de P ` circunferncia C formem um ngulo c a e a , dado. 12.100 Exerc cio. Traar uma circunferncia que passe por dois pontos dados e que seja tanc e gente ` outra circunferncia dada. a e 12.101 Exerc cio. Complete a resoluo do problema 12.31, analisando os casos no considerca a ados.

118

Cap tulo 13 Area de Figuras Planas


Area de regio poligonal denida mediante axiomas que pera e mitiro introduzir as frmulas usuais para reas de tringulos, a o a a retngulos, trapzios e outros pol a e gonos particulares.

13.1

Denies e axiomas co

Figura 13.1: Tringulo ABC, regio triangular RABC e interior de RABC . a a

1. Relembremos que, dados trs pontos A, B e C, no colineares, o tringulo ABC e a a e o conjunto AB AC BC, onde AB, AC e BC representam segmentos determinados pelos pontos A, B, C (gura 13.1 ` esquerda). a 2. Regio triangular um conjunto dado pela unio de todos os segmentos, cujas exa e a tremidades pertencem a um tringulo. a Assim, se ABC um tringulo, o conjunto RABC = e a XY a regio triangular e a
X,Y ABC

determinada por ABC (gura 13.1, no centro). 3. Fronteira de uma regio triangular o tringulo que a dene. a e a Assim, ABC = AB AC BC a fronteira de RABC . e 4. Interior de uma regio triangular o conjunto de pontos de uma regio triangular que a e a no pertencem a sua fronteira. a Assim, RABC ABC o interior de RABC (gura 13.1, ` direita) e ser representado por e a a int(RABC ).

119

Denies e axiomas co
5. Regio poligonal o conjunto unio de a e a um nmero nito de regies triangulares que, duas u o a duas, no tm pontos interiores em comum. a e Assim, se Ri , i = 1, 2, ..., n, so regies triangua o lares tais que int(Ri ) int(Rj ) = , i = j; i, j =
n

1, 2, ..., n, o conjunto R =
i=1

Ri uma regio polie a

gonal (gura 13.2). 6. Um ponto de uma regio poligonal dito a e ser ponto interior ` regio, se existe uma regio a a a Figura 13.2: Regio poligonal. a triangular contida na regio poligonal e que contm a e o ponto no seu interior (gura 13.5). Interior de uma regio poligonal o conjunto de todos os seus pontos interiores. a e 7. Fronteira de uma regio poligonal o cona e junto dos pontos da regio que no so pontos intea a a riores. Daremos, a seguir, os axiomas que denem o conceito de rea de uma regio poligonal; para isso, a a chamaremos de o conjunto de todas as regies o P poligonais do plano. Q

Figura 13.3: P ponto interior e Q e ponto de fronteira.


 

Axioma 19 Existe uma funo a : R+ que, a cada regio poligonal R , associa um ca a nmero real positivo, a (R) . O nmero a (R) ser chamado de rea da regio poligonal R. u u a a a
 #
n

Axioma 20 Se R =
i=1

Ri (n 2), onde Ri , i = 1, 2, ..., n, so regies poligonais tais que a o


n

int(Ri ) int(Rj ) = , i = j, i, j = 1, 2, ..., n, ento a(R) = a


i=1

a(Ri ).
! 

" 

Axioma 21 Se um tringulo ABC congruente ao tringulo ABC, ento a(RABC ) = a e a a a(RABC ).


  

Axioma 22 Se ABCD um quadrado, cujo lado mede 1, ento a(RABCD ) = 1. e a




120

Area de um quadrado
13.1 Observao. Se R1 e R2 so regies poligonais tais que R1 R2 , ento a(R1 ) a(R2 ). ca a o a 13.2 Observao. No dif ver que todo pol ca a e cil gono convexo determina uma regio poligonal. a 13.3 Observao. No lugar de dizer rea da regio poligonal cuja fronteira um dado pol ca a a e gono, diremos somente rea de um dado pol a gono. 13.4 Exerc cio. Prove as declaraes das observaes 13.1 e 13.2. co co 13.5 Exerc cio. Prove que dois retngulos congruentes tm a mesma rea. a e a

13.2

Area de um quadrado

Quadrado um quadriltero que tem os quatro lados congruentes e os quatro ngulos retos. e a a 13.6 Problema. Calcular a rea de um quadrado Q, cujo lado mede p, sendo p um nmero a u inteiro positivo. Soluo: Sabemos que Q uma regio poligoca e a nal, pois todo quadrado um pol e gono convexo. Dividamos cada lado do quadrado Q em p partes congruentes, atravs de paralelas aos lados (gura 13.4). e Ficam, assim, determinadas p2 regies quadradas o 2 , cada uma com lado de medida Qi , i = 1, 2, ..., p
p2

Qi

p divises

unitria e tais que Q = a


i=1

Qi , int(Qi )int(Qj ) = ,

i = j, i, j = 1, 2, ..., p2 . Pelo axioma 19, existe a(Q) > 0, sendo a(Q) a rea do quadrado. Pelo axioma 20, segue que a
n

a(Q) =
i=1

a(Qi ) e, pelo axioma ??, sabemos que

p divises

a(Qi ) = 1, i = 1, 2, ..., p2 . Assim, a(Q) = p2 .

Figura 13.4: As regies Qi . o

13.7 Problema. Calcular a rea de um quadrado Q, cujo lado mede r, sendo r um nmero a u racional positivo. Consideremos, inicialmente, um caso particular deste problema em que o quadrado Q tem e u lado medindo 1 , onde q um nmero inteiro positivo. q Um quadrado de lado unitrio pode ser olhado como a unio de q 2 quadrados justapostos a a Qi , i = 1, 2, ..., q 2 , com int(Qi ) int(Qj ) = , i = j, i, j = 1, 2, ..., q 2 , tais que todos os Qi so a congruentes entre si e tm lados medindo 1 . Tomemos, ento, Qi congruente a Q, i = 1, 2, ..., q 2 . e a q
q2

Assim,
i=1

a(Qi ) = 1, ou seja, a(Q) q 2 = 1; assim, a(Q) =

1 . q2

Soluo: Seja Q um quadrado, cujo lado mede r, r racional positivo. Assim, existem p e q, ca inteiros positivos, tais que r = p . Cada lado de Q pode ser dividido em p partes congruentes, q o atravs de paralelas aos lados, tendo, cada uma, medida 1 . Ficam, assim, determinadas p2 regies e q

121

Area de um retngulo a
quadradas, cada uma com lado medindo 1 . A rea de cada um desses quadrados, como vimos a q acima, e
1 . q2

Como Q a unio desse p2 quadrados congruentes, segue que a(Q) = p2 e a


p q 2

1 q2

logo, a(Q) =

. Assim, a(Q) = r2 .

13.8 Problema. Calcular a rea de um quadrado Q, cujo lado mede l, sendo l um nmero a u irracional. o e Soluo: Inicialmente, conjecturamos que a(Q) = l2 . Mostremos que essa frmula verca dadeira utilizando um racioc nio indireto como segue: se k1 qualquer nmero tal que k1 < l2 , e u mostraremos que k1 < a(Q). Por outro lado, se k2 qualquer nmero tal que l2 < k2 , e u mostraremos que a(Q) < k2 . Isto implicar que a(Q) no poder ser um nmero k1 menor a a a u 2 e nem um n mero k maior que l2 , da dever ser igual a l2 (tricotomia!). do que l u , a 2 2 (i) Tomemos ento 0 < k1 < l2 . Seja r1 um nmero racional tal que k1 < r1 < l2 . No interior a u e de Q tomemos um quadrado Qr1 , com lado medindo r1 . Como r1 racional, segue, do problema 2 a ca 13.7 que a(Qr1 ) = r1 . Como Qr1 est contido no interior de Q, devemos ter (observao 13.1) 2 a(Qr1 ) a(Q), isto , r1 a(Q). Como k1 < r1 , conlcu e 2 mos que k1 < a(Q). Assim, todo nmero u 2 , tal que k < a(Q). real k1 , menor do que l e 1 (ii) Tomemos agora um nmero real k2 tal que l2 < k2 . Seja r2 um nmero racional tal que u u 2 l2 < r2 < k2 . Tomemos um quadrado Qr2 , com lado medindo r2 , de modo que Q esteja contido no interior 2 de Qr2 . Como r2 racional, segue, do problema 13.7 que a(Qr2 ) = r2 . Como Q est contido no e a 2 . Como r 2 < k , interior de Qr2 , devemos ter (observao 13.1) a(Q) a(Qr2 ), isto , a(Q) r2 ca e 2 2 conclu mos que a(Q) < k2 . Assim, todo nmero real k2 , maior do que l2 , tal que a(Q) < k2 . u e 2 Se a (Q) < l2 , ento o nmero k1 = a(Q)+l seria tal que a (Q) < k1 < l2 , contradizendo (i) . a u 2 Se a (Q) > l2 , ento o nmero k2 = a(Q)+l seria tal que l2 < k1 < a (Q) , contradizendo (ii) . a u 2 Da tricotomia dos nmeros reais, segue que a (Q) = l2 . u Desde que qualquer nmero real positivo, l, pode ser medida do lado de um quadrado e todo u nmero real racional ou irracional, dos problemas 13.7 e 13.8, segue que a rea de qualquer u e a quadrado, Q, de lado com medida l expressa pela frmula e o a(Q) = l2 isto , a rea de um quadrado igual ao quadrado da medida de seu lado. e a e
2

13.3

Area de um retngulo a

Retngulo um quadriltero que tem os quatro ngulos retos. a e a a 13.9 Problema. Calcular a rea de um retngulo R, cujos lados medem b e h. a a Soluo: Seja R = ABCD uma regio retangular cujos lados medem b e h, isto , AB = ca a e CD = b e BC = AC = h. Sobre os lados AB e BC constru mos quadrados ABEF e BCGH, respectivamente, de modo a no conterem o ponto D. Os prolongamentos dos lados EF e GH se a interceptam no ponto I, determinando o quadrado DGIF (e da o retngulo BHIE, conforme a a gura 13.5).

122

Area de paralelogramos e tringulos a


Introduzindo as notaes Q = DGIF (quadrado co de lado medindo b + h), R1 = ABEF, R2 = BHIE, R3 = ABCD e R4 = BCGH para indicar as regies poligonais obtidas, observamos que o int(Ri ) int(Rj ) = , i = j, i, j = 1, 2, 3, 4 e que Q = R1 R2 R3 R4 . Assim, a(Q) = a(R1 ) + a(R2 ) + a(R3 ) + a(R4 ) (13.1)

R1

R2

A
R3

B
R4

H G

Como R2 e R3 so retngulos congruentes ao a a retngulo R = ABCD, segue que a a(R) = a(R2 ) = a(R3 ). (13.2)

Figura 13.5: Problema 13.9.

Por outro lado, a(Q) = (b + h)2 , a(R1 ) = b2 a(R4 ) = h2 . Substitu ndo (13.2) e (13.3) em (13.1), obtemos (b + h)2 = b2 + 2a(R) + h2 e, da , a(R) = bh. Isto , a rea de um retngulo igual ao produto dos comprimentos, b e h, de seus lados. e a a e (13.3)

13.4

Area de paralelogramos e tringulos a


Q . D C

Paralelogramo um quadriltero em que os lae a dos opostos so paralelos. A um dos lados de a um paralelogramo chamaremos de base e ao segmento de perpendicular comum ` base e ao a lado oposto (ou ao seu prolongamento), cujas extremidades repousam sobre estes, chamaremos altura (gura 13.6).

A B 13.10 Problema. Dado um paralelogramo ABCD, onde AB sua base, com comprie mento b, e P Q sua altura, com comprimento Figura 13.6: Paralelogramo ABCD com base AB e altura P Q. h, calcular sua rea. a
Soluo: O paralelogramo ABCD est contido num retngulo AECF, como indicado na ca a a fcil provar que os tringulos ADF e gura 13.7 (certique-se da veracidade de tal armao). E a ca a CBE so congruentes; logo, pelo axioma 22, eles tm a mesma rea, isto , a(RADF ) = a(RCBE ). a e a e

. P

123

Area de paralelogramos e tringulos a

Q
.
.

Por outro lado, RAECF = RABCD RADF RCBE e, do axioma 20, segue que a(RAECF ) = a(RABCD )+a(RADF )+a(RCBE ) (13.4) Consideremos um retngulo BECG com a base igual a BE e altura EC. A diagonal BC, divide-o em dois tringulos CBE e BCG, a onde BCG congruente a ADF , e, da e , a(BCG) = a(ADF ). Pelo axioma 20,

Figura 13.7: Problema 13.10.

a(RBECG ) = a(RCBE ) + a(RCBG ) = a(RCBE ) + a(RADF ). Como a(RBECG ) = BE CE, temos a(RCBE ) + a(RADF ) = BE CE. Como AECF um retngulo, do problema 13.9, segue que e a a(RAECF ) = AE CE. De (13.4), (13.5) e (13.6), temos que AE CE = a(RABCD ) + BE CE e, como AB = AE BE, temos que a(RABCD ) = AB CE. Uma vez que P Q = CE (!), temos que a(RABCD ) = AE P Q = bh Assim, a rea do paralelogramo ABCE, cuja base mede b e altura mede h, dada pela a e frmula o a(RABCD ) = b h isto , a rea de um paralelogramo igual ao produto do comprimento de qualquer uma de suas e a e bases pelo comprimento da correspondente altura. 13.11 Observao. A rea de um paralelogramo independente da base escolhida. ca a e 13.12 Exerc cio. Dadas duas retas r e s, paralelas, e um segmento AB, contido em r; todos os paralelogramos ABCD, com C e D pertencentes ` s, tm a mesma rea (gura 13.11). a e a (13.6) (13.5)

124

Area de paralelogramos e tringulos a

D'

C'

Figura 13.8: Paralelogramos de mesma base e mesma altura.

Dado um tringulo ABC, ` um de seus lados chamaremos de base. a a 13.13 Problema. Dado um tringulo ABC, onde AB sua base com comprimento b e CH a e a sua altura, relativa ao lado AB, com comprimento h, calcular sua rea.

Figura 13.9: Paralelogramo ABCD.

Soluo: Seja ABC o tringulo ca a dado (gura 13.9); pelos vrtices B e C e tracemos paralelas a AC e AB, respectivamente. Essas retas interceptam-se num ponto D e do origem a um paralea logramo ABCD. Consideremos a altura CH desse paralelogramo; observemos que CH = h e AB = b. Pelo axioma 20, uma vez que RABCD = RABC RDCB , temos que

a(RABCD ) = a(RABC ) + a(RDCB ). Como ABCD um paralelogramo com base AB = b e altura CH = h, do problema 13.10, e temos que a(RABCD ) = b h, e da , 1 a(RABC ) = b h 2 isto , a rea de um tringulo igual ` metade do produto do comprimento da base pelo e a a e a comprimento da altura correspondente. 13.14 Observao. A rea de um tringulo independente do par (base, altura corresponca a a e dente) escolhido. 13.15 Exerc cio. Dadas duas retas r e s, paralelas, e um segmento AB contido em r; todos os tringulos com base AB e um vrtice C pertencente a s, tm a mesma rea (gura 13.10). a e e a

125

Area de um c rculo e de um setor circular

C'

Figura 13.10: Tringulos de mesma base e mesma altura. a

13.16 Observao. Uma vez que qualquer regio poligonal pode ser subdividida em tringulos, ca a a sua rea pode ser obtida como a soma das reas dos tringulos que a compe. a a a o 13.17 Exerc cio. Se ABCD um trapzio (gura 13.11) com base maior AB = b1 , base menor e e CD = b2 e altura CH = h, mostre que sua rea dada pela frmula a e o b1 + b2 h. 2

.
A H
Figura 13.11: Trapzio ABCD. e

13.5

Area de um c rculo e de um setor circular

13.18 Denio. Dada uma circunferncia C (O, R), chamaremos de c ca e rculo de centro O e raio R ao conjunto de todos os pontos X do plano, tais que OX R. Considere um c rculo qualquer de centro O e raio R. Pelo exerc 10.55, para cada natural cio n, existe um pol gono regular de n lados inscrito na circunferncia C (O, R). Denotaremos o e lado e a rea deste pol a gono por ln e An , respectivamente. A cada lado do pol gono est associado um tringulo issceles com um vrtice em O, laterais a a o e de medida R e alturas relativas `s laterais de medida R sen n , sendo n o ngulo de vrtice O a a e de cada um destes tringulos. a

126

Equivalncia plana e
Desde que n =
2 n ,

de An = n R2 senn 2 sen
2 n 2 n

temos que An = R2 Como


n

lim

sen

2 n 2 n

=1

e, ` medida que n cresce a rea dos pol a a gonos inscritos se aproxima cada vez mais da rea do a c rculo, adotamos a seguinte denio: ca 13.19 Denio. A rea de um c ca a rculo de raio R R2 . e 13.20 Denio. Dada uma circunferncia C (O, R) , tome dois pontos A, B sobre ela. A ca e interseo de OA,B OB,A com o c ca rculo de centro O e raio R chamada de setor circular e AB (gura (13.12). 13.21 Denio. Um setor circular no qual AB um dimetro de C (O, R) , recebe o nome de ca e a semic rculo. Observe que ao multiplicarmos o ngulo central por a qualquer natural n, a rea do setor circular dever car a a multiplicada por n. Alm disso, a rea do setor circular e a aumenta ` medida que o ngulo central cresce, isto , a a a e a rea uma funo crescente do ngulo central. Pelo teoe ca a rema fundamental da proporcionalidade (vide [14]), temos que a rea a de um setor circular diretamente propora e cional ` medida do ngulo central, isto , a = k , sendo a a e k a constante de proporcionalidade. Ela pode ser deter2 minada observando que para = temos a = R ; logo, 2 2 ca k = R . Por isso, adotamos a seguinte denio: 2

A O B

Figura 13.12: Setor circular.

13.22 Denio. A rea de um setor circular de um c ca a rculo de raio R e cujo ngulo central a 2 mede radianos R . e 2

13.6

Equivalncia plana e

13.23 Denio. Dizemos que dois pol ca gonos so equivalentes, quando as regies poligonais a o por eles determinadas possuem a mesma rea. a 13.24 Problema. Dividir um tringulo em quatro tringulos equivalentes. a a 13.25 Problema. Construir um tringulo cuja rea seja o qudruplo da de um tringulo dado. a a a a 13.26 Problema. Construir um tringulo retngulo equivalente a um tringulo dado. a a a 13.27 Problema. Construir um quadrado equivalente a um tringulo dado. a

127

Exerc cios complementares

13.7

Exerc cios complementares

13.28 Exerc cio. Mostre que a rea de um losango igual ` metade da medida do produto a e a das diagonais. 13.29 Exerc cio. Mostre que se dois tringulos so semelhantes ento a razo entre as suas a a a a a reas igual ao quadrado da razo de semelhana. e a c 13.30 Exerc cio. Mostre que se dois pol gonos so semelhantes ento a razo entre as suas a a a a reas igual ao quadrado da razo de semelhana. e a c 13.31 Exerc cio. Prolongando os lados no paralelos de um trapzio, considere a sua altura a e como a diferena entre as alturas de dois tringulos. Obtenha, assim, uma frmula para a rea c a o a do trapzio. e 13.32 Exerc cio. Demonstre que as medidas das trs alturas de um tringulo so inversamente e a a proporcionais as medidas dos lados correspondentes. 13.33 Exerc cio. Por um ponto arbitrrio da diagonal de um paralelogramo trace uma paralela a a cada um dos lados no paralelos, decompondo-o, assim, em quatro paralelogramos menores. a Dois deles tm reas iguais. Identique-os e prove a armao. e a ca 13.34 Exerc cio. Sejam E, F pontos mdios dos lados no paralelos do trapzio ABCD. Prove e a e que a (RABF ) + a (RCDF ) = a (RAEF ) + a (RCEF ) . 13.35 Exerc cio. Prove que a rea do quadrado inscrito numa circunferncia igual ` metade a e e a da rea do quadrado circunscrito na mesma circunferncia. a e 13.36 Exerc cio. Prove que as trs medianas de um tringulo o decompem em seis tringulos e a o a de mesma rea. a 13.37 Exerc cio. Os pontos mdios de um quadriltero convexo qualquer so vrtices de um e a a e paralelogramo cuja rea a metade da rea do quadriltero dado. Mostre isso. a e a a 13.38 Exerc cio. Calcule a rea de um hexgono regular de lado l. a a 13.39 Exerc cio. Utilize o exerc 7.71, para dar uma outra demonstrao para o Teorema cio ca de Pitgoras. A gura 13.13 d a sugesto. a a a 13.40 Exerc cio. D uma outra prova para o Teorema de Pitgoras, utilizando o desenho ` e a a esquerda, na gura 13.14. 13.41 Exerc cio. D uma outra prova para o Teorema de Pitgoras utilizando o desenho ` e a a direita na gura 13.14. Esta foi a prova feita por Bhskara. a 13.42 Exerc cio. D uma outra prova para o Teorema de Pitgoras utilizando o desenho da e a gura 13.15 ` esquerda . a

128

Exerc cios complementares

c b a a b c a a

b c

Figura 13.13: Exerc 13.39. cio

a c c

b a

b
Figura 13.14: Exerc cios 13.40 e 13.41.

13.43 Exerc cio. D uma outra prova para o Teorema de Pitgoras utilizando a gura 13.15 e a a ` direita. Esta prova foi feita por Leonardo da Vinci.

13.44 Exerc cio. D uma outra prova para o Teorema de Pitgoras utilizando a gura da capa e a deste texto. Esta prova foi feita por Euclides. 13.45 Exerc cio. Num tringulo ABC, se o ngulo A obtuso (agudo) ento a rea do a a e a a quadrado constru sobre o lado BC maior (menor) do que a soma das reas dos quadrados do e a constru dos sobre os lados AB e AC. Conclua, da a rec , proca do Teorema de Pitgoras. a 13.46 Exerc cio. (Generalizao do Teorema de Pitgoras) Prove que se trs pol ca a e gonos semelhantes esto dispostos sobre os lados de um tringulo retngulo, de tal modo que os lados desse a a a tringulo sejam homlogos uns aos outros, ento a soma das reas dos pol a o a a gonos situados sobre os catetos igual ` rea do pol e aa gono situado sobre a hipotenusa. Observe que se os pol gonos forem quadrados temos o Teorema de Pitgoras. a 13.47 Exerc cio. (Lnulas de Hipcrates) Sobre cada cateto de um tringulo retngulo conu o a a stri-se um semi c o rculo justaposto ao tringulo e, sobre a hipotenusa um semic a rculo contendo o tringulo. Prove que a soma das reas dos semic a a rculos, situados sobre os catetos, igual ` e a

129

Exerc cios complementares

Figura 13.15: Exerc cios 13.42 e 13.43.

a rea do semic rculo que contm o tringulo (os dois ultimos exerc e a cios podem ser generalizados para guras semelhantes quaisquer desenhadas sobre os lados de um tringulo retngulo). a a 13.48 Exerc cio. Mostre que a rea de um tringulo de semiper a a metro p e lados de medida a, b, c igual ` p (p a) (p b) (p c). Esta chamada a frmula de Heron para a rea de um e a e o a tringulo. a 13.49 Exerc cio. Mostre que a rea de um tringulo igual a pr, sendo p o semiper a a e metro e r o raio da circunferncia inscrita. e 13.50 Exerc cio. Mostre que a rea de um tringulo igual a a a e lados do tringulo e R o raio da circunferncia circunscrita. a e
abc 4R ,

sendo a, b, c as medidas dos

13.51 Exerc cio. Mostre que a rea de um tringulo ABC igual a a a e medidas dos lados opostos aos vrtices B, C, respectivamente. e

1 2 bcsenA,

sendo b, c as

13.52 Exerc cio. Calcule a rea hachurada na gura 13.16 conhecendo o raio R do c a rculo exterior. 13.53 Exerc cio. Calcule a rea hachurada na gura 13.17 conhecendo o lado l do quadrado. a 13.54 Exerc cio. Calcule a rea hachurada na gura 13.18 conhecendo o raio R do c a rculo exterior. 13.55 Exerc cio. Calcule a rea hachurada na gura 13.19 dados AB = R e sabendo que a AB = 3 BC.

130

Exerc cios complementares

Figura 13.16: Exerc 13.52. cio

Figura 13.17: Exerc 13.53. cio

A
Figura 13.18: Exerc 13.54. cio

Figura 13.19: Exerc 13.55. cio

13.56 Exerc cio. Calcule a rea hachurada na gura 13.20 dados AG = R e sabendo que todos a os pontos consecutivos so eqidistantes. a u 13.57 Exerc cio. Dada uma circunferncia C (O, r) , considere uma de suas tangentes a qual e chamaremos de t. Pelo exerc 10.12, C (O, r) est contida em um dos semi planos determinados cio a por t. Mostre que a interseo de todos os semi planos Ot , sendo t tangente a C (O, r) , o ca e c rculo de centro O e raio r (isto implica que o c rculo um conjunto convexo, conforme o e exerc 2.44). cio 13.58 Exerc cio. Mostre que a unio de todas as cordas da circunferncia C (O, R) o c a e e rculo de centro O e raio R. 13.59 Exerc cio. Construir um tringulo issceles equivalente a um tringulo dado. a o a 13.60 Exerc cio. Construir um tringulo retngulo equivalente a um quadrado dado. a a 13.61 Exerc cio. Construir um tringulo equivalente a um retngulo dado. a a 13.62 Exerc cio. Construir um tringulo equivalente a um losango dado. a 13.63 Exerc cio. Construir um tringulo equivalente a um paralelogramo dado. a

131

Exerc cios complementares

Figura 13.20: Exerc 13.56. cio

13.64 Exerc cio. Construir um tringulo equivalente a um trapzio dado. a e 13.65 Exerc cio. Sejam A, B, C, D vrtices consecutivos de um pol e gono com n lados. Pelo ponto B trace uma paralela ` diagonal AC. Seja E a interseo dessa paralela com o prolongaa ca mento do lado DC. Substitua os lados AB, BC e CD por AE e ED. a Mostre que o pol gono, com n 1 lados assim obtido tem a mesma rea que o anterior. b Mostre que pode-se construir a um tringulo com a mesma rea que um pol a a gono dado. 13.66 Exerc cio. Dado um pol gono convexo qualquer com cinco lados, construa um tringulo a equivalente. 13.67 Exerc cio. Trace por um ponto P arbitrrio de um tringulo ABC uma reta que divida-o a a em dois pol gonos equivalentes. 13.68 Exerc cio. Faa a quadratura de um tringulo (fazer a quadratura de um pol c a gono signica construir um quadrado equivalente). 13.69 Exerc cio. Trace por um ponto P de um tringulo ABC duas retas que o divida em a trs pol e gonos equivalentes. 13.70 Exerc cio. Trace por um ponto P de um quadriltero, uma reta que divida o quadriltero a a em dois pol gonos equivalentes. 13.71 Exerc cio. Por um dos vrtices de um quadriltero, trace duas retas que o divida em e a trs pol e gonos equivalentes. 13.72 Exerc cio. Descreva um mtodo para efetuar a quadratura de um pol e gono de n lados (sugesto: use os exerc a cios 13.65 e 13.68).

132

Apndice A e
Conforme dissemos na apresentao deste texto, toda demonstrao em geometria deve estar ca ca baseada nos axiomas ou nos teoremas j demonstrados a partir dos axiomas. As ilustraes a co apenas aguam a intuio. c ca A no observao disso pode nos conduzir a situaes inusitadas, como o a ca co Teorema Todo tringulo issceles. a e o Prova: Sejam ABC um tringulo qualquer, b a bissetriz do ngulo A e m a mediatriz a a do lado BC. Do teorema 2.9 segue que as retas b, m podem ser coincidentes, paralelas ou concorrentes.

E B

D
. .

F E C B
.

F C

B E
.

C
.

1o caso: Se b, m so paralelas ento, pelo teorema 6.11, b bissetriz e altura relativa a a a e um mesmo vrtice, logo, pelo exerc 4.30, ABC um tringulo issceles e, do exerc 4.19 e cio e a o cio resulta que b e m so coincidentes, contradizendo o fato de b e m serem paralelas. a 2o caso: Se b, m so concorrentes, chamemos de D o ponto comum a ambas. Sejam E, F os a ps das perpendiculares baixadas por D a AC, respectivamente. e Existem trs possibilidades para o ponto D: ele pode estar no interior, no exterior ou sobre o e tringulo ABC. Em qualquer um dos casos, vale a argumentao que segue. Como b bissetriz a ca e do ngulo A, do exerc 5.28 segue que a cio DE DF . Pelo caso LAA de congruncia de tringulos, ADE ADF, logo, e a AE AF . (2) (1)

133

Apndice A e
Como D est sobre a mediatriz de BC, do exerc 5.27 resulta que a cio BD CD. (3)

De (1) e (3) segue que BDE CDF (pelo caso hipotenusa-cateto de congruncia de e tringulos retngulos), logo, a a BE CF . (4) As congruncias (2) e (4) implicam que AB AC. Como ABC um tringulo issceles, e e a o do exerc cio 4.19 resulta que b e m so coincidentes, contradizendo o fato de b e m serem a concorrentes. Portanto, s pode ocorrer de b e m serem coincidentes, o que implica que o tringulo ABC o a issceles, conforme o exer 4.30. e o cio

134

Indice Remissivo
a rea de um quadrado, 122 de um retngulo, 122 a de um setor circular, 127 de um trapzio, 126 e de um tringulo, 125, 130 a do c rculo, 127 a real de um paralelogramo, 123 a ngulo, 28 agudo, 31 bissetriz de um, 30 central, 78 complemento de um, 30 de Brocard, 99 externo de um pol gono convexo, 59 externo de um tringulo, 43 a inscrito, 79 interior de um, 33 interno de um pol gono convexo, 59 lados de um, 28 medida de um, 28 nulo, 28 obtuso, 31 ponto interior de um, 33 prolongamento de um, 30 raso, 28 reto, 31 semi-inscrito, 81 suplemento de um, 30 unidades de medida de, 29 vrtice de um, 28 e a ngulos adjacentes, 30 alternos externos, 52 alternos internos, 52 colaterais externos, 52 colaterais internos, 52 complementares, 30 congruentes, 35 consecutivos, 30 correspondentes, 52 correspondentes em dois tringulos congrua entes, 36 homlogos em dois tringulos semelhantes, o a 69 opostos de um quadriltero, 60 a opostos pelo vrtice, 31 e suplementares, 30 transporte de, 36 arco capaz, 82 complementar, 82 comprimento de um, 89 correspondente, 81 correspondente a um ngulo, 81 a maior, 78 medida de um, 79 subentendido por um ngulo, 79 a bola aberta, 27 c rculo, 126 circunferncia e semi-reta tangente a uma, 78 circunferncia, 27, 77 e centro de uma, 77 circunscrita, 83 comprimento de uma, 89 corda de uma, 77 dimetro, 77 a inscrita, 85 pontos exteriores a uma, 27

135

INDICE REMISSIVO
pontos interiores a uma, 27 raio de uma, 77 reta secante a uma, 77 tangente a uma, 78 circunferncias e tangentes, 91 conjunto convexo, 20 limitado, 27 cosseno de um nmero, 86 u

INDICE REMISSIVO

altura de um, 123 base de um, 123 plano, 13 pol gono, 57 aptema de um, 86 o centro de um, 86 circunscrit vel, 85 circunscrito, 85 convexo, 59 diagonal de um, 58 equingulo, 58 a equiltero, 58 a Desigualdade triangular, 46 exterior de um, 59 disco inscrit vel, 83 aberto, 27 inscrito, 83 distncia a interior de um, 59 de ponto a reta, 45 per metro de um, 59 entre dois pontos, 23 regular, 59 entre duas retas, 54 semiper metro de um, 59 pol gonos funo ca congruentes, 74 congruncia, 36 e semelhantes, 74 cosseno, 87 poligonal reexo, 48 a a ngulo de uma, 57 semelhana, 69, 74 c lados de uma, 57 seno, 87 linha, 57 vrtice de uma, 57 e grado, 29 ponto, 13 grau, 29 de contato, 91 Lnulas de Hipcrates, 129 u o de tangncia, 78 e lados interseo de duas retas, 14 ca correspondentes em dois tringulos congrua potncia de, 82 e entes, 36 reexo de um ponto em relao a uma reta, ca homlogos em dois tringulos semelhantes, o a 48 69 pontos lei dos cossenos, 88 colineares, 14 lei dos senos, 87 de Brocard, 97 losango, 61 homlogos de dois pol o gonos semelhantes, 74 mediatriz, 32 pontos em lados opostos em relao a uma ca reta, 17 nmero ureo, 27 u a pontos em um mesmo lado em relao a ca p da perpendicular, 45 e uma reta, 17 paralelas quadrado, 61 axioma das, 51 paralelogramo, 60 quadriltero a

136

INDICE REMISSIVO
a ngulos opostos de um, 60 convexo, 60 lados opostos de um, 60 vrtices opostos de um, 60 e quarta proporcional, 114 radiano, 29 razo de semelhaa a c de pol gonos, 74 de tringulos, 69 a regio poligonal, 119 a a rea de uma, 120 fronteira de uma, 120 interior de uma, 120 ponto interior de uma, 120 regio triangular, 119 a fronteira de uma, 119 interior de uma, 119 retngulo, 60 a reta, 13 de Euler, 97 dos centros, 91 retas concorrentes, 14 paralelas, 44 perpendiculares, 31

INDICE REMISSIVO
semi-reta oposta, 17 semi-retas perpendiculares, 31 semicircunferncia, 78 e semiplano, 17 origem, 17 seno de um nmero, 86 u setor circular, 127 sistema de coordenadas da reta coordenada de um ponto em um, 24 origem de um, 24 parte negativa da reta em um, 24 parte positiva da reta em um, 24 sistema de coordenadas do plano, 55 abscissa do ponto em um, 55 cartesianas ortogonais, 55 coordenadas de um ponto em um, 55 eixo das abscissas de um, 55 eixo das ordenadas de um, 55 ordenada do ponto em um, 55 origem de um, 55

teorema da bissetriz interna, 66 de Ceva, 95 de Euler, 96 segmento, 16 de Pitgoras, 73 a comprimento de um, 23 de Tales, 66 construt vel, 108 do ngulo externo, 43 a extremidades, 16 trapzio, 61 e medida de um, 23 base de um, 61 obl quo, 45 issceles, 61 o ponto mdio de um, 26 e laterais de um, 61 projeo de um, 45 ca retngulo, 61 a segmento nulo, 16 tringulo, 16 a segmentos a ureo, 75 congruentes, 35 a ngulo interno de um, 28 homlogos de dois pol o gonos semelhantes, altura de um, 38 74 baricentro de um, 94 transporte de, 36 base de um, 125 semi c rculo, 127 bissetriz de um, 38 semi-reta, 17 circuncentro de um, 83 coordenada de uma, 29 equiltero, 38 a origem, 17 excentro de um, 94 incentro de um, 86 que divide o semiplano, 29

137

INDICE REMISSIVO
interior de um tringulo, 20 a issceles, 38 o a ngulos da base de um, 38 base de um, 38 laterais de um, 38 vrtice de um, 38 e lados de um, 16 mediana de um, 38 ortocentro de um, 93 per metro de um, 47 retngulo, 38 a catetos de um, 38 hipotenusa, 38 semiper metro de um, 47 vrtices, 16 e tringulos a congruncia de tringulos retngulos, 47 e a a congruentes, 35 primeiro caso de congruncia de, 37 e quarto caso de congruncia de, 44 e segundo caso de congruncia de, 37 e semelhantes, 69 terceiro caso de congruncia de, 38 e vrtices e correspondentes em dois tringulos congrua entes, 36

INDICE REMISSIVO

138

Referncias Bibliogrcas e a
[1] Barbosa, J. L. M. Geometria Euclidiana Plana. Publicao da Sociedade Brasileira de ca Matemtica, Rio de Janeiro, 1997. a [2] Serro, A. N. Geometria no Plano - Parte A. Ao Livro Tcnico, Rio de Janeiro, 1968. a e [3] Serro, A. N. Geometria no Plano - Parte B. Ao Livro Tcnico, Rio de Janeiro, 1968. a e [4] Pogorelov, A.Geometry. Mir, Moscou, 1987. [5] Prasolov, V. V. Essays on numbers and gures. American Mathematical Society, 2000. [6] Greenberg, M. J.Euclidean and Non-Euclidean Geometry. W.H. Freeman, San Francisco, 1974. [7] Lima, E. L. Medida e Forma em Geometria. Publicao da Sociedade Brasileira de ca Matemtica, Rio de Janeiro, 1991. a [8] Wagner, E. Construes Geomtricas. Publicao da Sociedade Brasileira de Matemtica, co e ca a Rio de Janeiro, 1998. [9] Eves, H. Estudio de las Geometrias. Centro Regional de Ayuda Tecnica, Mxico, 1963. e [10] Boyer, C. B. Histria da Matemtica, Edgard Blcher, So Paulo, 1996. o a u a [11] Courant, R; Robbins, Herbert. O que a matemtica?, Cincia Moderna, So Paulo, 2000. e a e a [12] Heath, T. L. The Thirteen Books of the Elements, Dover, New York, 1956. [13] Cendberg, J. N. A Course in Modern Geometries. 1989. [14] Lima, E. L; Carvalho, P. C. C; Wagner, E; Morgado, A. C; A Matemtica do Ensino Mdio a e - Vol. 1 ; SBM, Rio de Janeiro, 2001. [15] Carvalho, B.A. Desenho Geomtrico, Ao Livro Tcnico, Rio de Janeiro, 1959. e e

139

Potrebbero piacerti anche